Pharmacology Final

Pataasin ang iyong marka sa homework at exams ngayon gamit ang Quizwiz!

Answer: A RATIONALE: Assuring a patient airway is the first step in providing care to any patient.

(18/27) A patient presents to the clinic with a swollen face and tongue, difficulty breathing, and audible wheezes after eating a peanut butter sandwich for lunch. What is the first action the nurse should take? a. Assure a patient airway b. Obtain an electrocardiogram (ECG) c. Administer 1 mg of 1:1000 epinephrine subQ. d. Start an IV of normal saline

Answer: C, E

(18/34) Where are beta-1 receptors located? (Select all that apply) a. gastrointestinal tract b. lungs c. kidneys d. brain e. heart

Answer: 1 RATIONALE: Furosemide is a loop diuretic. The medication causes a decrease in the client's electrolytes, especially potassium, sodium, and chloride. Administering furosemide to a client with low electrolyte levels could precipitate ventricular dysrhythmias. Options 2 and 4 reflect renal function. The cholesterol level is unrelated to the administration of this medication.

171.) A nurse is preparing to administer furosemide (Lasix) to a client with a diagnosis of heart failure. The most important laboratory test result for the nurse to check before administering this medication is: 1. Potassium level 2. Creatinine level 3. Cholesterol level 4. Blood urea nitrogen

Answer: 1 RATIONALE: Metoprolol (Lopressor) is classified as a beta-adrenergic blocker and is used in the treatment of hypertension, angina, and myocardial infarction. Baseline nursing assessments include measurement of BP and AP immediately before administration. If the systolic BP is below 90 mm/Hg and the AP is below 60 beats/min, the nurse should withhold the medication and document this action. Although the registered nurse should be informed of the client's vital signs, it is not necessary to do so immediately. The medication should not be administered because the data is outside of the prescribed parameters for this medication. The nurse should not administer half of the medication, or alter any dosages at any point in time.

195.) A nurse is caring for a client who is taking metoprolol (Lopressor). The nurse measures the client's blood pressure (BP) and apical pulse (AP) immediately before administration. The client's BP is 122/78 mm/Hg and the AP is 58 beats/min. Based on this data, which of the following is the appropriate action? 1. Withhold the medication. 2. Notify the registered nurse immediately. 3. Administer the medication as prescribed. 4. Administer half of the prescribed medication.

Answer: 1 RATIONALE: Prednisone may decrease the effect of oral hypoglycemics, insulin, diuretics, and potassium supplements. Option 2, a monoamine oxidase inhibitor, and option 3, a β-blocker, have their own intrinsic hypoglycemic activity. Option 4 decreases urinary excretion of sulfonylurea agents, causing increased levels of the oral agents, which can lead to hypoglycemia.

30.) A client with diabetes mellitus visits a health care clinic. The client's diabetes mellitus previously had been well controlled with glyburide (DiaBeta) daily, but recently the fasting blood glucose level has been 180 to 200 mg/dL. Which medication, if added to the client's regimen, may have contributed to the hyperglycemia? 1. Prednisone 2. Phenelzine (Nardil) 3. Atenolol (Tenormin) 4. Allopurinol (Zyloprim)

Answer: C RATIONALE: Administering oral doses with food (C) helps minimize GI discomfort. (A) would be appropriate only if changing the time of the dose corresponds to meal times while at the same time maintaining an appropriate time interval between doses. (B) would disrupt the dosing schedule, and could result in a nontherapeutic serum level of the medication. (D) should not be attempted before other interventions, such as (C), have been proven ineffective in relieving the nausea.

A client receiving albuterol (Proventil) tablets complains of nausea every evening with her 9 p.m. dose. What action should the nurse take to alleviate this side effect? A) Change the time of the dose. B) Hold the 9 p.m. dose. C) Administer the dose with a snack. D) Administer an antiemetic with the dose.

Answer: C

A client taking an oral theophylline preparation is due for her next dose and has a blood pressure of 100/50 mm Hg and a heart rate of 110. The client is irritable. What is the best action for the nurse to take? a. Continue to monitor the client. b. Call the health care provider. c. Hold the next dose of theophylline. d. Administer oxygen 2 lpm via nasal cannula.

ANS: C Pregnant women should avoid antiemetics during the first trimester of pregnancy because of possible teratogenic effects. The nurse should recommend nonpharmacologic measures such as saltines. If this is not effective, intravenous fluids may become necessary. Pregnant women should consult with their provider before taking prescription or over the-counter antiemetics.

A woman who is 2 months pregnant reports having morning sickness every day and asks if she can take any medications to treat this problem. The nurse will recommend that the patient take which action first? a. Contact the provider to discuss a possible need for intravenous fluids. b. Contact the provider to discuss a prescription antiemetic. c. Use nonpharmacologic measures such as saltines. d. Take over-the-counter antiemetics such as diphenhydramine.

Answer: B RATIONALE: When used for a client with pulmonary embolus, the therapeutic goal for warfarin therapy is a PT 1½ to 2½ times greater than the control, or an INR of 2 to 3 (B). A perfusion might be performed to monitor lung function, but not monthly (A). APTT is monitored for the client receiving heparin therapy (C). A blood level for Coumadin cannot be measured (D).

Following heparin treatment for a pulmonary embolism, a client is being discharged with a prescription for warfarin (Coumadin). In conducting discharge teaching, the nurse advises the client to have which diagnostic test monitored regularly after discharge? A) Perfusion scan. B) Prothrombin Time (PT/INR). C) Activated partial thromboplastin (APTT). D) Serum Coumadin level (SCL).

Answer: D

Nonselective beta blockers may be used to treat hypertension and A.chronic obstructive pulmonary disease (COPD). B. heart failure. C.heart block. D. supraventricular dysrhythmias.

Answer: A

Patient teaching for a patient being discharged on a beta blocker includes which statement? A. "If you take your pulse and it is less than 60, hold your medicine and call your health care provider for instructions." B."If you become dizzy, do not take your medication for 2 days and then restart on the third day." C."This medication may make you fatigued; increasing caffeine in your diet may help alleviate this problem." D. "Increase intake of green leafy vegetables to prevent bleeding problems that can be caused by this medication."

ANS: D The client should not stop taking this drug abruptly because it suppresses the adrenal gland's production of corticosteroids. This could create a life-threatening situation. The client can expect weight gain and hyperglycemia. Use of the drug with the onset of asthma symptoms is not characteristic of Deltasone. The client should take the drug with food.

The client has recently been placed on prednisone (Deltasone). What is the highest priority instruction the nurse will provide? a. "Expect to experience weight loss and hypoglycemia." b. "Use the drug with the onset of asthma symptoms." c. "Take the drug on an empty stomach." d. "Do not stop taking the drug abruptly."

ANS: D St. John's wort can increase the sedative effects of opioids. It does not enhance other side effects.

The emergency department nurse is caring for a patient who has received morphine sulfate for severe pain following an injury. The nurse performs a drug history and learns that the patient takes St. John's wort for symptoms of depression. The nurse will observe this patient closely for an increase in which opioid adverse effect? a. Constipation b. Pruritis c. Respiratory depression d. Sedation

Answer: B

The nurse administering donepezil (Aricept) to a patient understands that the expected therapeutic action of this drug is to A. relieve anxiety and restless behavior of the patient B. increase levels of acetylcholine in the brain by blocking its breakdown. C. block the effects of acetylcholine at the presynaptic neurons. D. help control associated urinary incontinence.

Answer: B RATIONALE: The term first-pass effect refers to the rapid hepatic inactivation of certain oral drugs. Drugs that undergo the first-pass effect often are administered parenterally. The therapeutic range is the range of drug level between the minimum effective concentration (MEC) and the toxic concentration. The biologic half-life is the time required for the amount of drug in the body to decrease by 50%. Plasma protein binding is involved with the transport of drugs through the bloodstream.

The nurse administers 100 mg of drug X by mouth. After the drug moves through the hepatic system, very little active drug is left in the general circulation as a result of what? A. Therapeutic range B. First-pass effect C. Biologic half-life D. Plasma protein binding

ANS: C Eight hours after the drug is given, there will be 400 mg left. Eight hours after that (16 hours), there will be 200 mg left. At 24 hours, there will be 100 mg left.

The nurse gives 800 mg of a drug that has a half-life of 8 hours. How much drug will be left in the body in 24 hours if no additional drug is given? a. None b. 50 mg c. 100 mg d. 200 mg

Answer: D

The nurse instructs the client to avoid which over-the-counter products when taking theophylline (Theo-Dur)? a. acetaminophen (Tylenol) b. echinacea c. diphenhydramine (Benadryl) d. St. John's wort

ANS: C Patients who take aspirin or nonsteroidal antiinflammatory drugs (NSAIDs) should be monitored closely for excessive bleeding when given thrombolytics. There are no contraindications or precautions for patients with diabetes or renal disease. Foods rich in vitamin K are of concern for patients taking warfarin.

The nurse is assessing a patient prior to administering thrombolytic therapy. Which is an important assessment for this patient? a. Determining whether the patient has a history of diabetes b. Finding out about a history of renal disease c. Assessing which medications are taken for discomfort d. Assessing whether the patient eats green, leafy vegetables

Answer: C RATIONALE: Tinnitus is an early sign of salicylate toxicity. The healthcare provider should be notified immediately (C), and the medication discontinued. (A and D) are not needed, and (B) is inaccurate.

The nurse is assessing the effectiveness of high dose aspirin therapy for an 88-year-old client with arthritis. The client reports that she can't hear the nurse's questions because her ears are ringing. What action should the nurse implement? A) Refer the client to an audiologist for evaluation of her hearing. B) Advise the client that this is a common side effect of aspirin therapy. C) Notify the healthcare provider of this finding immediately. D) Ask the client to turn off her hearing aid during the exam.

ANS: D Older HIV infected patients may have age-related comorbid illness that can complicate management of HIV infection.

The nurse is caring for a 55-year-old patient who has been HIV-infected for 15 years. The nurse understands that this patient a. has an increased risk of transmitting the HIV infection. b. is less likely to develop AIDS than younger persons with HIV infection. c. is less likely to respond to antiretroviral agents. d. may have comorbid illnesses that can complicate HIV.

ANS: D The interaction of furosemide and an aminoglycoside can produce ototoxicity in the patient. Tinnitus is a sign of ototoxicity. Dizziness can occur as a result of diuretic therapy but not necessarily as a result of this combination. Dysuria and nausea are not common signs of these drugs interacting.

The nurse is caring for a patient who is receiving furosemide (Lasix) and an aminoglycoside antibiotic. The nurse will be most concerned if the patient reports which symptom? a. Dizziness b. Dysuria c. Nausea d. Tinnitus

ANS: D Thiazide diuretics can cause hypokalemia, which enhances the effects of digoxin and can lead to digoxin toxicity. Thiazides can cause hypercalcemia.

The nurse is caring for a patient who is taking hydrochlorothiazide (HydroDIURIL) and digoxin (Lanoxin). Which potential electrolyte imbalance will the nurse monitor for in this patient? a. Hypermagnesemia b. Hypernatremia c. Hypocalcemia d. Hypokalemia

Answer: C

The nurse is caring for a patient with a theophylline level of 14 mcg/mL. What is the priority nursing intervention? A Increase the IV drip rate. B Monitor the patient for toxicity. C Continue to assess the patient's oxygenation. D Stop the IV for an hour then restart at lower rate.

ANS: D Certain classifications of medications have different effects in individuals whose genetic markers are predominantly characteristic of a certain biologic group. Persons of Asian descent may have a decreased response to some drugs because they are more likely to have higher levels of CYP2D6 enzymes.

The nurse is caring for a postoperative patient who is of Asian descent. The patient reports little relief from pain even while taking an opioid analgesic containing codeine and acetaminophen. What does the nurse suspect that this patient is exhibiting? a. Drug-seeking behavior b. Heightened pain perception c. Poor understanding of expected drug effects d. Rapid metabolism of one of the drug's components

ANS: C Older patients experience a higher risk of orthostatic hypotension when taking antihypertensive medications. Fall risk also increases with a need for increased trips to the bathroom. A fall risk protocol should be implemented. Increasing fluids and activity and limiting potassium are not indicated.

The nurse is caring for an 80-year-old patient who has just begun taking a thiazide diuretic to treat hypertension. What is an important aspect of care for this patient? a. Encouraging increased fluid intake b. Increasing activity and exercise c. Initiating a fall risk protocol d. Providing a low potassium diet

ANS: B Patients who take anticoagulants have an increased risk of hemorrhage. Older patients have an increased risk of falls that can lead to bleeding complications. Initiating a fall risk protocol is important. Warfarin does not affect blood pressure and would not cause orthostatic hypotension. Warfarin does not alter urine output.

The nurse is caring for an 80-year-old patient who is taking warfarin (Coumadin). Which action does the nurse understand is important when caring for this patient? a. Encouraging the patient to rise slowly from a sitting position b. Initiating a fall-risk protocol c. Maintaining strict intake and output measures d. Monitoring blood pressure frequently

ANS: C For the most part, drugs to treat AD do not result in improvement of symptoms but help slow the progress. The most realistic care plan for a patient with advanced AD is that they will receive appropriate and safe care.

The nurse is helping to develop a plan of care for a patient who has advanced Alzheimer's disease. The patient will be taking a new medication. Which is a realistic goal for this patient? a. Demonstrate improved cognitive function. b. Exhibit improved ability to provide self-care. c. Receive appropriate assistance for care needs. d. Show improved memory for recent events.

ANS: A Drugs given IM are absorbed faster in muscles that have more blood vessels, such as the deltoid, rather than those with fewer blood vessels, such as the gluteals. Subcutaneous routes are used when absorption needs to be slower and more sustained.

The nurse is preparing an injectable drug and wants to administer it for rapid absorption. How will the nurse give this medication? a. IM into the deltoid muscle b. IM into the gluteal muscle c. SubQ into abdominal tissue d. SubQ into the upper arm

ANS: C Liquid drugs are already in solution, which is the form necessary for absorption in the GI tract. The other forms must disintegrate into small particles and then dissolve before being absorbed.

The nurse is preparing to administer an oral medication and wants to ensure a rapid drug action. Which form of the medication will the nurse administer? a. Capsule b. Enteric-coated pill c. Liquid suspension d. Tablet

ANS: A Digoxin requires a loading dose when first prescribed.

The nurse is preparing to administer the first dose of digoxin (Lanoxin) to a patient and notes that the dose ordered is much higher than the usual recommended dose. Which action will the nurse perform? a. Administer the dose as ordered. b. Give the dose and monitor for toxicity. c. Hold the dose until reviewing it with the provider. d. Refuse to give the dose.

ANS: A Patients who have taken hydantoins for long periods might have an elevated blood sugar. The nurse should discuss this test with the provider.

The nurse is preparing to assist with blood collection on a newly admitted patient who has been taking phenytoin for several years. The provider has ordered a complete blood count and liver function tests. Which other blood test will the nurse discuss with the provider? a. Blood glucose b. Coagulation studies c. Renal function tests d. Serum electrolytes

ANS: B Ginkgo can prolong bleeding time and therefore should be discontinued 2 weeks prior to surgery.

The nurse is providing preoperative education to a patient who will have surgery in several weeks. The patient denies taking anticoagulant medications but reports using herbal supplements. Which herb would cause the nurse to be concerned? a. Echinacea b. Ginkgo biloba c. Kava d. Sage

ANS: D Anticholinergic medications cause dry mouth, so patients should be advised to use gum or lozenges to counter this side effect. Anticholinergics cause increased heart rate and increased blood pressure. Anticholinergics can cause urinary retention so patients should not increase fluid intake.

The nurse is teaching a patient about the use of an anticholinergic medication. What information will the nurse include when teaching this patient about this medication? a. "Check your heart rate frequently to monitor for bradycardia." b. "Drink extra fluids while you are taking this medication." c. "Rise from a chair slowly to avoid dizziness when taking this drug." d. "Use gum or lozenges to decrease dry mouth caused by this drug."

Answer: B RATIONALE: When trough levels remain elevated, aminoglycosides are unable to diffuse out of inner ear cells, thus exposing the cells to the medication for an extended time. Prolonged exposure (i.e., high trough levels), rather than brief exposure to high levels, underlies cellular injury.

The nurse knows that there is an increased risk of ototoxicity in a patient receiving an aminoglycoside if which level is high? A Concentration B Trough C Peak D Dose

ANS: A, B, E The patient receiving an antihypertensive medication should be warned to rise slowly to avoid orthostatic hypotension. Patients should be counseled to continue to make lifestyle changes, including decreasing sodium. Often, more than one medication is required. Patients should not stop taking the drug abruptly to avoid rebound hypertension and will not stop the drug when blood pressure returns to normal.

The nurse teaches a patient about antihypertensive medication. Which statements by the patient indicate understanding of the teaching? (Select all that apply.) a. "I should be careful when I stand up from a chair." b. "I should not add extra salt to my foods." c. "If I have side effects, I should stop taking the drug immediately." d. "If my blood pressure returns to normal, I can stop taking this drug." e. "I may need to take a combination of drugs, including diuretics." f. "I will not need to make lifestyle changes since I am taking a medication."

ANS: C African Americans are more likely to be susceptible to low renin hypertension. Beta blockers reduce heart rate, contractility, and renin release, and there is a greater hypotensive response in patients with higher renin levels. Changes in heart rate, contractility, and vascular resistance explain why there is some response in this group.

The nurse understands that a medication such as carvedilol (Coreg) may not be effective in an African-American patient because of its effects on a. cardiac contractility. b. heart rate. c. renin release. d. vascular resistance.

Answer: C

The nurse would question an order for a calcium channel blocker in a patient with which condition? A Angina pectoris B Increased intracranial pressure C Hypotension D Dysrhythmia

Answer: D RATIONALE: Although pharmacokinetic changes in older adults affect all phases of kinetics, drug accumulation secondary to reduced renal excretion is the most important cause of ADRs in the older adult.

When assessing for drug effects in the older adult, which phase of pharmacokinetics is the greatest concern? A. Absorption B. Distribution C. Metabolism D. Excretion

Answer: C

When caring for a patient with angina, the nurse would question an order for a noncardioselective beta blocker in a patient with what coexisting medical diagnosis? A Hypertension B Atrial fibrillation C Bronchial asthma D Myocardial infarction

Answer: C

Where does disintegration of enteric-coated tablets occur? a. colon b. liver c. small intestine d. stomach

Answer: D RATIONALE: Nifedipine, amlodipine, and isradipine, which are dihydropyridine calcium channel blockers, cause less risk of constipation than diltiazem and verapamil.

Which agent is most likely to cause the side effect of constipation? A Nifedipine (Adalat) B Amlodipine (Norvasc) C Isradipine (DynaCirc) D Diltiazem (Cardizem)

Answer: C RATIONALE: Caution must be exercised in combining an alpha blocker (terazosin [Hytrin], doxazosin [Cardura], Tamsulosin [Flomax], alfuzosin [Uroxatral]) with an antihypertensive medication such as metoprolol, because severe hypotension could result. Blood glucose, hemoglobin, and urine output are not affected by combining an alpha blocker and a hypertensive medication.

Which assessment should a nurse monitor more frequently in a patient who takes both an alpha blocker for BPH and metoprolol (Lopressor)? A Blood glucose level B Hemoglobin level C Blood pressure D Urine output

Answer: C

Which drug for is most rapidly absorbed from the GI tract? a. capsule b. sublingual c. suspension d. tablet

ANS: C When drugs are administered parenterally, there is no pharmaceutic phase, which occurs when a drug becomes a solution that can cross the biologic membrane.

Which drugs will go through a pharmaceutic phase after it is administered? a. Intramuscular cephalosporins b. Intravenous vasopressors c. Oral analgesics d. Subcutaneous antiglycemics

Answer: D

Which medication would the nurse question if prescribed concurrently with ACE inhibitors? A furosemide (Lasix) B morphine C Colace D potassium

Answer: B

Which statement indicates that the patient understands discharge teaching about nitroglycerin? A "I will need to refill my prescription once a year in order to maintain potency." B "I will need to keep the nitroglycerin stored in the bottle it comes in." C "I will take a nitroglycerin tablet every 15 minutes until my chest pain is gone." D "I should take a nitroglycerin tablet 2 hours before I want to engage in activity that will cause chest pain."

Answer: C

the nurse is giving a large initial dose of a drug to rapidly achieve minimum effective concentration in the plasma. What is this type of dosage called? a. therapeutic dose b. toxic dose c. loading dose d. peak dose

Answer: C

The health care provider prescribes lansoprazole (Prevacid) to a client. Which assessment indicates to the nurse that the medication has had a therapeutic effect? a. The client has no diarrhea. b. The client has no gastric pain. c. The client has no throat pain. d. The client is able to eat.

Answer: 3 RATIONALE: Glucocorticoids can elevate blood glucose levels. Clients with diabetes mellitus may need their dosages of insulin or oral hypoglycemic medications increased during glucocorticoid therapy. Therefore the other options are incorrect.

39.) Prednisone is prescribed for a client with diabetes mellitus who is taking Humulin neutral protamine Hagedorn (NPH) insulin daily. Which of the following prescription changes does the nurse anticipate during therapy with the prednisone? 1. An additional dose of prednisone daily 2. A decreased amount of daily Humulin NPH insulin 3. An increased amount of daily Humulin NPH insulin 4. The addition of an oral hypoglycemic medication daily

Answer: 2 RATIONALE: A common side effect of the TB drug INH is peripheral neuritis. This is manifested by numbness, tingling, and paresthesias in the extremities. This side effect can be minimized by pyridoxine (vitamin B6) intake. Options 1, 3, and 4 are incorrect.

47.) A client has been taking isoniazid (INH) for 2 months. The client complains to a nurse about numbness, paresthesias, and tingling in the extremities. The nurse interprets that the client is experiencing: 1. Hypercalcemia 2. Peripheral neuritis 3. Small blood vessel spasm 4. Impaired peripheral circulation

Answer: D RATIONALE: Although all pieces of information are important, the nurse should ask the patient how many puffs of the inhaler were taken to determine that the patient did not overdose on the medication. Other side effects of albuterol, besides shaking and trembling, include sweating, nausea, headaches, blurred vision, and flushing.

(18/28) A patient calls the home health agency to tell the nurse she is having a reaction to her albuterol (Proventil) inhaler. She tells the nurse she is shaking and trembling. What is the first question the nurse should ask the patient? a. "Are you having any other symptoms?" b. "How long ago did this start?" c. "When was the last time you took your inhaler?" d. "How many puffs on the inhaler did you take?"

Answer: D

(19/25) A specific group of anticholinergics may be prescribed in the early treatment of which neuromuscular disorder? a. multiple sclerosis b. muscular dystrophy c. myasthenia gravis d. parkinsonism

Answer: B

(2/22) In the United States, how many categories/schedules are controlled substances. grouped into? a. 3 b. 5 c. 7 d. 9

Answer: C

(2/23) Drugs on which schedule have accepted medical use? a. I through IV b. I through V c. II through V d. III through V

Answer: C

(2/24) What is correct about schedule drugs' potential for abuse? a. V > IV b. III > I c. I > III d. V > III

Answer: D

(2/25) The patient presents to the emergency department with hallucinations. The patient's friends state he has been using lysergic acid diethylamide (LSD) and mescaline. To which schedule do these drugs belong? a. Schedule IV b. Schedule III c. Schedule II d. Schedule I

Answer: B

(22/28) Phenytoin levels must be monitored carefully as there is a narrow therapeutic range. Which result is within therapeutic range? a. 8mcg/mL b. 18mcg/mL c. 28mcg/mL d. 38mcg/mL

Answer: 1 RATIONALE: Donepezil hydrochloride is a cholinergic agent used in the treatment of mild to moderate dementia of the Alzheimer type. It enhances cholinergic functions by increasing the concentration of acetylcholine. It slows the progression of Alzheimer's disease. Options 2, 3, and 4 are incorrect.

105.) A nurse is collecting data from a client and the client's spouse reports that the client is taking donepezil hydrochloride (Aricept). Which disorder would the nurse suspect that this client may have based on the use of this medication? 1. Dementia 2. Schizophrenia 3. Seizure disorder 4. Obsessive-compulsive disorder

Answer: 1 RATIONALE: NPH is intermediate-acting insulin. Its onset of action is 1 to 2½ hours, it peaks in 4 to 12 hours, and its duration of action is 24 hours. Hypoglycemic reactions most likely occur during peak time.

115.) A client received 20 units of NPH insulin subcutaneously at 8:00 AM. The nurse should check the client for a potential hypoglycemic reaction at what time? 1. 5:00 PM 2. 10:00 AM 3. 11:00 AM 4. 11:00 PM

Answer: 3 Rationale: Zollinger-Ellison syndrome is a hypersecretory condition of the stomach. The client should avoid taking medications that are irritating to the stomach lining. Irritants would include aspirin and nonsteroidal antiinflammatory drugs (ibuprofen). The client should be advised to take acetaminophen for headache. *Remember that options that are comparable or alike are not likely to be correct. With this in mind, eliminate options 1 and 2 first.*

120.) A client is taking lansoprazole (Prevacid) for the chronic management of Zollinger-Ellison syndrome. The nurse advises the client to take which of the following products if needed for a headache? 1. Naprosyn (Aleve) 2. Ibuprofen (Advil) 3. Acetaminophen (Tylenol) 4. Acetylsalicylic acid (aspirin)

Answer: 2,3,4 RATIONALE: Furosemide is a loop diuretic; therefore, an expected effect is increased urinary frequency. Nausea is a frequent side effect, not an adverse effect. Photosensitivity is an occasional side effect. Adverse effects include tinnitus (ototoxicity), hypotension, and hypokalemia and occur as a result of sudden volume depletion.

126.) A nurse is caring for a client who has been prescribed furosemide (Lasix) and is monitoring for adverse effects associated with this medication. Which of the following should the nurse recognize as a potential adverse effect Select all that apply. 1. Nausea 2. Tinnitus 3. Hypotension 4. Hypokalemia 5. Photosensitivity 6. Increased urinary frequency

Answer: 1 RATIONALE: Lisinopril is an antihypertensive angiotensin-converting enzyme (ACE) inhibitor. The usual dosage range is 20 to 40 mg per day. Adverse effects include headache, dizziness, fatigue, orthostatic hypotension, tachycardia, and angioedema. Specific client teaching points include taking one pill a day, not stopping the medication without consulting the health care provider (HCP), and monitoring for side effects and adverse reactions. The client should notify the HCP if side effects occur.

127.) The nurse provides medication instructions to an older hypertensive client who is taking 20 mg of lisinopril (Prinivil, Zestril) orally daily. The nurse evaluates the need for further teaching when the client states which of the following? 1. "I can skip a dose once a week." 2. "I need to change my position slowly." 3. "I take the pill after breakfast each day." 4. "If I get a bad headache, I should call my doctor immediately."

Answer: A Rationale: Tachycardia is a response to activation of beta1 receptors. It can occur when large doses of albuterol are used and selectivity decreases. The nurse should question the patient about the number of inhalations used. Isoproterenol can cause hyperglycemia in diabetic patients. Isoproterenol has more side effects than albuterol. Tremors are an expected side effect and are not an indication for stopping the drug.

136. A patient with asthma uses albuterol (Ventolin) for wheezing. The nurse assesses the patient and notes vital signs of HR, 96 beats per minute; RR, 18 breaths per minute; and BP, 116/78 mm Hg. The patient has clear breath sounds and hand tremors. What will the nurse do? a. Ask the patient how often the inhaler is used. b. Check the patient's blood glucose level. c. Request an order for isoproterenol (Isuprel) to reduce side effects. d. Stop the medication and report the tremors to the provider.

Answer: 3 RATIONALE: Corticosteroids (glucocorticoids) should be administered before 9:00 AM. Administration at this time helps minimize adrenal insufficiency and mimics the burst of glucocorticoids released naturally by the adrenal glands each morning. *Note the suffix "-sone," and recall that medication names that end with these letters are corticosteroids.*

138.) A daily dose of prednisone is prescribed for a client. A nurse reinforces instructions to the client regarding administration of the medication and instructs the client that the best time to take this medication is: 1. At noon 2. At bedtime 3. Early morning 4. Anytime, at the same time, each day

Answer: 4 RATIONALE: Metoclopramide is a gastrointestinal (GI) stimulant and antiemetic. Because it is a GI stimulant, it is contraindicated with GI obstruction, hemorrhage, or perforation. It is used in the treatment of emesis after surgery, chemotherapy, and radiation.

140.) The client has a new prescription for metoclopramide (Reglan). On review of the chart, the nurse identifies that this medication can be safely administered with which condition? 1. Intestinal obstruction 2. Peptic ulcer with melena 3. Diverticulitis with perforation 4. Vomiting following cancer chemotherapy

Answer: 3 RATIONALE: Because antineoplastic medications lower the resistance of the body, clients must be informed not to receive immunizations without a HCP's approval. Clients also need to avoid contact with individuals who have recently received a live virus vaccine. Clients need to avoid aspirin and aspirin-containing products to minimize the risk of bleeding, and they need to avoid alcohol to minimize the risk of toxicity and side effects.

16.) The clinic nurse is reviewing a teaching plan for the client receiving an antineoplastic medication. When implementing the plan, the nurse tells the client: 1. To take aspirin (acetylsalicylic acid) as needed for headache 2. Drink beverages containing alcohol in moderate amounts each evening 3. Consult with health care providers (HCPs) before receiving immunizations 4. That it is not necessary to consult HCPs before receiving a flu vaccine at the local health fair

Answer: 2 RATIONALE: Insulin glargine is a long-acting recombinant DNA human insulin used to treat type 1 and type 2 diabetes mellitus. It has a 24-hour duration of action and is administered once a day, at the same time each day.

169.) Insulin glargine (Lantus) is prescribed for a client with diabetes mellitus. The nurse tells the client that it is best to take the insulin: 1. 1 hour after each meal 2. Once daily, at the same time each day 3. 15 minutes before breakfast, lunch, and dinner 4. Before each meal, on the basis of the blood glucose level

Answer: 2 RATIONALE: Warfarin sodium is an anticoagulant. Anticoagulant medications act by antagonizing the action of vitamin K, which is needed for clotting. When a client is taking an anticoagulant, foods high in vitamin K often are omitted from the diet. Vitamin K-rich foods include green, leafy vegetables, fish, liver, coffee, and tea.

172.) A nurse provides dietary instructions to a client who will be taking warfarin sodium (Coumadin). The nurse tells the client to avoid which food item? 1. Grapes 2. Spinach 3. Watermelon 4. Cottage cheese

Answer: 2 RATIONALE: Lansoprazole is a gastric acid pump inhibitor used to treat gastric and duodenal ulcers, erosive esophagitis, and hypersecretory conditions. It also is used to treat gastroesophageal reflux disease (GERD). It is not used to treat visual problems, problems with appetite, or leg pain. *NOTE: "-zole" refers to gastric acid pump inhibitors

176.) A nurse notes that a client is taking lansoprazole (Prevacid). On data collection, the nurse asks which question to determine medication effectiveness? 1. "Has your appetite increased?" 2. "Are you experiencing any heartburn?" 3. "Do you have any problems with vision?" 4. "Do you experience any leg pain when walking?"

Answer: 4 RATIONALE: An important component of taking this medication is monitoring the pulse rate; however, it is not necessary for the client to take both the radial and carotid pulses. It is not necessary for the client to check the blood pressure every morning and evening because the medication does not directly affect blood pressure. It is most important for the client to know the guidelines related to withholding the medication and calling the HCP. The client should not stop taking a medication.

180.) A health care provider (HCP) writes a prescription for digoxin (Lanoxin), 0.25 mg daily. The nurse teaches the client about the medication and tells the client that it is important to: 1. Count the radial and carotid pulses every morning. 2. Check the blood pressure every morning and evening. 3. Stop taking the medication if the pulse is higher than 100 beats per minute. 4. Withhold the medication and call the HCP if the pulse is less than 60 beats per minute.

Answer: 3 RATIONALE: Nitroglycerin acts directly on the smooth muscle of the blood vessels, causing relaxation and dilation. As a result, hypotension can occur. The nurse would check the client's blood pressure before administering the second nitroglycerin tablet. Although the respirations and apical pulse may be checked, these vital signs are not affected as a result of this medication. The temperature also is not associated with the administration of this medication.

182.) A client with angina pectoris is experiencing chest pain that radiates down the left arm. The nurse administers a sublingual nitroglycerin tablet to the client. The client's pain is unrelieved, and the nurse determines that the client needs another nitroglycerin tablet. Which of the following vital signs is most important for the nurse to check before administering the medication? 1. Temperature 2. Respirations 3. Blood pressure 4. Radial pulse rate

Answer: 3 RATIONALE: In treating an acute asthma attack, a short acting β2 agonist such as albuterol (Proventil HFA) will be given to produce bronchodilation. Options 1, 2, and 4 are long-term control (preventive) medications.

191.) A child is brought to the emergency department for treatment of an acute asthma attack. The nurse prepares to administer which of the following medications first? 1. Oral corticosteroids 2. A leukotriene modifier 3. A β2 agonist 4. A nonsteroidal anti-inflammatory

Answer: 4 RATIONALE: Because acetylcysteine has a pervasive odor of rotten eggs, it must be disguised in a flavored ice drink. It is consumed preferably through a straw to minimize contact with the mouth. It is not administered by the intramuscular or subcutaneous route. *Knowing that the medication is a solution that is also used for nebulization treatments will assist you to select the option that indicates an oral route*

205.) A nurse is assisting in preparing to administer acetylcysteine (Mucomyst) to a client with an overdose of acetaminophen (Tylenol). The nurse prepares to administer the medication by: 1. Administering the medication subcutaneously in the deltoid muscle 2. Administering the medication by the intramuscular route in the gluteal muscle 3. Administering the medication by the intramuscular route, mixed in 10 mL of normal saline 4. Mixing the medication in a flavored ice drink and allowing the client to drink the medication through a straw

Answer: 3 RATIONALE: The therapeutic phenytoin level is 10 to 20 mcg/mL. At a level higher than 20 mcg/mL, involuntary movements of the eyeballs (nystagmus) appear. At a level higher than 30 mcg/mL, ataxia and slurred speech occur.

211.) A client with epilepsy is taking the prescribed dose of phenytoin (Dilantin) to control seizures. A phenytoin blood level is drawn, and the results reveal a level of 35 mcg/ml. Which of the following symptoms would be expected as a result of this laboratory result? 1. Nystagmus 2. Tachycardia 3. Slurred speech 4. No symptoms, because this is a normal therapeutic level

Answer: 1 RATIONALE: Mild intoxication with acetylsalicylic acid (aspirin) is called salicylism and is commonly experienced when the daily dosage is higher than 4 g. Tinnitus (ringing in the ears) is the most frequently occurring effect noted with intoxication. Hyperventilation may occur because salicylate stimulates the respiratory center. Fever may result because salicylate interferes with the metabolic pathways involved with oxygen consumption and heat production. Options 2, 3, and 4 are incorrect.

213.) A client is admitted to the hospital with complaints of back spasms. The client states, "I have been taking two or three aspirin every 4 hours for the past week and it hasn't helped my back." Aspirin intoxication is suspected. Which of the following complaints would indicate aspirin intoxication? 1. Tinnitus 2. Constipation 3. Photosensitivity 4. Abdominal cramps

Answer: 3 RATIONALE: Sildenafil (Viagra) enhances the vasodilating effect of nitric oxide in the corpus cavernosum of the penis, thus sustaining an erection. Because of the effect of the medication, it is contraindicated with concurrent use of organic nitrates and nitroglycerin. Sildenafil is not contraindicated with the use of vitamins. Neuralgia and insomnia are side effects of the medication.

27.) Sildenafil (Viagra) is prescribed to treat a client with erectile dysfunction. A nurse reviews the client's medical record and would question the prescription if which of the following is noted in the client's history? 1. Neuralgia 2. Insomnia 3. Use of nitroglycerin 4. Use of multivitamins

Answer: 2 RATIONALE: Humulin NPH is an intermediate-acting insulin. The onset of action is 1.5 hours, it peaks in 4 to 12 hours, and its duration of action is 24 hours. Hypoglycemic reactions most likely occur during peak time.

29.) A client is taking Humulin NPH insulin daily every morning. The nurse reinforces instructions for the client and tells the client that the most likely time for a hypoglycemic reaction to occur is: 1. 2 to 4 hours after administration 2. 4 to 12 hours after administration 3. 16 to 18 hours after administration 4. 18 to 24 hours after administration

Answer: A

5. Which of the following drugs on overdose is known to cause liver failure? A. Acetaminophen B. Ibuprofen C. Piroxicam D. Ketorelac

Answer: 4 RATIONALE: Ecotrin is an aspirin-containing product and should be avoided. Alcohol consumption should be avoided by a client taking warfarin sodium. Taking prescribed medication at the same time each day increases client compliance. The Medic-Alert bracelet provides health care personnel emergency information.

54.) A nurse reinforces discharge instructions to a postoperative client who is taking warfarin sodium (Coumadin). Which statement, if made by the client, reflects the need for further teaching? 1. "I will take my pills every day at the same time." 2. "I will be certain to avoid alcohol consumption." 3. "I have already called my family to pick up a Medic-Alert bracelet." 4. "I will take Ecotrin (enteric-coated aspirin) for my headaches because it is coated."

Answer: 4 RATIONALE: The PT will assess for the therapeutic effect of warfarin sodium (Coumadin) and the aPTT will assess the therapeutic effect of heparin sodium. Heparin sodium doses are determined based on these laboratory results. The hemoglobin and hematocrit values assess red blood cell concentrations.

56.) Heparin sodium is prescribed for the client. The nurse expects that the health care provider will prescribe which of the following to monitor for a therapeutic effect of the medication? 1. Hematocrit level 2. Hemoglobin level 3. Prothrombin time (PT) 4. Activated partial thromboplastin time (aPTT)

Answer: 3 RATIONALE: Tissue plasminogen activator is a thrombolytic. Hemorrhage is a complication of any type of thrombolytic medication. The client is monitored for bleeding. Monitoring for renal failure and monitoring the client's psychosocial status are important but are not the most critical interventions. Heparin is given after thrombolytic therapy, but the question is not asking about follow-up medications.

59.) A client is diagnosed with an acute myocardial infarction and is receiving tissue plasminogen activator, alteplase (Activase, tPA). Which action is a priority nursing intervention? 1. Monitor for renal failure. 2. Monitor psychosocial status. 3. Monitor for signs of bleeding. 4. Have heparin sodium available.

Answer: 3 RATIONALE: Thiazide diuretics such as hydrochlorothiazide are sulfa-based medications, and a client with a sulfa allergy is at risk for an allergic reaction. Also, clients are at risk for hypokalemia, hyperglycemia, hypercalcemia, hyperlipidemia, and hyperuricemia.

60.) A nurse is planning to administer hydrochlorothiazide (HydroDIURIL) to a client. The nurse understands that which of the following are concerns related to the administration of this medication? 1. Hypouricemia, hyperkalemia 2. Increased risk of osteoporosis 3. Hypokalemia, hyperglycemia, sulfa allergy 4. Hyperkalemia, hypoglycemia, penicillin allergy

Answer: 4 RATIONALE: Clients taking trimethoprim-sulfamethoxazole (TMP-SMZ) should be informed about early signs of blood disorders that can occur from this medication. These include sore throat, fever, and pallor, and the client should be instructed to notify the health care provider if these symptoms occur. The other options do not require health care provider notification.

66.) Trimethoprim-sulfamethoxazole (TMP-SMZ) is prescribed for a client. A nurse should instruct the client to report which symptom if it developed during the course of this medication therapy? 1. Nausea 2. Diarrhea 3. Headache 4. Sore throat

Answer: 4 RATIONALE: Dyskinesia and impaired voluntary movement may occur with high levodopa dosages. Nausea, anorexia, dizziness, orthostatic hypotension, bradycardia, and akinesia (the temporary muscle weakness that lasts 1 minute to 1 hour, also known as the "on-off phenomenon") are frequent side effects of the medication.

76.) Carbidopa-levodopa (Sinemet) is prescribed for a client with Parkinson's disease, and the nurse monitors the client for adverse reactions to the medication. Which of the following indicates that the client is experiencing an adverse reaction? 1. Pruritus 2. Tachycardia 3. Hypertension 4. Impaired voluntary movements

Answer: C RATIONALE: The primary and most serious adverse effect of heparin is bleeding. Bleeding can occur from any site and may be manifested in various ways, including reduced blood pressure, increased heart rate, bruises, petechiae, hematomas, red or black stools, cloudy or discolored urine, pelvic pain, headache, and lumbar pain.

A 63-year-old male patient has been receiving a heparin infusion for 2 days for treatment of pulmonary embolism. Which symptom most clearly relates to an adverse effect of heparin? A Heart rate of 60 beats per minute B Blood pressure of 160/88 mm Hg C Discolored urine D Wheezing

Answer: 1 RATIONALE: Phenytoin (Dilantin) is an anticonvulsant. Gingival hyperplasia, bleeding, swelling, and tenderness of the gums can occur with the use of this medication. The client needs to be taught good oral hygiene, gum massage, and the need for regular dentist visits. The client should not skip medication doses, because this could precipitate a seizure. Capsules should not be chewed or broken and they must be swallowed. The client needs to be instructed to report a sore throat, fever, glandular swelling, or any skin reaction, because this indicates hematological toxicity.

77.) Phenytoin (Dilantin), 100 mg orally three times daily, has been prescribed for a client for seizure control. The nurse reinforces instructions regarding the medication to the client. Which statement by the client indicates an understanding of the instructions? 1. "I will use a soft toothbrush to brush my teeth." 2. "It's all right to break the capsules to make it easier for me to swallow them." 3. "If I forget to take my medication, I can wait until the next dose and eliminate that dose." 4. "If my throat becomes sore, it's a normal effect of the medication and it's nothing to be concerned about."

Answer: 2 RATIONALE: The therapeutic serum drug level range for phenytoin (Dilantin) is 10 to 20 mcg/mL. * A helpful hint may be to remember that the theophylline therapeutic range and the acetaminophen (Tylenol) therapeutic range are the same as the phenytoin (Dilantin) therapeutic range.*

78.) A client is taking phenytoin (Dilantin) for seizure control and a sample for a serum drug level is drawn. Which of the following indicates a therapeutic serum drug range? 1. 5 to 10 mcg/mL 2. 10 to 20 mcg/mL 3. 20 to 30 mcg/mL 4. 30 to 40 mcg/mL

Answer: 3 RATIONALE: Phenytoin (Dilantin) enhances the rate of estrogen metabolism, which can decrease the effectiveness of some birth control pills. Options 1, 2, are 4 are not accurate.

80.) A nurse is caring for a client who is taking phenytoin (Dilantin) for control of seizures. During data collection, the nurse notes that the client is taking birth control pills. Which of the following information should the nurse provide to the client? 1. Pregnancy should be avoided while taking phenytoin (Dilantin). 2. The client may stop taking the phenytoin (Dilantin) if it is causing severe gastrointestinal effects. 3. The potential for decreased effectiveness of the birth control pills exists while taking phenytoin (Dilantin). 4. The increased risk of thrombophlebitis exists while taking phenytoin (Dilantin) and birth control pills together.

Answer: 4. Complete blood count RATIONALE: A common side effect of therapy with zidovudine is leukopenia and anemia. The nurse monitors the complete blood count results for these changes. Options 1, 2, and 3 are unrelated to the use of this medication.

94.) The client with acquired immunodeficiency syndrome has begun therapy with zidovudine (Retrovir, Azidothymidine, AZT, ZDV). The nurse carefully monitors which of the following laboratory results during treatment with this medication? 1. Blood culture 2. Blood glucose level 3. Blood urea nitrogen 4. Complete blood count

ANS: B Female patients who take oral contraceptives and anticonvulsants should be advised to use a backup method of contraception because of reduced effectiveness of OCPs. Patients should be cautioned to consult with a provider if considering pregnancy because of the teratogenic effects of anticonvulsants. Patients should not stop taking OCPs and do not need to take precautions against stroke.

A 25 year-old female patient will begin taking phenytoin for epilepsy. The patient tells the nurse she is taking oral contraceptives (OCPs). Which response will the nurse give? a. "Continue taking OCPs because phenytoin is not safe during pregnancy." b. "You should use a backup method of contraception along with OCPs." c. "You should stop taking OCPs because of drug-drug interactions with phenytoin." d. "You should take low-dose aspirin while taking these medications to reduce your risk of stroke."

ANS: C Patients who require less than 40 units of insulin per day and who have a fasting blood glucose less than or equal to 200 mg/dL are candidates for oral antidiabetic agents. Being overweight is an indication, not a contraindication.

A 45-year-old patient who is overweight has had a diagnosis of type 2 diabetes for 2 years. The patient uses 20 units of insulin per day. The patient's fasting blood glucose (FBG) is 190 mg/dL. The patient asks the nurse about using an oral antidiabetic agent. The nurse understands that oral antidiabetic agents a. cannot be used if the patient is overweight. b. cannot be used once a patient requires insulin. c. may be used since this patient meets criteria. d. may not be used since this patient's fasting blood glucose is too high.

Answer: C RATIONALE: Unlike streptokinase, alteplase does not cause an allergic reaction or hypotension. The greatest risk with this drug is bleeding, with intracranial bleeding being the greatest concern. A decreasing level of consciousness indicates intracranial bleeding. Thrombolytic agents, such as alteplase, do not typically cause an elevated potassium level.

A 52-year-old male patient presents to the emergency department with symptoms of acute myocardial infarction. After a diagnostic workup, the healthcare provider prescribes a 15-mg IV bolus of alteplase (tPA), followed by 50 mg infused over 30 minutes. In monitoring this patient, the nurse understands that which symptom, if present, indicates the most likely adverse reaction to this drug? A Urticaria, itching, and flushing B Blood pressure of 90/50 mm Hg C Decreasing level of consciousness D Potassium level of 5.5 mEq/L

ANS: D Opioid analgesics can cause constipation. Stool softeners can help minimize this effect. Opioids do not cause dry mouth. Drug absorption may be decreased with an antacid. Opioid analgesics should be taken with food or milk to decrease gastrointestinal irritation.

A 75-year-old patient will be discharged home with a prescription for an opioid analgesic. To help the patient minimize adverse effects, what will the nurse recommend for this patient? a. Sucking on lozenges to moisten oral mucosa b. Taking an antacid with each dose c. Taking the medication on an empty stomach d. Using a stool softener

ANS: B Non European cultural groups such as those of Native American descent have less dependence on time schedules. Disease management will likely focus more on present concerns about alleviating current discomfort and less on measures to promote long-term wellness or treat a chronic illness.

A Native American patient has just been diagnosed with diabetes mellitus. The nurse preparing a teaching plan for this patient understands that which aspect of the disease and disease management may be most difficult for this patient? a. Body image changes b. Management of meal and medication schedules c. Perception of the disease as punishment from God d. The sense of dependence on others

Answer: A RATIONALE: Drugs classified in the category X place a client who is in the first trimester of pregnancy at risk for teratogenesis, so women in the childbearing years should be counseled to use a reliable form of birth control (A) during drug therapy. (B) is not a specific precaution with Category X drugs. The client should be encouraged to discuss plans for pregnancy with the healthcare provider, so a safer alternative prescription (C) can be provided if pregnancy occurs. Although the risk of birth defects during pregnancy explains the restriction of these drugs during pregnancy, (D) is not indicated.

A category X drug is prescribed for a young adult female client. Which instruction is most important for the nurse to teach this client? A) Use a reliable form of birth control. B) Avoid exposure to ultra violet light. C) Refuse this medication if planning pregnancy. D) Abstain from intercourse while on this drug.

ANS: A This child should receive (10 kg × 8 mg) 80 mg of TMP and (10 kg × 40 mg) 400 mL of SMX per day. When divided into two doses, the correct dose is 40 mg TMP and 200 mg SMX, or 5 mL per dose.

A child who weighs 10 kg will begin taking oral trimethoprim-sulfamethoxazole (TMP-SMX). The liquid preparation contains 40 mg of TMP and 200 mg of SMX per 5 mL. The nurse determines that the child's dose should be 8 mg of TMP and 40 mg of SMX/kg/day divided into two doses. Which order for this child is correct? a. 5 mL PO BID b. 5 mL PO daily c. 10 mL PO BID d. 10 mL PO daily

Answer: A RATIONALE: Nitroglycerin (A) is a nitrate that causes peripheral vasodilation and decreases contractility, thereby decreasing both preload and afterload. (B) is a beta adrenergic blocker that decreases both heart rate and contractility, but only decreases afterload. Morphine (C) decreases myocardial oxygen consumption and preload. Capoten (D) is an angiotensin converting enzyme (ACE) inhibitor that acts to prevents vasoconstriction, thereby decreasing blood pressure and afterload.

A client is admitted to the coronary care unit with a medical diagnosis of acute myocardial infarction. Which medication prescription decreases both preload and afterload? A) Nitroglycerin. B) Propranolol (Inderal). C) Morphine. D) Captopril (Capoten).

Answer: C RATIONALE: It is most important to monitor the blood pressure (C) of clients taking this medication because Lopressor is an antianginal, antiarrhythmic, antihypertensive agent. While (A and B) are important data to obtain on any client, they are not as important for a client receiving Lopressor as (C). Intake and output ratios and daily weights should be monitored while taking Lopressor to assess for signs and symptoms of congestive heart failure, but (D) alone does not have the importance of (C).

A client is receiving metoprolol (Lopressor SR). What assessment is most important for the nurse to obtain? A) Temperature. B) Lung sounds. C) Blood pressure. D) Urinary output.

Answer: D RATIONALE: Carbidopa significantly reduces the need for levodopa in clients with Parkinson's disease, so the new prescription should not be started until eight hours after the previous dose of levodopa (D), but can be started the same day (B). (A and C) may result in toxicity.

A client who has been taking levodopa PO TID to control the symptoms of Parkinson's disease has a new prescription for sustained release levodopa/carbidopa (Sinemet 25/100) PO BID. The client took his levodopa at 0800. Which instruction should the nurse include in the teaching plan for this client? A) Take the first dose of Sinemet today, as soon as your prescription is filled. B) Since you already took your levodopa, wait until tomorrow to take the Sinemet. C) Take both drugs for the first week, then switch to taking only the Sinemet. D) You can begin taking the Sinemet this evening, but do not take any more levodopa.

Answer: D RATIONALE: Carbidopa significantly reduces the need for levodopa in clients with Parkinson's disease, so the new prescription should not be started until eight hours after the previous dose of levodopa (D), but can be started the same day (B). (A and C) may result in toxicity.

A client who has been taking levodopa PO TID to control the symptoms of Parkinson's disease has a new prescription for sustained release levodopa/carbidopa (Sinemet 25/100) PO BID. The client took his levodopa at 0800. Which instruction should the nurse include in the teaching plan for this client? A) Take the first dose of Sinemet today, as soon as your prescription is filled. B) Since you already took your levodopa, wait until tomorrow to take the Sinemet. C) Take both drugs for the first week, then switch to taking only the Sinemet. D) You can begin taking the Sinemet this evening, but do not take any more levodopa.

Answer: C RATIONALE: Myopathy, suggested by the leg pain and weakness, is a serious, and potentially life-threatening, complication of Lipitor, and should be evaluated immediately by the healthcare provider (C). Although electrolyte imbalances such as (A or B) can cause muscle spasms in some cases, this is not the likely cause of leg pain in the client receiving Lipitor, and evaluation by the healthcare provider should not be delayed for any reason. A low-cholesterol diet is recommended for those taking Lipitor since the drug is used to lower total cholesterol (D), but diet is not related to the leg pain symptom.

A client who was prescribed atorvastatin (Lipitor) one month ago calls the triage nurse at the clinic complaining of muscle pain and weakness in his legs. Which statement reflects the correct drug-specific teaching the nurse should provide to this client? A) Increase consumption of potassium-rich foods since low potassium levels can cause muscle spasms. B) Have serum electrolytes checked at the next scheduled appointment to assess hyponatremia, a cause of cramping. C) Make an appointment to see the healthcare provider, because muscle pain may be an indication of a serious side effect. D) Be sure to consume a low-cholesterol diet while taking the drug to enhance the effectiveness of the drug.

Answer: B RATIONALE: Sinemet increases the amount of levodopa to the CNS (dopamine to the brain). Increased amounts of dopamine improve the symptoms of Parkinson's, such as involuntary movements, resting tremors (B), shuffling gait, etc. (A) is a side effect of Sinemet. Decreased drooling would be a desired effect, not (C). Sinemet does not affect (D).

A client with Parkinson's disease is taking carbidopa-levodopa (Sinemet). Which observation by the nurse should indicate that the desired outcome of the medication is being achieved? A) Decreased blood pressure. B) Lessening of tremors. C) Increased salivation. D) Increased attention span.

Answer: B RATIONALE: Angiotensin-converting enzyme (ACE) inhibitors are used in CHF to reduce afterload by reversing vasoconstriction common in heart failure. This vasodilation can cause hypotension and resultant dizziness (B). (A) is desired if fluid overload is present, and may occur as the result of effective combination drug therapy such as diuretics with ACE inhibitors. (C) often indicates hypokalemia in the client receiving diuretics. Excessive diuretic administration may result in fluid volume deficit, manifested by symptoms such as (D).

A client with congestive heart failure (CHF) is being discharged with a new prescription for the angiotensin-converting enzyme (ACE) inhibitor captopril (Capoten). The nurse's discharge instruction should include reporting which problem to the healthcare provider? A) Weight loss. B) Dizziness. C) Muscle cramps. D) Dry mucous membranes.

Answer: D Vomiting, anorexia and abdominal pain are early indications of digitalis toxicity. Since Lipitor increases the risk for digitalis toxicity, this finding requires the most immediate intervention by

A client with coronary artery disease who is taking digoxin (Lanoxin) receives a new prescription for atorvastatin (Lipitor). Two weeks after initiation of the Lipitor prescription, the nurse assesses the client. Which finding requires the most immediate intervention? A) Heartburn. B) Headache. C) Constipation. D) Vomiting.

Answer: D RATIONALE: Flagyl, an amoebicide and antibacterial agent, may cause gastric distress, so the client should be instructed to take the medication on a full stomach (D). Urine may be red-brown or dark from Flagyl, but this side effect is not necessary to report (A). Photosensitivity (B) is not a side effect associated with Flagyl. Despite the resolution of clinical symptoms, antiinfective medications should be taken for their entire course because stopping the medication (C) can increase the risk of resistant organisms.

A client with giardiasis is taking metronidazole (Flagyl) 2 grams PO. Which information should the nurse include in the client's instruction? A) Notify the clinic of any changes in the color of urine. B) Avoid overexposure to the sun. C) Stop the medication after the diarrhea resolves. D) Take the medication with food.

ANS: C Superinfection can occur with a secondary infection. Vaginal itching and discharge is a sign of superinfection. This is not symptomatic of pregnancy. These are not common side effects and do not indicate a hematologic reaction.

A female patient who is taking trimethoprim-sulfamethoxazole (TMP-SMZ) (Bactrim, Septra) to treat a urinary tract infection reports vaginal itching and discharge. The nurse will perform which action? a. Ask the patient if she might be pregnant. b. Reassure the patient that this is a normal side effect. c. Report a possible superinfection to the provider. d. Suspect that the patient is having a hematologic reaction.

ANS: B Phenytoin has serious teratogenic effects, so women who are pregnant should not take it. Phenobarbital is typically used because possible teratogenic effects are less pronounced. Teratogenicity increases with multiple anticonvulsants.

A female patient who takes phenytoin for epilepsy becomes pregnant. The nurse will notify the patient's provider and will anticipate that the provider will take which action? a. Add valproic acid (Depakote) for improved seizure control. b. Change the medication to phenobarbital (Luminal). c. Closely monitor this patient's serum phenytoin levels. d. Discontinue all anticonvulsant medications.

Answer: C RATIONALE: Gentamicin causes irreversible ototoxicity, which results in both impaired hearing and disruption of balance. Headache is the first sign of impending vestibular damage (balance) and may last 1 to 2 days. Unsteadiness, vertigo, and dizziness appear after headache.

A nurse monitors a patient who is receiving an aminoglycoside (gentamicin) for symptoms of vestibular damage. Which finding should the nurse expect the patient to have first? A Unsteadiness B Vertigo C Headache D Dizziness

ANS: D The desired action of oral contraceptives can be lessened when taken with tetracyclines, so patients taking oral contraceptives should be advised to use a backup contraception method while taking tetracyclines. Nausea and vomiting are common adverse effects. Doxycycline should not be taken with dairy products. Tetracycline may cause teratogenic effects.

A female patient will receive doxycycline to treat a sexually transmitted illness (STI). What information will the nurse include when teaching this patient about this medication? a. Nausea and vomiting are uncommon adverse effects. b. The drug may cause possible teratogenic effects. c. Increase intake of dairy products with each dose of this medication. d. Use a backup method of contraception if taking oral contraceptives.

Answer: C RATIONALE: Beta-1 agonists improve cardiac output by increasing the heart rate and blood pressure and are indicated in heart failure (C), shock, atrioventricular block dysrhythmias, and cardiac arrest. Glaucoma (A) is managed using adrenergic agents and beta-adrenergic blocking agents. Beta-1 blocking agents are used in the management of hypertension (B). Medications that stimulate beta-2 receptors in the bronchi are effective for bronchoconstriction in respiratory disorders, such as asthma (D).

A medication that is classified as a beta-1 agonist is most commonly prescribed for a client with which condition? A) Glaucoma. B) Hypertension. C) Heart failure. D) Asthma.

Answer: A RATIONALE: Metoclopramide works by blocking serotonin and dopamine receptors in the chemoreceptor trigger zone (CTZ) and by increasing motility in the small intestine. This action minimizes gastric distention and the accompanying stimulation of the vomiting center. Metoclopramide does not block H1 receptors or activate chloride channels in the intestine.

A nurse administers metoclopramide (Reglan) to a patient who is having nausea and vomiting postoperatively. The nurse should expect which therapeutic action if the medication is having the desired result? A Blocking of serotonin and dopamine receptors B Inactivation of histaminergic (H1) receptors C Activation of chloride channels in the intestine D Reduced motility in the small intestine

Answer: D RATIONALE: Tetracycline is contraindicated in children younger than 8 years of age, because it binds to calcium in developing teeth, resulting in permanent discoloration of the teeth. Delay in long bone growth, early onset of puberty, and severe face and body acne are not adverse effects associated with tetracyclines.

A nurse assessing a patient who is 12 years old should associate which complication with the patient's receiving tetracycline (Sumycin) as a younger child? A Delay in long bone growth B Early onset of puberty C Severe face and body acne D Discoloration of the teeth

Answer: D RATIONALE: The use of NSAIDs, which include ibuprofen (Advil), naproxen (Aleve), indomethacin (Indocin), and especially aspirin (Bayer), by children with influenza or chickenpox may precipitate Reye's syndrome. This is a potentially fatal multisystem organ disease. Acetaminophen (Tylenol) may be used safely to reduce fever in children with influenza.

A nurse instructs the parent of a child with influenza that which medication or medications may be used safely to reduce fever? (Select all that apply.) A Ibuprofen (Advil) B Naproxen (Aleve) C Aspirin (Bayer) D Acetaminophen (Tylenol) E Indomethacin (Indocin)

Answer: D RATIONALE: Amoxicillin disrupts the cell wall of H. pylori, which causes lysis and death. Inhibition of an enzyme to block acid secretion is a function of the proton pump inhibitors (PPIs). Coating of the ulcer crater as a barrier to acid is an action of sucralfate (Carafate). Selective blockade of parietal cell histamine2 receptors is an action of the histamine2 receptor antagonists cimetidine, ranitidine, famotidine, and nizatidine.

A nurse is planning care for a patient who has peptic ulcer disease and is taking amoxicillin (Amoxil). Which of these should the nurse establish as a therapeutic outcome? A Inhibition of an enzyme to block acid secretion B Coating of the ulcer crater as a barrier to acid C Selective blockade of parietal cell histamine2 receptors D Disruption of the bacterial cell wall, causing lysis and death

ANS: C Food can stimulate the production of gastric acid so medications requiring an acidic environment should be given with a meal. High fat foods are useful for drugs that are lipid soluble.

A nurse is preparing to administer an oral drug that is best absorbed in an acidic environment. How will the nurse give the drug? a. On an empty stomach b. With a full glass of water c. With food d. With high-fat food

Answer: D RATIONALE: Activation of beta2 receptors leads to bronchodilation; decreased uterine contractions; hyperglycemia; and vasodilation of the heart, lungs, and skeletal muscle. It does not produce increased peristalsis, constriction of the pupil, or hypoglycemia.

A nurse is preparing to give a medication that stimulates the beta2 receptors. What response will the nurse expect from this drug? A Increased peristalsis B Constriction of the pupil C Hypoglycemia D Bronchodilation

Answer: B RATIONALE: Montelukast is an antileukotriene agent. Combining montelukast with an inhaled glucocorticoid medication can improve asthma symptoms and thus may allow a reduction in the glucocorticoid dosage. The effects of montelukast develop slowly, so it cannot be used as a quick-relief agent. Short-acting beta2 agonists are preferred for exercise-induced asthma. Montelukast does not affect coagulation, so bleeding and bruising do not occur.

A nurse is teaching a patient about montelukast (Singulair). Which statement by the patient would indicate that the nurse's teaching was effective? A "I'll take a dose as soon as I feel short of breath and start to cough." B "While taking this medicine, I may be able to reduce my steroid medication." C "This is the priority medication for preventing exercise-induced asthma symptoms." D "If I have nosebleeds or excessive bruising, I'll stop the medication immediately."

Answer: A RATIONALE: Through several mechanisms, regular alcohol consumption while taking acetaminophen (Tylenol) increases the risk of liver injury when dosages are excessive. Therapeutic doses of acetaminophen (Tylenol) may be safe for patients who drink alcohol; however, the U.S. Food and Drug Administration (FDA) requires that acetaminophen (Tylenol) labels state an alcohol warning for patients who consume three or more drinks a day to consult their prescriber to determine whether acetaminophen (Tylenol) can be taken safely. It is not necessary to avoid leafy green foods, bananas, or dairy products when taking acetaminophen.

A nurse provides discharge instructions for a patient who is taking acetaminophen (Tylenol) after surgery. The nurse should instruct the patient to avoid which product while taking acetaminophen? A Alcoholic drinks B Leafy green foods C Bananas D Dairy products

Answer: C RATIONALE: High-dose aspirin therapy should be avoided in patients taking ACE inhibitors. In susceptible patients, these medications can impair renal function when they are combined with aspirin. Liver toxicity, congestive heart failure, and hemorrhage are not effects of ACE inhibitor and aspirin interactions.

A nurse should recognize that a patient who takes an angiotensin-converting enzyme (ACE) inhibitor while also taking high-dose aspirin is at risk of developing what complication? A Congestive heart failure B Liver toxicity C Renal failure D Hemorrhage

Answer: B RATIONALE: The CD4 T-cell count is the principal indicator of how much immunocompetence remains when a patient is infected with HIV. It is used as a guide in the initiation, discontinuation, and resumption of medications for opportunistic infections. A plasma HIV RNA assay is a measure of viral load that indicates HIV replication and magnitude and accordingly is used to predict clinical outcomes. The Western blot assay and OralQuick Rapid HIV-1 Antibody Test, respectively, are used for initial screening and follow-up confirmation of HIV infection.

A nurse should recognize that which laboratory result is used as a major factor in deciding when antiretroviral therapy is indicated for a patient infected with HIV? A Plasma HIV RNA assay B CD4 T-lymphocyte count C Western blot assay D OralQuick Rapid HIV-1 Antibody Test

ANS: B NSAIDs act by inhibiting COX 1 and COX-2 to help block prostaglandin synthesis. They do not have direct action on tissues, nor do they interfere with chemical receptor sites or neuronal pathways.

A nursing student asks how nonsteroidal antiinflammatory drugs (NSAIDs) work to suppress inflammation and reduce pain. The nurse will explain that NSAIDs a. exert direct actions to cause relaxation of smooth muscle. b. inhibit cyclooxygenase that is necessary for prostaglandin synthesis. c. interfere with neuronal pathways associated with prostaglandin action. d. suppress prostaglandin activity by blocking tissue receptor sites.

ANS: C Parkinson's disease (PD) is characterized by an imbalance of dopamine (DA) and acetylcholine (ACh) caused by an unexplained degeneration of the dopaminergic neurons allowing the excitatory response of acetylcholine to exceed the inhibitory response of dopamine. Alzheimer's disease (AD) may result from decreased ACh, degeneration of cholinergic neurons, and neuritic plaques. Dopamine does not appear to play a role in Alzheimer's disease.

A nursing student asks the nurse to differentiate the pathology of Alzheimer's disease from that of Parkinson's disease. Which description is correct? a. Alzheimer's disease involves a possible excess of acetylcholine and neuritic plaques. b. Alzheimer's disease is caused by decreased amounts of dopamine and degeneration of cholinergic neurons. c. Parkinson's disease is characterized by an imbalance of dopamine and acetylcholine. d. Parkinson's disease involves increased dopamine production and decreased acetylcholine.

ANS: D Without carbidopa, about 99% of levodopa is converted to dopamine before crossing the blood brain barrier, causing peripheral adverse effects. When carbidopa is added, the enzyme decarboxylase is inhibited, allowing levodopa to cross into the brain before being converted to the active metabolite dopamine. The result is less levodopa required to achieve the desired effect. The drug still has many side effects, but the peripheral effects are lessened.

A nursing student asks the nurse why patients who have parkinsonism receive a combination of carbidopa and levodopa. The nurse will explain that the combination product a. allows larger doses of levodopa to be given without causing increased adverse reactions. b. causes more levodopa to be converted to dopamine before crossing the blood-brain barrier. c. eliminates almost all drug side effects of both levodopa and carbidopa. d. reduces peripheral side effects by inhibiting decarboxylase in the peripheral nervous system.

Answer: A Rationale: Although albuterol affects the beta2 receptors, at higher doses it may also activate beta1 receptors, causing an increase in the heart rate. Bronchodilation does not lower the blood pressure, and activation of beta1 receptors actually may increase it. Tachycardia is a common side effect. The inhaled route reduces the likelihood of systemic side effects.

A nursing student asks why albuterol, which is selective for beta2 receptors, causes an increased heart rate in some patients. How does the nurse respond? a. "Adrenergic agonists can lose their selectivity when given at higher doses." b. "Bronchodilation lowers blood pressure, which causes a reflex tachycardia." c. "Some patients metabolize the drug differently and have unusual side effects." d. "Systemic effects are intensified with inhaled doses."

ANS: A The primary use of heparin for patients with DIC is to prevent venous thrombosis, which can lead to pulmonary embolism or stroke. Heparin does not break down blood clots, enhance the formation of fibrous clots, or preserve platelet function.

A nursing student asks why the anticoagulant heparin is given to patients who have disseminated intravascular coagulation (DIC) and are at risk for excessive bleeding. The nurse will explain that heparin is used in this case for which reason? a. To decrease the risk of venous thrombosis b. To dissolve blood clots as they form c. To enhance the formation of fibrous clots d. To preserve platelet function

ANS: C A rash can indicate a serious drug reaction. Patients should stop taking the drug immediately and notify the provider.

A patient who is taking trimethoprim-sulfamethoxazole (TMP-SMX) calls to report developing an all-over rash. The nurse will instruct the patient to perform which action? a. Increase fluid intake. b. Take diphenhydramine. c. Stop taking TMP-SMX immediately. d. Continue taking the medication.

ANS: D The patient probably has a food allergy, since eating strawberries is the precipitating event. The nurse should assess cardiac and respiratory status to determine whether the patient is developing an anaphylactic reaction. Diphenhydramine will be given for mild allergic symptoms of rash, runny nose, and sneezing, but epinephrine must be given for anaphylaxis. Urinary retention is a side effect of diphenhydramine and will be assessed if diphenhydramine is given.

A patient arrives in the emergency department after developing a rash, runny nose, and sneezing after eating strawberries. What action will the nurse expect to take first? a. Administer diphenhydramine. b. Administer epinephrine. c. Assess for urinary retention. d. Assess heart rate, respiratory rate, and lung sounds.

ANS: D OTC medications should always be included when listing medications because they can cause drug interactions. OTC medications can be as effective and as safe as prescription medications and have as many side effects and adverse reactions.

A patient asks the nurse about using over-the-counter (OTC) medications. The nurse will tell the patient that OTC medications a. are not as effective as prescription medications. b. are not as safe as prescription medications. c. have fewer side effects and drug interactions than prescription medications. d. should be included when listing any medications taken by the patient.

ANS: A Enteric coated tablets must be swallowed whole to maintain a therapeutic drug level since they are designed to be absorbed in the small intestine. If crushed, an initial excessive release of the drug may occur, causing toxicity. Enteric-coated tablets should not be dissolved in liquid.

A patient asks the nurse if an enteric-coated tablet can be crushed and put in pudding to make it easier to swallow. How will the nurse respond to the patient? a. "Crushing the medication can lead to a possibly toxic medication dose." b. "Crushing the medication is safe and can prevent gagging on pills." c. "The tablet may be done if a small amount of pudding is used." d. "The tablet may be dissolved in liquid but not crushed and put in food."

ANS: B Nitroglycerin is given sublingually to avoid first pass metabolism by the liver, which would occur if the drug is swallowed. It does not prevent hypotension. Gastrointestinal upset and hepatotoxicity usually do not occur.

A patient asks the nurse why nitroglycerin is given sublingually. The nurse will explain that nitroglycerin is administered by this route for which reason? a. To avoid hypotension b. To increase absorption c. To minimize gastrointestinal upset d. To prevent hepatotoxicity

ANS: B Transdermal patches provide more consistent blood levels. Cutting the patch is not recommended. Drugs given transdermally can still produce side effects and toxicity.

A patient asks the nurse why the provider has ordered a transdermal form of a medication. How will the nurse respond? a. "The patch can always be cut when dosage adjustments are needed." b. "Drug levels fluctuate less with the patch." c. "There are fewer systemic side effects with transdermal patches." d. "There is less risk of toxicity when using a patch."

Answer: A RATIONALE: When vancomycin is infused too rapidly, histamine release may cause the patient to develop hypotension accompanied by flushing and warmth of the neck and face; this phenomenon is called red man syndrome. Diphenhydramine is not necessary if the infusion is administered slowly over at least 60 minutes. Changing the IV tubing would not help the symptoms. The patency of the IV needs to be checked before the administration is started.

A patient develops flushing, rash, and pruritus during an IV infusion of vancomycin (Vancocin). Which action should a nurse take? A Reduce the infusion rate. B Administer diphenhydramine (Benadryl). C Change the IV tubing. D Check the patency of the IV.

Answer: B RATIONALE: The patient is showing signs of anaphylaxis caused by a peanut allergy. Histamine1 activation plays a minor role in anaphylaxis; other substances are the principal mediators. Therefore, the drug of choice for anaphylaxis is epinephrine. The antihistamines promethazine, diphenhydramine, and hydroxyzine are effective only for symptoms of mild allergy; they may be used as adjuncts in the treatment of anaphylaxis, but they will have only limited benefit.

A patient develops hypotension, laryngeal edema, and bronchospasm after eating peanuts. Which medication should the nurse prepare to administer? A Promethazine (Phenergan) B Epinephrine (Adrenalin) C Diphenhydramine (Benadryl) D Hydroxyzine (Vistaril)

ANS: A Thrombolytic medications are given primarily to prevent permanent tissue damage caused by compromised blood flow to the affected area. Thrombolytics do not prevent clots from forming. Aspirin prevents, but does not dissolve, clots. Although the body will break down the clot, the drug is needed to prevent tissue damage due to active ischemia.

A patient experiences a blood clot in one leg, and the provider has ordered a thrombolytic medication. The patient learns that the medication is expensive and asks the nurse if it is necessary. Which response by the nurse is correct? a. "The drug will decrease the likelihood of permanent tissue damage." b. "This medication also acts to prevent future blood clots from forming." c. "You could take aspirin instead of this drug to achieve the same effect." d. "Your body will break down the clot, so the drug is not necessary."

ANS: D Prehypertension is defined as a systolic pressure of 120 to 139 and a diastolic pressure between 80 and 89. Drug therapy is recommended if the blood pressure is greater than 20/10 over the goal, which would be140/90. Prehypertension is generally treated first with lifestyle changes.

A patient has a blood pressure of 135/85 mm Hg on three separate occasions. The nurse understands that this patient should be treated with a. a beta blocker. b. a diuretic and a beta blocker. c. a diuretic. d. lifestyle changes.

ANS: C Cardiovascular disease (CVD) risk doubles with each increase of 20/10 mm Hg above normal, starting at 115/75 mm Hg. This patient's blood pressure is 40/20 above normal, which increases the risk four times. A blood pressure of 135/85 would be two times greater. The patient's risk would still be four times greater with a blood pressure of 155/70 or 130/95, since systolic and diastolic blood hypertension are each powerful predictors of CVD.

A patient has a blood pressure of 155/95 mm Hg. The nurse understands that this patient's risk of cardiovascular disease is _____ greater than normal. a. two times b. three times c. four times d. six times

ANS: A A value of 270 mg/dL for serum cholesterol puts the patient at high risk.

A patient has a serum cholesterol level of 270 mg/dL. The patient asks the nurse what this level means. Which response by the nurse is correct? a. "You have a high risk for coronary artery disease." b. "You have a moderate risk for coronary artery disease." c. "You have a low risk for coronary artery disease." d. "You have no risk for coronary artery disease."

ANS: B The patient is symptomatic and has hypoglycemia. The nurse should give orange juice. Glucagon is given for patients unable to ingest carbohydrates. The kitchen should be notified, and bedside glucose testing should be performed, but only after the patient is given carbohydrates.

A patient has administered regular insulin 30 minutes prior but has not received a breakfast tray. The patient is experiencing nervousness and tremors. What is the nurse's first action? a. Administer glucagon. b. Give the patient orange juice. c. Notify the kitchen to deliver the tray. d. Perform bedside glucose testing.

ANS: B Protamine sulfate is given as an antidote to heparin when patient's clotting times are elevated. Oral warfarin will not stop the anticoagulant effects of heparin. Vitamin K is used as an antidote for warfarin. Administering heparin by another route is not indicated when there is a need to reverse the effects of heparin.

A patient has been receiving intravenous heparin. When laboratory tests are drawn, the nurse has difficulty stopping bleeding at the puncture site. The patient has bloody stools and is reporting abdominal pain.The nurse notes elevated partial thromboplastin time (PTT) and activated partial thromboplastin time (aPTT). Which action will the nurse perform? a. Ask for an order for oral warfarin (Coumadin). b. Obtain an order for protamine sulfate. c. Request an order for vitamin K. d. Suggest that the patient receive subcutaneous heparin.

ANS: C Adding another highly protein bound drug will displace the first drug from protein-binding sites and release more free drug increasing the drug's effects. This does not alter the therapeutic range, which is the serum level between drug effectiveness and toxicity.

A patient has been taking a drug that has a protein-binding effect of 75%. The provider adds a new medication that has a protein-binding effect of 90%. The nurse will expect a. decreased drug effects of the first drug. b. decreased therapeutic range of the first drug. c. increased drug effects of the first drug. d. increased therapeutic range of the first drug.

ANS: B Patients taking statins should report immediately any muscle aches or weakness, which can lead to rhabdomyolysis, a muscle disintegration that can become fatal. All statins carry this risk, so changing to another statin is not indicated. Ibuprofen may be useful, but notifying the provider is essential. Patients should not abruptly discontinue statins without discussing this with the provider.

A patient has been taking atorvastatin (Lipitor) for several months to treat hyperlipidemia. The patient reports muscle weakness and tenderness. The nurse will counsel the patient to a. ask the provider about switching to simvastatin. b. contact the provider to report these symptoms. c. start taking ibuprofen to combat these effects. d. stop taking the medication immediately.

ANS: D Famotidine is a histamine2 (H2) blocker. When patients fail therapy with these agents, proton pump inhibitors, which can inhibit gastric acid secretion up to 90% greater than the H2 blockers, are used. Lansoprazole is not for long term treatment and has drug interactions and drug side effects as do all other medications.

A patient has been taking famotidine (Pepcid) 20 mg bid to treat an ulcer but continues to have pain. The provider has ordered lansoprazole (Prevacid) 15 mg per day. The patient asks why the new drug is necessary, since it is more expensive. The nurse will explain that lansoprazole a. can be used for long-term therapy. b. does not interact with other drugs. c. has fewer medication side effects. d. is more potent than famotidine.

ANS: B The addition of a highly protein bound drug will compete with warfarin for protein binding sites, releasing more free warfarin into the system, increasing drug effects and increasing the chance of toxicity.

A patient has been taking warfarin (Coumadin), which is highly protein-bound. The patient will begin taking gemfibrozil, which is also highly protein-bound. The nurse will observe the patient closely for a. decreased effects of warfarin. b. increased effects of warfarin. c. decreased effects of gemfibrozil. d. decreased effects of both drugs.

ANS: B, D, E Vomiting, yellow haloes in the visual field, and diarrhea are classic signs of digoxin toxicity. Bradycardia, not tachycardia, will likely be noted.

A patient has congestive heart failure and has been taking digoxin (Lanoxin) for 9 years. The patient is admitted with signs and symptoms of digoxin toxicity. Which signs and symptoms are associated with digoxin toxicity? (Select all that apply.) a. Dysuria b. Vomiting c. Tachycardia d. Yellow haloes in the visual field e. Diarrhea f. Insomnia

ANS: B Reddish brown urine is a harmless side effect of phenytoin. The nurse should reassure the patient. It is not necessary to order a serum drug level or renal function studies. It is not a symptom of thrombocytopenia.

A patient has recently begun taking phenytoin (Dilantin) for a seizure disorder. The nurse notes a reddish-brown color to the patient's urine. Which action will the nurse take? a. Ask the provider to order a serum drug level. b. Reassure the patient that this is a harmless side effect. c. Report possible thrombocytopenia to the provider. d. Request an order for a urinalysis and creatinine clearance.

ANS: A Broad spectrum antibiotics are frequently used to treat infections when the offending organism has not been identified by culture and sensitivity (C&S). Narrow-spectrum antibiotics are usually effective against one type of organism and are used when the C&S indicates sensitivity to that antibiotic. The use of multiple antibiotics, unless indicated by C&S, can increase resistance. The pneumococcal vaccine is used to prevent, not treat, an infection.

A patient is admitted to the hospital for treatment of pneumonia after complaining of high fever and shortness of breath. The patient was not able to produce sputum for a culture. The nurse will expect the patient's provider to order a. a broad-spectrum antibiotic. b. a narrow-spectrum antibiotic. c. multiple antibiotics. d. the pneumococcal vaccine.

ANS: A Acetylcysteine is used as an antidote for acetaminophen overdose if given within 12 to 24 hours of ingestion. Dornase alfa is used to treat cystic fibrosis. Gastric lavage is no longer used as treatment. Liver enzyme tests are indicated since acetaminophen is hepatotoxic.

A patient is brought to the emergency department and reports having taken "a lot" of acetaminophen extra-strength tablets 16 hours prior. The nurse will expect the provider to order a. acetylcysteine (Mucomyst). b. dornase alfa (Pulmozyme). c. gastric lavage. d. renal enzyme tests.

ANS: D Weight loss decreases the stress on the heart and the afterload. Decreasing salt intake decreases the amount of retained fluid. Changing to yoga from weight bearing exercise, limiting fluids, and increasing potassium are not indicated. Stress reduction is recommended, but increasing protein is not.

A patient is diagnosed with borderline hypertension and states a desire to make lifestyle changes to avoid needing to take medication. The nurse will recommend which changes? a. Changing from weight bearing exercise to yoga b. Decreased fluid intake and increased potassium intake c. Stress reduction and increased protein intake d. Weight reduction and decreased sodium intake

ANS: B Of all seizure cases, 75% are primary, or idiopathic, with no known cause. The remaining are secondary and may be related to head trauma, stroke, or anoxic events.

A patient is diagnosed with epilepsy and asks the nurse what may have caused this condition. The nurse explains that epilepsy is most often a. caused by head trauma. b. idiopathic in origin. c. linked to a stroke. d. related to brain anoxia.

ANS: D Alcohol should either be completely avoided or restricted to no more than one per day. Mild exercise, such as walking, is recommended. All nicotine deprives the heart of oxygen. Salt should be limited to no more than one teaspoon per day.

A patient is diagnosed with heart failure, and the prescriber has ordered digoxin. The patient asks what lifestyle changes will help in the management of this condition. The nurse will recommend which changes? a. Aerobic exercise and weight lifting 2 or 3 times weekly b. Changing from cigarette smoking to pipe smoking c. Consuming 2 teaspoons or less of salt every day d. Having no more than one alcoholic beverage per day

ANS: D The pounding headache is related to vasodilation of the cerebral vessels. Nitrates decrease blood pressure.

A patient is ordered to receive a nitrate to relieve stable angina. What side effect(s) will the nurse anticipate in a patient receiving this medication? a. Nausea and vomiting b. Increased blood pressure c. Pruritus and skin rash d. Pounding headache

ANS: A Lispro acts faster than other insulins, and patients should be taught to give this medication not more than 5 minutes before eating.

A patient is ordered to receive insulin lispro at mealtimes. The nurse will instruct this patient to administer the medication at which time? a. 5 minutes before eating b. 15 minutes after eating c. 30 minutes before eating d. 10 minutes after eating

ANS: A Enteric coated and extended-release tablets must be swallowed whole to maintain a therapeutic drug level since they are designed to be absorbed in the small intestine. If crushed, an initial excessive release of the drug may occur, causing toxicity. Enteric-coated or extended-release tablets should not be dissolved in liquid. The nurse should contact the provider to discuss another form of the medication.

A patient is ordered to take an extended-release medication twice daily but has difficulty swallowing the tablet because of its size. The nurse will perform which action? a. Contact the provider to discuss an alternate form of the medication. b. Crush the tablet and put it in applesauce to help the patient swallow it. c. Cut the tablet in half so the patient can take it more easily. d. Dissolve the tablet in liquid.

ANS: D The generic name is the official, non proprietary name for a drug. The brand name is the trademark name and is always capitalized. The chemical name describes the chemical structure of the drug.

A patient is prescribed a medication and asks the nurse if the drug is available in a generic form. The nurse understands that a generic medication will have a name that a. is a registered trademark. b. is always capitalized. c. describes the drug's chemical structure. d. is non-proprietary.

Answer: D Rationale: Drug combination therapy is commonly used to manage chemotherapy-induced nausea and vomiting. Both lorazepam (Ativan) and the glucocorticoids have been found to be effective medications to assist in preventing and managing chemotherapy-induced nausea and vomiting.

A patient is prescribed lorazepam (Ativan) and a glucocorticoid during chemotherapy treatments. What is the nurse's best action? A Call the health care provider and question the order. B Only administer the Ativan if the patient seems anxious. C Administer the two medications at least 12 hours apart. D Administer the medications and assess the patient for relief.

ANS: A The antithrombolytic drug aminocaproic acid is used to treat hemorrhage. Nurses giving thrombolytic drugs should monitor patients for bleeding from the mouth and rectum. Enoxaparin is given for disseminated intravascular coagulation. Protamine sulfate is an antidote for heparin. Vitamin K is an antidote for warfarin.

A patient is receiving a thrombolytic medication. The patient calls the nurse to report having bloody diarrhea. The nurse will anticipate administering which medication? a. Aminocaproic acid (Amicar) b. Enoxaparin sodium (Lovenox) c. Protamine sulfate d. Vitamin K

Answer: D RATIONALE: Amphotericin B is toxic to cells of the kidneys. To evaluate renal injury, the plasma creatinine level should be monitored every 3 or 4 days, as well as intake and output. It is not necessary to monitor the serum protein or glucose levels or the serum pH in patients taking amphotericin B.

A patient is receiving amphotericin B. It is most important for the nurse to monitor which laboratory result? A Serum pH B Protein level C Glucose level D Creatinine level

Answer: D RATIONALE: Renal injury from amphotericin B may cause severe hypokalemia. Serum potassium levels should be monitored more frequently and potassium supplements given to correct low plasma levels. Furosemide, insulin, and vitamin K do not prevent any adverse effects of amphotericin B.

A patient is receiving amphotericin B. The nurse identifies which medication as useful in preventing adverse effects of amphotericin B? A Furosemide (Lasix) B Insulin C Vitamin K D Potassium

Answer: A RATIONALE: The patient's comment suggests that the person did not hear the instructions. Aminoglycoside antibiotics can cause ototoxicity. The first sign may be tinnitus (ringing in the ears), progressing to loss of high-frequency sounds. Audiometric testing is needed to detect it. Nutrition, confusion, and a family history of dementia do not address the problem of possible hearing loss associated with aminoglycosides.

A patient is receiving an aminoglycoside (tobramycin) antibiotic. A nurse asks the patient to choose daily meal selections, to which the patient responds, "Oh, dear, I don't want another IV." The nurse makes which assessment about the patient's response? A Some hearing loss may have occurred. B The confusion is due to the hospital stay. C A nutrition consult most likely is needed. D The patient has a family history of dementia.

Answer: B RATIONALE: The risk of gastrointestinal irritation and ulceration for a patient taking glucocorticoids is increased by concurrent use of other medications, such as aspirin and nonsteroidal anti-inflammatory drugs (NSAIDs).

A patient is receiving glucocorticoids for the treatment of rheumatoid arthritis. The patient complains of having a headache. Which ordered medication should the nurse administer? A Aspirin (Bayer) B Acetaminophen (Tylenol) C Ibuprofen (Advil) D Naproxen sodium (Aleve)

ANS: A Cefazolin will produce an increase in the patient's BUN, creatinine, AST, ALT, ALP, LDH, and bilirubin.

A patient is receiving high doses of a cephalosporin. Which laboratory values will this patient's nurse monitor closely? a. Blood urea nitrogen (BUN), serum creatinine, and liver function tests b. Complete blood count and electrolytes c. Serum calcium and magnesium d. Serum glucose and lipids

Answer: A RATIONALE: Penicillin G in high IV doses may cause hyperkalemia, which can result in dysrhythmias or cardiac arrest. Hypernatremia occurs with high IV doses of ticarcillin. Lung sounds and the RBC count are unrelated to the administration of penicillin G.

A patient is receiving penicillin G (Bicillin C-R). Which assessment should the nurse monitor as an indicator of an undesired effect? A Cardiac rhythm B Serum sodium level C Lung sounds D Red blood cell (RBC) count

Answer: C RATIONALE: The most common toxic effect of vancomycin (Vancocin) therapy is renal toxicity. Although ototoxicity may occur, it is rare. The liver and heart are not affected when vancomycin is used.

A patient is receiving vancomycin (Vancocin). The nurse identifies what as the most common toxic effect of vancomycin therapy? A Ototoxicity B Hepatotoxicity C Renal toxicity D Cardiac toxicity

Answer: A RATIONALE: Insulin glargine is administered as a once-daily subcutaneous injection for patients with type 1 diabetes. It is used for basal insulin coverage, not mealtime coverage. It has a prolonged duration, up to 24 hours, with no peaks. Blood glucose monitoring is still an essential component to achieve tight glycemic control.

A patient is scheduled to start taking insulin glargine (Lantus). On the care plan, a nurse should include which of these outcomes related to the therapeutic effects of the medication? A Blood glucose control for 24 hours B Mealtime coverage of blood glucose C Less frequent blood glucose monitoring D Peak effect achieved in 2 to 4 hours

ANS: C Aspirin is used to inhibit platelet aggregation to prevent cardiovascular accident and myocardial infarction. Patients taking aspirin for this purpose would not benefit from COX 2 inhibitors, since the COX-1 enzyme is responsible for inhibiting platelet aggregation. The patient should take enteric-coated aspirin to lessen the gastrointestinal distress. Celecoxib and nabumetone are both COX-2 inhibitors.

A patient is taking aspirin to help prevent myocardial infarction and is experiencing moderate gastrointestinal upset. The nurse will contact the patient's provider to discuss changing from aspirin to which drug? a. A COX-2 inhibitor b. Celecoxib (Celebrex) c. Enteric-coated aspirin d. Nabumetone (Relafen)

ANS: A With treatment, ulcer pain may subside in 10 days, but the healing process may take 1 to 2 months. Patients should be counseled to take the drug for the length of time prescribed. Reducing the dose or taking less frequently is not indicated.

A patient is taking esomeprazole (Nexium) 15 mg per day to treat a duodenal ulcer. After 10 days of treatment, the patient reports that the pain has subsided. The nurse will counsel the patient to a. continue the medication for 4 more weeks. b. reduce the medication dose by half. c. stop taking the medication. d. take the medication every other day.

ANS: B Methadone is a category II drug, with a high potential for drug abuse.

A patient is taking methadone as part of a heroin withdrawal program. The nurse understands that, in this instance, methadone is classified as which drug schedule? a. C-I b. C-II c. C-III d. C-V

Answer: D RATIONALE: Theophylline is a methylxanthine that provides benefits through bronchodilation. It is used to reduce the frequency and severity of asthma attacks, especially those occurring at night. Caffeine also is a methylxanthine, and its pharmacologic actions may intensify the adverse effects of theophylline on the central nervous system (CNS) and heart. Sources of caffeine should be avoided. It is not necessary to avoid taking cimetidine or echinacea or using sunscreen products while taking theophylline.

A patient is taking oral theophylline for maintenance therapy of stable asthma. A nurse instructs the patient to avoid using which substance to prevent a complication? A Echinacea B Cimetidine (Tagamet) C Sunscreen products D Caffeine

ANS: B Phenytoin is protein bound. When patients have a low serum albumin, there are fewer protein-binding sites, leaving more free drug in the system. The nurse should expect an increase in sedative side effects.

A patient is taking phenytoin to prevent seizures. The nurse knows that phenytoin is highly protein-bound and has sedative side effects. The nurse reviews the patient's chart and notes a low serum albumin. The nurse will notify the provider and observe the patient for which effects? a. Decreased sedative effects b. Increased sedative effects c. Increased seizures d. No change in effects

Answer: D Rationale: A beta agonist dilates respiratory smooth muscle, but as a side effect, it can stimulate the heart. A heart rate of 110 beats per minute is a concern, because this medication may further increase the already elevated heart rate. A pulse oximetry reading of 88% is a concern, but the medication causes bronchodilation and increased oxygenation; this should increase the pulse oximetry reading. A blood pressure of 100/60 mm Hg is on the low side, but this medication may actually cause an increase in blood pressure as a side effect; this should not concern the nurse before administration of the medication. A respiratory rate of 28 breaths per minute is elevated; however, this medication should increase oxygenation by bronchodilation, and the patient's respiratory rate should decrease once oxygenation has improved. Therefore, this should not concern the nurse.

A patient is to receive a beta agonist. Before administration of this medication, which assessment finding would most concern the nurse? a. Pulse oximetry reading of 88% b. Blood pressure of 100/60 mm Hg c. Respiratory rate of 28 breaths per minute d. Heart rate of 110 beats per minute

Answer: D RATIONALE: The hormone insulin promotes the passage of glucose into cells, where it is metabolized for energy. Insulin does not stimulate the pancreas to reabsorb glucose or synthesize amino acids into glucose. It does not stimulate the liver to convert glycogen into glucose.

A patient newly diagnosed with type 1 diabetes asks a nurse, "How does insulin normally work in my body?" The nurse explains that normal insulin has which action in the body? A It stimulates the pancreas to reabsorb glucose. B It promotes the synthesis of amino acids into glucose. C It stimulates the liver to convert glycogen to glucose. D It promotes the passage of glucose into cells for energy.

ANS: C Generic drugs are approved by the FDA if they are proved to be bioequivalent to the brand name drug. They tend to be less expensive because manufacturers of these drugs do not have to do the extensive testing required of brand-name drugs before marketing. They are not identical to brand-name drugs and often have different inert ingredients.

A patient receives a prescription on which the provider has noted that a generic medication may be given. The patient asks the nurse what this means. What will the nurse tell the patient about generic drugs? a. They contain the same inert ingredients as brand-name drugs. b. They have chemical structures that are identical to proprietary drugs. c. They tend to be less expensive than brand-name drugs. d. They undergo extensive testing before they are marketed.

Answer: C Rationale: The patient is displaying adverse reactions to theophylline, and her blood level should be assessed before another dose of the medication. The nurse should hold the medication.

A patient taking an oral theophylline drug is due for her next dose and has a blood pressure of 100/50 mm Hg and a heart rate of 110 bpm. The patient is irritable. What is the nurse's best action? A Continue to monitor the patient. B Call the health care provider. C Hold the next dose of theophylline. D Administer oxygen 2 L per minute via nasal cannula.

ANS: A A sore throat can indicate a life threatening anemia, so a complete blood count with differential should be ordered.

A patient taking trimethoprim-sulfamethoxazole (TMP-SMX) to treat a urinary tract infection complains of a sore throat. The nurse will contact the provider to request an order for which laboratory test(s)? a. Complete blood count with differential b. Throat culture c. Urinalysis d. Coagulation studies

Answer: C Resolution of pneumonia RATIONALE: Trimethoprim/sulfamethoxazole is the treatment of choice for Pneumocystis pneumonia (PCP), an infection caused by Pneumocystis jiroveci (formerly thought to be Pneumocystis carinii). PCP is an opportunistic pneumonia caused by a fungus that thrives in immunocompromised hosts. It does not increase the number of CD4 T cells, the targeted cells of the human immunodeficiency virus (HIV), nor does it affect joint pain. Increased appetite and weight gain are not therapeutic actions of trimethoprim/sulfamethoxazole.

A patient who has acquired immunodeficiency syndrome (AIDS) is receiving trimethoprim/sulfamethoxazole (Bactrim). Which response should a nurse expect if the medication is achieving the desired effect? A Increase in CD4 T cells B Increased appetite and weight gain C Resolution of pneumonia D Decrease in joint pain

ANS: B Metoprolol is a selective adrenergic blocker that has a greater affinity for receptors that decrease heart rate and blood pressure and is less likely to cause bronchospasm. The other adrenergic blockers are not selective and can cause bronchoconstriction.

A patient who has asthma is diagnosed with hypertension. The nurse understands that which drug will be safe to give this patient? a. Pindolol (Visken) b. Metoprolol (Lopressor) c. Nadolol (Corgard) d. Propranolol (Inderal)

ANS: B Spacers are used to enhance the delivery of medication to the lower, smaller airways. They do not allow higher dosing or minimize drug side effects.

A patient who has asthma will begin taking an inhaled corticosteroid medication to be used with a spacer. The patient asks why the spacer is necessary. The nurse will explain that the spacer a. allows a larger dose to be given safely. b. distributes medication to target tissues. c. minimizes adverse effects of the steroid. d. prevents contamination of the metered-dose inhaler.

ANS: D Digoxin is given for atrial fibrillation to restore a normal heart rhythm. To prevent thromboemboli, warfarin is given concurrently. Hydrochlorothiazide is a diuretic medication. Inamrinone and milrinone are inotropic agents that would be used instead of digoxin.

A patient who has atrial fibrillation is taking digoxin. The nurse expects which medication to be given concurrently to treat this condition? a. Hydrochlorothiazide (HydroDIURIL) b. Inamrinone (Inocor) c. Milrinone (Primacore) d. Warfarin (Coumadin)

ANS: C Metformin can lead to renal failure. It does not produce hypoglycemia or hyperglycemia. It does not increase the risk of respiratory distress.

A patient who has been taking a sulfonylurea antidiabetic medication will begin taking metformin (Glucophage). The nurse understands that this patient is at increased risk for which condition? a. Hypoglycemia b. Hyperglycemia c. Renal failure d. Respiratory distress

ANS: D Verapamil is a calcium channel blocker and is used to relax coronary artery spasm in patients with variant angina. It does not facilitate coronary muscle oxygen use, improve renal function, or increase cardiac contractility.

A patient who has been taking nitroglycerin for angina has developed variant angina, and the provider has added verapamil (Calan) to the patient's regimen. The nurse will explain that verapamil is given for which purpose? a. To facilitate oxygen use by the heart b. To improve renal perfusion c. To increase cardiac contractility d. To relax coronary arteries

ANS: D This patient does not have H. pylori ulcer disease, so dual and triple drug therapy with antibiotics is not indicated. Patients who fail treatment with a histamine2 blocker should be changed to a proton pump inhibitor (PPI) such as lansoprazole. PPIs tend to inhibit gastric acid secretion up to 90% greater than the histamine antagonists.

A patient who has been taking ranitidine (Zantac) continues to have pain associated with peptic ulcer. A noninvasive breath test is negative. Which treatment does the nurse expect the provider to order for this patient? a. Adding an over-the-counter antacid to the patient's drug regimen b. A dual drug therapy regimen c. Amoxicillin (Amoxil), clarithromycin (Biaxin), and omeprazole (Prilosec) d. Lansoprazole (Prevacid) instead of ranitidine

ANS: A Ephedra is a stimulant that potentiates theophylline and may increase side effects. Patients should be questioned about use of herbal medications. To determine toxicity, serum drug levels must be drawn; at this point, the patient reports symptoms of theophylline side effects. Increasing fluid intake will not alleviate symptoms. Renal function studies are not indicated.

A patient who has been taking theophylline at home reports having palpitations and jitteriness. What action will the nurse take? a. Ask the patient if herbal medications are used. b. Notify the provider to report theophylline toxicity. c. Recommend that the patient increase fluid intake. d. Request an order for renal function studies.

ANS: D Hypotension is a common effect of calcium channel blockers and is more common with nifedipine. It is less common with diltiazem, so the provider may order that drug. Adding digoxin, changing to a beta blocker, or ordering serum liver enzymes are not indicated.

A patient who has begun taking nifedipine (Procardia) to treat variant angina has had a recurrent blood pressure of 90/60 mm Hg or less. The nurse will anticipate that the provider will a. add digoxin to the drug regimen. b. change to a beta blocker. c. order serum liver enzymes. d. switch to diltiazem (Cardizem).

ANS: B Using two or more chemotherapeutic agents can have a synergistic effect. Combination therapy typically uses two drugs with different dose limiting toxicities, but the use of more than one drug does not allow for using less toxic doses. Combination therapy allows cell kill in all phases of the cell cycle. Combination therapy does not shorten the length of time chemotherapy is needed.

A patient who has cancer is about to begin chemotherapy. The patient asks the nurse why two chemotherapeutic agents are being used instead of just one. Which response by the nurse is correct? a. "The drugs may be given in less toxic doses if two drugs are used." b. "Two agents used together can have synergistic effects." c. "Use of two drugs will increase tumorcidal activity in the G0 phase of the cell." d. "Using two agents will shorten the length of time chemotherapy is needed."

ANS: B Patients who have exposure to TB should have a TB skin test. A chest x ray is performed if the skin test is positive. LFTs do not need to be done simply because of TB exposure. This patient is not a candidate for antitubercular drug prophylaxis.

A patient who has chronic liver disease reports contact with a person who has tuberculosis (TB). The nurse will counsel this patient to contact the provider to discuss a. a chest x-ray. b. a TB skin test. c. liver function tests (LFTs). d. prophylactic antitubercular drugs.

ANS: A Anticonvulsants are given to prevent seizures and are usually taken throughout the patient's lifetime. Stopping the medication will lead to recurrence of seizures in most patients. Some patients may attempt to stop taking the medications after 3 to 5 years of no seizure activity. Anticonvulsants are not given as needed.

A patient who has epilepsy will begin an anticonvulsant medication. The patient asks the nurse how long the medication will be necessary. How will the nurse respond? a. "The medication is usually taken for a lifetime." b. "The medication will be given until you are seizure-free." c. "You will need to take the medication for 3 to 5 years." d. "You will take the medication as needed for seizure activity."

ANS: D Spironolactone is a potassium sparing diuretic that blocks production of aldosterone, causing improved heart rate variability and decreased myocardial fibrosis. It is given in congestive heart failure for its cardioprotective effects. Spironolactone does not directly alter cardiac contractility but may slightly decrease contractility if fluid volume is decreased. It is a mild diuretic but is not given in this instance to minimize fluid losses.

A patient who has heart failure receives digoxin (Lanoxin) and an angiotension-converting enzyme (ACE) inhibitor. The patient will begin taking spironolactone (Aldactone). The patient asks why the new drug is necessary. The nurse will tell the patient that spironolactone will be given for which reason? a. To enhance potassium excretion b. To increase cardiac contractility c. To minimize fluid losses d. To provide cardioprotective effects

ANS: D Glucocorticoids can cause hyperglycemia, so the insulin dose may need to be increased. Changing the glucocorticoid dose is not recommended. Decreasing the insulin dose will only compound the hyperglycemic effects.

A patient who has insulin-dependent diabetes mellitus must take a glucocorticoid medication for osteoarthritis. When teaching this patient, the nurse will explain that there may be a need to a. decrease the glucocorticoid dose. b. decrease the insulin dose. c. increase the glucocorticoid dose. d. increase the insulin dose.

ANS: B Carbidopalevodopa can cause orthostatic hypotension, so patients should be taught to take care when getting out of bed or a chair. Darkening of the urine and perspiration is a harmless side effect. Patients should take the drug with low-protein foods to improve drug transport to the CNS. Carbidopa-levodopa should not be discontinued abruptly because rebound parkinsonism may occur; insomnia is an expected adverse effect of the drug, and the patient should report this effect to his or her health care provider.

A patient who has parkinsonism will begin taking carbidopa-levodopa. What information will the nurse include when teaching this patient about this medication? a. "Call your health care provider immediately if your urine or perspiration turn a dark color." b. "Rise slowly from your bed or your chair to avoid dizziness and falls." c. "Take the drug with foods high in protein to improve drug delivery." d. "Discontinue the drug if you experience insomnia."

ANS: A Enoxaparin is a low molecular-weight heparin, which produces more stable responses at lower doses, thus reducing the need for frequent lab monitoring. It has a longer half-life than heparin. It decreases the risk of hemorrhage because it is more stable at lower doses. It is given subcutaneously.

A patient who has received heparin after previous surgeries will be given enoxaparin sodium (Lovenox) after knee-replacement surgery. The patient asks how this drug is different from heparin. The nurse will explain that enoxaparin a. decreases the need for laboratory tests. b. has a shorter half-life than heparin. c. increases the risk of hemorrhage. d. may be taken orally instead of subcutaneously.

ANS: C An angiotensin converting enzyme (ACE) inhibitor, such as captopril, can cause a constant, irritated cough. The cough will stop with discontinuation of the drug, and many patients can switch to an ARB medication. It does not indicate a hypersensitivity reaction. The cough will not diminish while still taking the drug. The patient does not need to stop taking the drug immediately.

A patient who has recently begun taking captopril (Capoten) to treat hypertension calls a clinic to report a persistent cough. The nurse will perform which action? a. Instruct the patient to go to an emergency department because this is a hypersensitivity reaction. b. Reassure the patient that this side effect is nothing to worry about and will diminish over time. c. Schedule an appointment with the provider to discuss changing to an angiotensin II receptor blocker (ARB). d. Tell the patient to stop taking the drug immediately since this is a serious side effect of this drug.

ANS: A Aspirin is often used with clopidogrel to inhibit platelet aggregation to increase the effectiveness of this drug. Enoxaparin is used to prevent venous thrombosis. Ticagrelor is similar to clopidogrel and is not used along with clopidogrel. Warfarin is used to prevent thrombosis.

A patient who has recently had a myocardial infarction (MI) will begin taking clopidogrel (Plavix) to prevent a second MI. Which medication will the nurse expect the provider to order as adjunctive therapy for this patient? a. Aspirin b. Enoxaparin sodium (Lovenox) c. Ticagrelor (Brilinta) d. Warfarin (Coumadin)

ANS: A Avoiding extreme weather conditions is important to help prevent anginal attacks. Patients should be instructed to avoid strenuous exercise; avoid alcohol, which can enhance hypotensive effects of nitrates; and use nitroglycerin at the first sign of pain.

A patient who has stable angina pectoris is given nitroglycerin to use as needed. In addition to pharmacotherapy, the nurse will give the patient which instruction? a. Avoid extremes in weather. b. Begin a rigorous exercise program. c. Drink glass of red wine daily. d. Seek medical care at first sign of pain.

ANS: C Without multi drug therapy, patients easily develop resistance to antitubercular drugs. Using more than one antitubercular drug does not prevent relapse, hypersensitivity reactions, or adverse effects.

A patient who has tuberculosis asks the nurse why three drugs are used to treat this disease. The nurse will explain that multi-drug therapy is used to reduce the likelihood of a. disease relapse. b. drug hypersensitivity reactions. c. drug resistance. d. drug adverse effects.

Answer: A RATIONALE: Dose-related peripheral neuropathy is the most common adverse effect of isoniazid. It results from a vitamin B6 deficiency, which is corrected by taking oral supplements. Symptoms include numbness and tingling in the fingers and toes. Alopecia and flaking scalp, oral ulcers and tongue fissures, and dry skin and brittle nails are not adverse effects of isoniazid-induced vitamin B6 deficiency.

A patient who has tuberculosis is treated with isoniazid. The nurse should monitor for which symptoms, which could indicate a vitamin B6 deficiency caused by the medication? A Numbness and tingling in the fingers and toes B Alopecia and flaking scalp C Dry skin and brittle nails D Oral ulcers and tongue fissures

ANS: A Combination products are convenient because the patient does not have to mix insulin, but the products depend on individual needs, since the doses are fixed. They are not used for patients with insulin resistance. Patients must continue to rotate injection sites. They do not require refrigeration after first use.

A patient who has type 1 diabetes mellitus asks the nurse about using a combination insulin product such as Humalog 75/25. The nurse will tell the patient that use of this product a. depends on individual insulin needs. b. is useful for patient with insulin resistance. c. means less rotation of injection sites. d. requires refrigeration at all times.

ANS: A Patients should not take aspirin during the third trimester of pregnancy because it can cause premature closure of the ductus arteriosus in the fetus. It does not increase her risk of Reye's syndrome. Aspirin taken within a week of delivery will increase the risk of bleeding. It can cause gastrointestinal distress, but this is not the reason for caution.

A patient who is 7 months pregnant and who has arthritis asks the nurse if she can take aspirin for pain. The nurse will tell her not to take aspirin for which reason? a. It can result in adverse effects on her fetus. b. It causes an increased risk of Reye's syndrome. c. It increases hemorrhage risk. d. It will cause increased gastrointestinal distress.

ANS: B Patients should have dosage adjustments of NRTIs if creatinine clearance is less than 50 mL/min. The patient will need single dose medications so that adjustments can be made. Taking the medication prior to meals improves absorption of didanosine but does not alter the side effect of nausea for Combivir, which should subside in the next week or so. This combination product is not given once daily. Increasing fluid intake will not affect this patient's symptoms.

A patient who is HIV-positive begins therapy with the fixed-dose combination nucleoside reverse transcriptase inhibitor (NRTI) Combivir (lamivudine/zidovudine) twice daily. The patient is in the clinic for follow-up 1 week after initiation of therapy and reports having nausea. The patient's creatinine clearance is 40 mL/minute. Based on these findings, the nurse will perform which action? a. Instruct the patient to take the medication 60 minutes prior to meals. b. Notify the provider to discuss single-dose NRTI products. c. Request an order for once-daily dosing of this medication. d. Suggest that the patient increase fluid intake.

ANS: A Following chemotherapy administration, the time at which the blood count, including white blood cells, is lowest is typically 7 to 10 days after treatment.

A patient who is about to begin chemotherapy asks the nurse when the risk of infection is highest. The nurse will tell the patient that infection risk is greatest at which point? a. A week to 10 days after each chemotherapy dose b. During the week immediately after chemotherapy c. Immediately prior to each dose of chemotherapy d. When the patient's temperature is elevated by 1° F

ANS: A Acute retroviral syndrome often occurs 2 to 12 weeks after exposure and is caused by rapid viral replication that triggers an immune response, resulting in CD4 cell replacement and HIV antibody production that causes the viral load to drop. This patient is experiencing symptoms of this syndrome. AIDS is a diagnosis that indicates advanced disease. Opportunistic infection symptoms are related to the type of infection.

A patient who is newly diagnosed with HIV infection after a recent exposure calls to report fever, sore throat, myalgia, and night sweats. The nurse will notify the provider that this patient is most likely experiencing a. acute retroviral syndrome. b. AIDS. c. an increased viral load. d. an opportunistic infection.

ANS: C Insulin, growth hormones, and other protein based drugs are destroyed in the small intestine by digestive enzymes and must be given parenterally. Because insulin is destroyed by digestive enzymes, it would not make it to the liver for metabolism with a first-pass effect. Subcutaneous tissue has fewer blood vessels, so absorption is slower in such tissue. Insulin is given subcutaneously because it is desirable to have it absorb slowly.

A patient who is newly diagnosed with type 1 diabetes mellitus asks why insulin must be given by subcutaneous injection instead of by mouth. The nurse will explain that this is because a. absorption is diminished by the first-pass effects in the liver. b. absorption is faster when insulin is given subcutaneously. c. digestive enzymes in the gastrointestinal tract prevent absorption. d. the oral form is less predictable with more adverse effects.

ANS: C In general, the Filipino culture expects that family members will stay at a patient's bedside and participate in his or her care. The nurse should include the family in appropriate tasks. It is not correct to ask the family to wait in the hall or to sit in chairs and not participate. Filipino families do not necessarily depend on family patriarchs.

A patient who is of Filipino descent is admitted to the hospital. The nurse goes to the room to start intravenous fluids and to perform an admission assessment and finds several family members in the room. Which action by the nurse is appropriate? a. Ask the family to wait in the hallway until the admission tasks are completed. b. Determine which family member is the family patriarch and address questions to him. c. Invite family members to assist with appropriate tasks during the admission process. d. Provide chairs for family members and ask them to stay seated during the admission.

ANS: D Patients with a hemoglobin A1c between 5.7% and 6.4% are considered to have prediabetes. A level of 6.5% or more indicates diabetes. The patient is hyperglycemic.

A patient who is overweight is being evaluated for diabetes. The patient has a blood glucose level of 160 mg/dL and a hemoglobin A1c of 5.8%. The nurse understands that this patient has which condition? a. Diabetes mellitus b. Hypoglycemia c. Normal blood levels d. Prediabetes

Answer: D RATIONALE: Aminoglycoside-induced nephrotoxicity usually presents as acute tubular necrosis. Symptoms of concern are protein in the urine, dilute urine, and elevation of the serum creatinine and blood urea nitrogen (BUN) levels. WBCs, glucose, and ketones are not specifically related to gentamicin use.

A patient who is receiving an aminoglycoside (gentamicin) has a urinalysis result with all of these findings. Which finding should a nurse associate most clearly with an adverse effect of gentamicin? A White blood cells (WBCs) B Glucose C Ketones D Protein

Answer: D RATIONALE: Because of its chemical size and weight, vancomycin is absorbed poorly in the gastrointestinal (GI) tract and is given parenterally for most infections. It is used for serious infections caused by organisms such as MRSA and in patients with susceptible organisms allergic to penicillins. Oral administration is used only for infections of the intestine. It is not associated with loss of appetite or nausea.

A patient who is receiving vancomycin (Vancocin) IV for a methicillin-resistant Staphylococcus aureus (MRSA) infection asks a nurse, "Why can't I take this medicine in a pill?" Which response should the nurse make? A "The prescription could be changed, because vancomycin comes in two forms." B "You're allergic to penicillin, and this is the only way this medication can be given." C "It will cause too much loss of appetite and nausea if given in the oral form." D "It is more effective by IV, because the pill form will stay in the digestive tract."

ANS: B Good oral care can prevent gingival hyperplasia in patients with HSV-1.

A patient who is taking acyclovir (Zovirax) to treat an oral HSV-1 infection asks the nurse why oral care is so important. The nurse will tell the patient that meticulous oral care helps to a. minimize transmission of disease. b. prevent gingival hyperplasia. c. reduce viral resistance to the drug. d. shorten the duration of drug therapy.

ANS: C Aspirin is a COX 1 and COX-2 inhibitor. COX-1 protects the stomach lining, so when it is inhibited, gastric upset occurs. Aspirin does not increase gastrointestinal secretions or hypersensitivity reactions. It is a weak acid.

A patient who is taking aspirin for arthritis pain asks the nurse why it also causes gastrointestinal upset. The nurse understands that this is because aspirin a. increases gastrointestinal secretions. b. increases hypersensitivity reactions. c. inhibits both COX-1 and COX-2. d. is an acidic compound.

ANS: D Because both drugs can prolong bleeding time, patients should discontinue the drugs 7 days prior to surgery.

A patient who is taking clopidogrel (Plavix) and aspirin is preparing for orthopedic surgery. The nurse will consult with the surgeon and provide which instruction to the patient? a. Continue taking aspirin and stop taking clopidogrel 2 weeks prior to surgery. b. Continue taking clopidogrel and stop taking aspirin 5 days prior to surgery. c. Continue both medications to prevent thromboembolic events during surgery. d. Stop taking both medications 7 days prior to surgery.

ANS: D Peripheral neuropathy is an adverse reaction to INH, so pyridoxine is usually given to prevent this. It is not necessary to change medications. Increasing fluids will not help with this.

A patient who is taking isoniazid (INH) as part of a two-drug tuberculosis treatment regimen reports tingling of the fingers and toes. The nurse will recommend discussing which treatment with the provider? a. Adding pyrazinamide b. Changing to ethambutol c. Increasing oral fluid intake d. Taking pyridoxine (B6)

ANS: B Reddish brown urine is a harmless side effect of metronidazole and is not cause for concern.

A patient who is taking metronidazole (Flagyl) reports reddish-brown urine. Which action will the nurse take? a. Obtain an order for BUN and creatinine levels. b. Reassure the patient that this is a harmless effect. c. Request an order for a urinalysis. d. Test her urine for occult blood.

ANS: C Vitamin K is an antagonist against warfarin, an oral anticoagulant. Patients with an INR of 5.5 should be given a low dose of oral vitamin K. Too much vitamin K may reduce the effectiveness of warfarin for up to 2 weeks. Fresh frozen plasma and intravenous iron are given for anemia caused by blood loss. Protamine sulfate is given for heparin overdose.

A patient who is taking warfarin has an international normalized ratio (INR) of 5.5. The nurse will anticipate giving a. fresh frozen plasma. b. intravenous iron. c. oral vitamin K. d. protamine sulfate.

ANS: C Opioid overdose is characterized by constricted pupils and respiratory depression.

A patient who is unconscious arrives in the emergency department with clammy skin and constricted pupils. The nurse assesses a respiratory rate of 8 to 10 breaths per minute. The paramedics report obvious signs of drug abuse in the patient's home. The nurse suspects that this patient has had an overdose of which substance? a. Alcohol b. LSD c. An opioid d. Methamphetamine

ANS: A, B, E Hot, spicy foods aggravate gastric upset, tobacco increases gastric secretions, and ibuprofen on an empty stomach increases gastric secretions, so patients should be taught to avoid these actions. Alcohol should be avoided since it increases gastric secretions. Eating at bedtime increases reflux, as does laying relatively flat to sleep, or wearing fitted clothing.

A patient who recently began having mild symptoms of GERD is reluctant to take medication. What measures will the nurse recommend to minimize this patient's symptoms? (Select all that apply.) a. Avoiding hot, spicy foods b. Avoiding tobacco products c. Drinking a glass of red wine with dinner d. Eating a snack before bedtime e. Taking ibuprofen with food f. Using a small pillow for sleeping g. Wearing well-fitted clothing

ANS: B Opioids slow gastric emptying, allowing more time for drugs absorbed in the stomach to be absorbed. The nurse should expect increased effects of the first drug.

A patient who takes a drug that undergoes gastric absorption will begin taking an opioid analgesic after sustaining an injury in a motor vehicle accident. The nurse will observe the patient closely for which effects? a. Decreased effects of the first drug b. Increased effects of the first drug c. Decreased effects of the narcotic d. Increased effects of the narcotic

Answer: B RATIONALE: Metronidazole is the treatment of choice for antibiotic-associated colitis caused by C. difficile. Rifaximin, daptomycin, and gemifloxacin are not used in the treatment of C. difficile infection.

A patient who takes multiple antibiotics starts to experience diarrheal stools. The nurse anticipates administration of which antibiotic if a stool sample tests positive for Clostridium difficile? A Rifaximin (Xifaxan) B Metronidazole (Flagyl) C Daptomycin (Cubicin) D Gemifloxacin (Factive)

ANS: A Chronic ingestion of alcohol increases hydantoin metabolism, which would decrease serum drug levels. The therapeutic range is 10 to 20 mcg/mL, so a level lower than this may be expected in patients who consume alcohol regularly.

A patient who takes phenytoin reports regular alcohol consumption. The nurse might expect a serum phenytoin level in this patient to be in which range? a. 5 to 10 mcg/mL b. 10 to 20 mcg/mL c. 20 to 30 mcg/mL d. 30 to 50 mcg/mL

ANS: A Aspirin and NSAIDs can increase the risk of hemorrhage in patients taking anticoagulants. Acetaminophen is safe and may be recommended.

A patient who takes warfarin (Coumadin) asks the nurse about taking a medication for headaches. The nurse will recommend which medication? a. Acetaminophen b. Aspirin c. Ibuprofen d. No medication

Answer: C RATIONALE: The patient is showing symptoms of hypoglycemia at 5 PM. NPH has a peak action of 8 to 10 hours after administration. Based on the duration of action of NPH insulin, the patient's hypoglycemic symptoms are from the 8 AM injection of NPH insulin. An injection of NPH insulin at 2 AM, 1 PM, or 3 PM would not cause hypoglycemic symptoms based on the average duration of action of NPH insulin.

A patient who took NPH insulin at 0800 reports feeling weak and tremulous at 1700. Which action should the nurse take? A Take the patient's blood pressure. B Give the patient's PRN dose of insulin. C Check the patient's capillary blood sugar. D Advise the patient to lie down with the legs elevated.

ANS: D When using inhaled glucocorticoid medications, Candida albicans oropharyngeal infections may be prevented by rinsing the mouth and throat with water after each dose. Patients should also use a spacer to reduce deposits of the drug in the oral cavity. The inhaler should be washed with warm water daily, but not after each use. There is no indication that yogurt is effective.

A patient who uses an inhaled glucocorticoid medication reports having a sore tongue. The nurse notes white spots on the patient's tongue and oral mucous membranes. After notifying the provider, the nurse will remind the patient to perform which action? a. Avoid using a spacer with the inhaled glucocorticoid medication. b. Clean the inhaler with hot, soapy water after each use. c. Consume yogurt daily while using this medication. d. Rinse the mouth thoroughly with water after each use.

ANS: D Headaches are one of the most common side effects of nitroglycerin, but they may become less frequent; acetaminophen is generally recommended for pain. The headaches are not an emergency, and the patient does not need to call 911 or notify the provider. The patch is applied once daily.

A patient who uses transdermal nitroglycerin reports having headaches. The nurse will counsel the patient to perform which action? a. Call 911 when this occurs. b. Notify the provider. c. Reapply the patch three times daily. d. Take acetaminophen as needed.

ANS: D Patients often are more motivated to adhere to a drug regimen if they understand the purpose of the medications. Patients should be encouraged to take responsibility for their medications. Side effects need to be discussed so patients can plan ways to manage these before they occur.

A patient who will begin antiretroviral therapy reports having trouble sticking with drug regimens in the past. Which action will the nurse take? a. Ask the patient's family members to administer the medications. b. Avoid discussing adverse effects to prevent focus on negative aspects of ART. c. Give a detailed list of medications and stress the need to adhere to the schedule. d. Offer written and verbal information about each drug's purpose.

ANS: B The combination drug is used to decrease bacterial resistance to sulfonamides. It does not broaden the spectrum, improve the taste, or decrease toxicity.

A patient who will begin taking trimethoprim-sulfamethoxazole (TMP-SMX) asks the nurse why the combination drug is necessary. The nurse will explain that the combination is used to a. broaden the antibacterial spectrum. b. decrease bacterial resistance. c. improve the taste. d. minimize toxic effects.

ANS: D Excessive use of an aerosol drug can occasionally cause severe paradoxical airway resistance, so patients should be cautioned against overuse. Excessive use can also lead to tolerance and loss of drug effectiveness, but patients should not increase the dose because of the risk of bronchospasm and the increased incidence of adverse effects such as tremors and tachycardia. Hyperglycemia can occur. Palpitations are common with increased doses but not at therapeutic doses.

A patient will be discharged home with albuterol (Proventil) to use for asthma symptoms. What information will the nurse include when teaching this patient about this medication? a. Failure to respond to the medication indicates a need for a higher dose. b. Monitor for hypoglycemia symptoms when using this medication. c. Palpitations are common with this drug even at normal, therapeutic doses. d. Overuse of this medication can result in airway narrowing and bronchospasm.

ANS: C Because of the action and side effects of beta blockers, heart rate and blood pressure should be monitored frequently.

A patient will be discharged on beta blockers. Which skill is essential for the nurse to teach the patient's family? a. How to prepare a low-sodium diet b. Assessments to detect fluid retention c. How to monitor heart rate and blood pressure d. Early signs of changing level of consciousness

ANS: A High dosages of albuterol may affect beta1 receptors, causing an increase in heart rate that could be dangerous. It is not necessary to consume extra fluids or take a calcium supplement while using this drug. Serum glucose may be elevated slightly, but this is not a concern in non diabetic patients.

A patient will begin taking albuterol (Proventil) to treat asthma. When teaching the patient about this drug, the nurse will make which recommendation? a. Report rapid or irregular heart rate. b. Drink 8 to 16 extra ounces of fluid each day. c. Monitor serum glucose daily. d. Take a calcium supplement.

ANS: D Acidic fruits and juices should be avoided while the client is being treated with amoxicillin because amoxicillin can be irritating to the stomach. Stomach irritation will be increased with the ingestion of citrus and acidic foods. Amoxicillin may also be less effective when taken with acidic fruit or juice.

A patient will begin taking amoxicillin. The nurse should instruct the patient to avoid which foods? a. Green leafy vegetables b. Beef and other red meat c. Coffee, tea, and colas d. Acidic fruits and juices

ANS: A Patients taking antihyperlipidemics should be advised to increase fluid intake. It is not necessary to take with food. Patients should never stop taking a statin without consulting the provider. Patients should continue a low fat diet while taking statins.

A patient will begin taking rosuvastatin calcium (Crestor) to treat hyperlipidemia. The patient asks the nurse how to take the medication for best effect. Which statement by the nurse is correct? a. "Increase your fluid intake while taking this medication." b. "Stop taking the medication if you develop muscle aches." c. "Take the medication with food to improve absorption." d. "You may increase dietary fat while taking this medication."

ANS: C Simvastatin is given in the evening. Laboratory tests are performed every 3 to 6 months, not annually. Statins do not cause postural hypotension. Patients taking statins should report muscle aches and weakness immediately.

A patient will begin taking simvastatin (Zocor) to decrease serum cholesterol. When teaching the patient about this medication, the nurse will counsel the patient to take which action? a. Return to the clinic annually for laboratory testing. b. Take care when rising from a sitting to standing position. c. Take the medication in the evening for best effect. d. Use ibuprofen as needed for muscle aches and pain.

ANS: A Inhaled medications have more immediate effects than oral preparations. As long as they are used correctly, systemic side effects are less common. Less drug is needed for therapeutic effects, since the drug is delivered directly to target tissues. Increased doses will lead to drug tolerance.

A patient will begin using an albuterol metered dose inhaler to treat asthma symptoms. The patient asks the nurse about the difference between using an oral form of albuterol and the inhaled form. The nurse will explain that the inhaled form of albuterol a. has a more immediate onset than the oral form. b. may cause more side effects than the oral preparation. c. requires an increased dose in order to have therapeutic effects. d. will not lead to tolerance with increased doses.

ANS: A Inhaled medications have more immediate effects than oral preparations. As long as they are used correctly, systemic side effects are less common. Less drug is needed for therapeutic effects, since the drug is delivered directly to target tissues. Increased doses will lead to drug tolerance.

A patient will begin using an albuterol metered-dose inhaler to treat asthma symptoms. The patient asks the nurse about the difference between using an oral form of albuterol and the inhaled form. The nurse will explain that the inhaled form of albuterol a. has a more immediate onset than the oral form. b. may cause more side effects than the oral preparation. c. requires an increased dose in order to have therapeutic effects. d. will not lead to tolerance with increased doses.

ANS: A Patients who use a beta agonist should be taught to use it 5 minutes before administering ipratropium bromide, and ipratropium bromide should be given 5 minutes prior to an inhaled glucocorticoid. This helps the bronchioles to dilate so the subsequent medication can be deposited in the bronchioles for improved effect.

A patient will begin using ipratropium bromide (Atrovent), albuterol (Proventil), and an inhaled glucocorticoid medication (steroid) to treat chronic bronchitis. When teaching this patient about disease and medication management, the nurse will instruct the patient to administer these medications in which order? a. Albuterol, ipratropium bromide, steroid b. Albuterol, steroid, ipratropium bromide c. Ipratropium bromide, albuterol, steroid d. Steroid, ipratropium bromide, albuterol

ANS: B The treatment goal would be a VL of < 20 copies/mL and a CD4 cell count between 800 and 1200 cells/mm3. This goal should be achieved in 16 to 24 weeks. Since this patient has shown improvement, progress has been made, and treatment should continue. A drug resistant strain is not likely to respond to therapy. Treatment failure is not evident.

A patient with HIV infection has been receiving antiretroviral therapy for 2 months. At the initiation of treatment, the patient had a viral load (VL) of 60 copies/mL and a CD4 count of 450 cells/mm3. Today's lab results reveal a VL of 20 copies/mL and a CD4 cell count of 800 cells/mm3. How will the nurse interpret the patient's results? a. A drug-resistant strain is likely. b. The patient is progressing as expected. c. The patient's treatment goals have been met. d. Treatment failure has occurred.

Answer: C RATIONALE: Patients who have been taking levodopa/carbidopa for a period of time may experience episodes of symptom return. Adding other medications to the drug regimen can help minimize this phenomenon.

A patient with Parkinson's disease who has been positively responding to drug treatment with levodopa/carbidopa (Sinemet) suddenly develops a relapse of symptoms. Which explanation by the nurse is appropriate? A "You have apparently developed resistance to your current medication and will have to change to another drug." B "This is an atypical response. Unfortunately, there are no other options of drug therapy to treat your disease." C "This is called the 'on-off' phenomenon. Your healthcare provider can change your medication regimen to help diminish this effect." D "You should try to keep taking your medication at the current dose. These effects will go away with time."

Answer: B RATIONALE: Patients taking levodopa/carbidopa (Sinemet) are at increased risk for the psychiatric side effects of levodopa, including visual hallucinations, vivid dreams, nightmares, and paranoid ideation. The other questions are not directly related to problems that are likely to occur with this drug.

A patient with Parkinson's disease who takes levodopa/carbidopa (Sinemet) comes to the clinic for a semiannual physical examination. Which question is the most important for the nurse to ask? A "Have you noticed any swelling in your feet?" B "Are you having vivid dreams or visual hallucinations?" C "Have you noticed any changes in your stool?" D "Have you had your flu vaccine?"

Answer: C Rationale: In an acute asthmatic attack, the short-acting sympathomimetics are the first line of defense. A beta2-adrenergic agonist will provide immediate relief, while a glucocorticoid will not. There is no need to call a code.

A patient with a history of asthma is short of breath and says, "I feel like I'm having an asthma attack." What is the nurse's highest priority action? A Calling a code B Asking the patient to describe the symptoms C Administering a beta2 adrenergic agonist D Administering a long-acting glucocorticoid

ANS: B The use of two antibiotics when treating H. pylori peptic ulcer disease helps to combat bacterial resistance because H. pylori develops resistance rapidly. Giving two antibiotics, in this case, is not to reduce the dose or to cause synergistic effects. Antibiotics do not affect acid production.

A patient with a peptic ulcer has been diagnosed with H. pylori. The provider has ordered lansoprazole (Prevacid), clarithromycin (Biaxin), and metronidazole (Flagyl). The patient asks the nurse why two antibiotics are needed. The nurse will explain that two antibiotics a. allow for less toxic dosing. b. combat bacterial resistance. c. have synergistic effects. d. improve acid suppression.

ANS: B The BNP is used to differentiate that dyspnea is due to HF and not lung dysfunction. The other tests will all be a part of the diagnostic workup but do not help with this distinction.

A patient with chronic obstructive pulmonary disease (COPD) has increasing dyspnea and is being evaluated for heart failure (HF). Which test will be ordered to help differentiate between dyspnea due to lung dysfunction and dyspnea due to HF? a. Atrial natriuretic hormone (ANH) level b. Brain natriuretic peptide (BNP) level c. Cardiac enzymes d. Electrocardiogram (ECG)

Answer: B RATIONALE: Heparin is an anticoagulant that works by helping antithrombin inactivate thrombin and factor Xa, reducing the production of fibrin and thus decreasing the formation of clots.

A patient with deep vein thrombosis is receiving an intravenous (IV) heparin infusion. He asks the nurse how this medication will help him. The nurse's response is accurately based on which concept? A Heparin prevents the activation of vitamin K and thus blocks synthesis of some clotting factors. B Heparin suppresses coagulation by helping antithrombin perform its natural functions. C Heparin works by converting plasminogen to plasmin, which in turn dissolves the clot matrix. D Heparin inhibits the enzyme responsible for platelet activation and aggregation within vessels.

ANS: C, D, F This medication is most effective with careful monitoring of diet. Atorvastatin does not affect renal function.

A patient with high cholesterol is ordered to take atorvastatin (Lipitor). What information will be included in the patient teaching? (Select all that apply.) a. Dietary management is not a priority with this medication. b. The medication should be taken on an empty stomach. c. The medicine should be taken with a full glass of water. d. The patient should watch for body aches or gastrointestinal upset as side effects. e. The patient should have renal function tests frequently. f. The patient should have liver function tests frequently.

Answer: B RATIONALE: Trough levels are drawn when the blood level is at its lowest, which is typically just before the next dose is given (B). (A, C, and D) do not describe the optimum time for obtaining a trough level of an antibiotic.

A peak and trough level must be drawn for a client receiving antibiotic therapy. What is the optimum time for the nurse to obtain the trough level? A) Sixty minutes after the antibiotic dose is administered. B) Immediately before the next antibiotic dose is given. C) When the next blood glucose level is to be checked. D) Thirty minutes before the next antibiotic dose is given.

ANS: C Antiretroviral therapy is strongly recommended for all pregnant HIV infected patients. The preferred dual nucleoside reverse transcriptase inhibitor is Combivir.

A pregnant patient is HIV-positive. Which antiretroviral agent will the nurse expect the patient's provider to order? a. Abacavir/lamivudine/zidovudine (Trizivir) b. Efavirenz/emtricitabine/tenofovir (Atripla) c. Lamivudine/zidovudine (Combivir) d. Rilpivirine/emtricitabine/tenofovir (Complera)

Answer: A RATIONALE: Lispro is a rapid-acting insulin and has an onset of action of 15 to 30 minutes with a peak action of about 2 hours, not 8 to 10 hours. Because of its rapid onset, it is administered immediately before a meal or with meals to control the blood glucose rise after meals. Lispro insulin must be combined with an intermediate- or a long-acting insulin, not regular insulin (which also is a short-duration insulin), for glucose control between meals and at night. To achieve tight glycemic control, patients must combine different types of insulin based on their duration of action.

A teaching plan for a patient who is taking lispro (Humalog) should include which instruction by the nurse? A "Inject this insulin with your first bite of food, because it is very fast acting." B "The duration of action for this insulin is about 8 to 10 hours, so you'll need a snack." C "This insulin needs to be mixed with regular insulin to enhance the effects." D "To achieve tight glycemic control, this is the only type of insulin you'll need."

ANS: A Patients who are taking metronidazole can experience a disulfiram like reaction when they drink alcohol. These are not harmless adverse effects or a sign of worsening of her infection.

A young adult female who is taking metronidazole (Flagyl) to treat trichomoniasis calls the nurse to report severe headache, flushing, palpitations, cramping, and nausea. What will the nurse do next? a. Ask about alcohol consumption. b. Reassure her that these are harmless side effects. c. Tell her that this signals a worsening of her infection. d. Tell her to go to the emergency department immediately.

Answer: D

ACE inhibitors and angiotensin receptor blockers both work to decrease blood pressure by A preventing the formation of angiotensin II. B enhancing sodium and water resorption. C increasing the breakdown of bradykinin. D preventing aldosterone secretion.

Answer: A RATIONALE: Unfractionated heparin or low molecular weight heparin (LMWH) is an anticoagulant that inhibits thrombin-mediated conversion of fibrinogen to fibrin and is given prophylactically to prevent postoperative venous thrombosis (A) or to treat pulmonary embolism or deep vein thrombosis following knee and abdominal surgeries. Heparin does not dissolve clots but prevents clot extension or further clot formation (C). The anticoagulant heparin does not prevent infection (B) or influence operative wound healing (D).

After abdominal surgery, a male client is prescribed low molecular weight heparin (LMWH). During administration of the medication, the client asks the nurse why he is receiving this medication. Which is the best response for the nurse to provide? A) This medication is a blood thinner given to prevent blood clot formation. B) This medication enhances antibiotics to prevent infection. C) This medication dissolves any clots that develop in the legs. D) This abdominal injection assists in the healing of the abdominal wound.

ANS: A Penicillins, cephalosporins, tetracyclines, and sulfonamides are normally considered safe for the older adult. Aminoglycosides are excreted in the urine and are not usually prescribed for patients older than 75 years.

An 80-year-old patient is being treated for an infection. An order for which type of antibiotic would cause concern for the nurse caring for this patient? a. Aminoglycoside b. Cephalosporin c. Penicillin d. Sulfonamide

Answer: A RATIONALE: Morphine and ketorolac (Toradol) can be administered concurrently (A), and may produce an additive analgesic effect, resulting in the ability to reduce the dose of morphine, as seen in this prescription. Toradol is an antiinflammatory analgesic, and does not have an antagonistic effect with morphine (B), like an agonist-antagonist medication would have. An additive analgesic effect is desirable (C), because it allows a reduced dose of morphine. This prescription does not require any clarification, and can be administered safely as written (D).

An adult client has prescriptions for morphine sulfate 2.5 mg IV q6h and ketorolac (Toradol) 30 mg IV q6h. Which action should the nurse implement? A) Administer both medications according to the prescription. B) Hold the ketorolac to prevent an antagonistic effect. C) Hold the morphine to prevent an additive drug interaction. D) Contact the healthcare provider to clarify the prescription.

ANS: A High doses of albuterol may affect beta1 receptors, causing an increase in heart rate. Patients having an asthma exacerbation may over use their albuterol inhalers when seeking relief. Patients may have wheezing and increased heart rate during an untreated asthma exacerbation, but they will not have tremors and restlessness.

An adult patient is brought to the emergency department for treatment of an asthma exacerbation. The patient uses inhaled albuterol as needed to control wheezing. The nurse notes expiratory wheezing, tremors, restlessness, and a heart rate of 120 beats per minute. The nurse suspects that the patient has a. over-used the albuterol. b. not been using albuterol. c. taken a beta-adrenergic blocker. d. taken a monoamine oxidase (MAO) inhibitor.

Answer: D RATIONALE: A decreased lean body mass in an older adult affects the distribution of drugs (D), which affects the pharmacokinetics of drugs. Decreased gastric pH, delayed gastric emptying, decreased splanchnic blood flow, decreased gastrointestinal absorption surface areas and motility affect (A) in the older adult population. Decreased hepatic blood flow, decreased hepatic mass, and decreased activity of hepatic enzymes affect (B) in older adults. Decreased renal blood flow, decreased glomerular filtration rate, decreased tubular secretion, and decreased number of nephrons affects (C) in an older adult.

An older client with a decreased percentage of lean body mass is likely to receive a prescription that is adjusted based on which pharmacokinetic process? A) Absorption. B) Metabolism. C) Elimination. D) Distribution.

ANS: A This patient is exhibiting signs of Parkinson's disease and should be treated with carbidopa levodopa. The other drugs are used to treat Alzheimer's disease.

An older patient exhibits a shuffling gait, lack of facial expression, and tremors at rest. The nurse will expect the provider to order which medication for this patient? a. Carbidopa-levodopa (Sinemet) b. Donepezil (Aricept) c. Rivastigmine (Exelon) d. Tacrine (Cognex)

ANS: D Nurses should take precautions when handling cytotoxic drugs if inhalation, ingestion, or contact with skin and mucous membranes is possible. When hanging an IV solution, it is possible to splash solution onto the skin, so the nurse should wear a disposable, impermeable gown. If the nurse has to prepare a solution at home, a plastic backed pad should be used as a surface. When there is a risk of aerosol exposure, a National Institute for Occupational Safety and Health-approved respirator is necessary. Surgical masks do not provide adequate respiratory protection. Surgical scrubs are permeable.

An oncology home care nurse is preparing to administer a chemotherapeutic agent to a patient at the patient's home. What precautions will the nurse take while administering the IV chemotherapeutic agent? a. Clear a counter space for preparation of the solution. b. Don a surgical mask while administering the drug. c. Take surgical scrubs to wear during the infusion. d. Wear an impermeable, disposable gown when hanging the drug.

Answer: C

Before administering a dose of an antidysrhythmic drug to an assigned patient, which assessments would be of highest priority? A. Urine output and specific gravity B. Temperature and pulse rate C. Apical pulse and blood pressure D. Peripheral pulses and level of consciousness

Answer: B RATIONALE: The principal adverse effect of penicillins is allergic reaction. Penicillins are contraindicated in patients with a history of severe allergic reactions to penicillins, cephalosporins, or carbapenems. IV patency is important, as is monitoring renal function, because impairment can cause penicillins to reach toxic levels; however, these are not as important as determining allergy status.

Before administering intravenous (IV) penicillin, the nurse should do what? A Flush the IV site with normal saline. B Assess the patient for allergies. C Review the patient's intake and output record. D Determine the latest creatinine clearance result.

Answer: A RATIONALE: Metformin can reach toxic levels in individuals with renal impairment, which is indicated by a rise in the serum creatinine level. The prescriber may need to be notified of the hemoglobin, sodium, and platelet values, but they would not affect the administration of

Before administering metformin (Glucophage), the nurse should notify the prescriber about which laboratory value? A Creatinine (Cr) level of 2.1 mg/dL B Hemoglobin (Hgb) level of 9.5 gm/dL C Sodium (Na) level of 131 mEq/dL D Platelet count of 120,000/mm3 metformin.

Answer: A,C,D

Beta blockers are used to treat which disorders? (Select all that apply.) A. Hypertension B. COPD C. Angina pectoris D. Cardiac dysrhythmias E. Raynaud's disease

Answer: C

Calcium channel blockers reduce myocardial oxygen demand by reducing afterload. How would the nurse explain afterload to the patient? A It is the blood volume within the heart. B It is the pressure within the heart. C It is the pressure against which the heart must pump. D It is the contractility of the heart muscle.

ANS: B, C, D, F, G Decreasing heart rate, stimulating gastric muscles, dilating blood vessels, increasing salivation, and constricting pupils are actions of the cholinergic drugs.

Cholinergic drugs have specific effects on the body. What are the actions of cholinergic medications? (Select all that apply.) a. Dilate pupils b. Decrease heart rate c. Stimulate gastric muscle d. Dilate blood vessels e. Dilate bronchioles f. Increase salivation g. Constrict pupils

Answer: D

Discharge teaching to a client receiving a beta-agonist bronchodilator should emphasize reporting which side effect? a. Hypoglycemia b. Nonproductive cough c. Sedation d. Tachycardia

Answer: A RATIONALE: Nitroglycerin reduces myocardial oxygen consumption which decreases ischemia and reduces chest pain (A). (B and D) would also occur if the angina was relieved, but are not as significant as the client's subjective report of decreased pain. (C) may indicate a reduction in pain, or a potentially serious side effect of the medication.

Following the administration of sublingual nitroglycerin to a client experiencing an acute anginal attack, which assessment finding indicates to the nurse that the desired effect has been achieved? A) Client states chest pain is relieved. B) Client's pulse decreases from 120 to 90. C) Client's systolic blood pressure decreases from 180 to 90. D) Client's SaO2 level increases from 92% to 96%.

ANS: B Steady state levels occur at 3 to 5 half-lives. Wednesday at 0800 is 4 half-lives from the original dose.

If a drug has a half-life of 12 hours and is given twice daily starting at 0800 on a Monday, when will a steady state be achieved? a. 0800 on Tuesday b. 0800 on Wednesday c. 0800 on Thursday d. 0800 on Friday

Answer: D Rationale: A beta-agonist bronchodilator stimulates the beta receptors of the sympathetic nervous system, resulting in tachycardia, bronchodilation, hyperglycemia (if severe), and alertness.

In discharge teaching, the nurse will emphasize to a patient receiving a beta-agonist bronchodilator the importance of reporting which side effect? A Hypoglycemia B Nonproductive cough C Sedation D Tachycardia

Answer: C

In order to prevent the development of tolerance, the nurse instructs the patient to perform which action? A Apply the nitroglycerin patch every other day. B Switch to sublingual nitroglycerin when the patient's systolic blood pressure elevates to >140 mm Hg. C Apply the nitroglycerin patch for 16 hours each and remove for 8 hours at night. D Use the nitroglycerin patch for acute episodes of angina only.

Answer: B RATIONALE: Beta blockers and ACE inhibitors, as well as aldosterone antagonists, are the drug classes recommended for initial therapy of hypertension after an MI. Diuretics and calcium channel blockers are not part of initial therapy for hypertension after an MI.

Initial therapy for hypertension after a myocardial infarction (MI) includes drugs from which classes? A Diuretic and beta blocker B Beta blocker and ACE inhibitor C ACE inhibitor and calcium channel blocker D Diuretic and calcium channel blocker

Answer: B RATIONALE: A disulfiram-like reaction can occur if metronidazole is used with alcohol; therefore, alcohol must be avoided during treatment with this drug. Although the use of alcohol is not promoted in patients who take the other medications, it does not create an adverse reaction.

Instruction by the nurse regarding alcohol abstinence is essential when a patient will be discharged taking which medication? A Tetracycline B Metronidazole C Bismuth subsalicylate D Clarithromycin

Answer: A,C,D RATIONALE: The primary adverse effects of nitroglycerin are headache; orthostatic hypotension, which can lead to dizziness; and reflex tachycardia.

Patients receiving nitroglycerin are at risk for which adverse effects? (Select all that apply.) A Headache B Wheezing C Dizziness D Tachycardia E Bradycardia

Answer: D RATIONALE: AAPMC, which is manifested initially by diarrhea and abdominal cramping, especially may develop with the use of broad-spectrum cephalosporins. Rigidity, ileus, and ascites are unrelated to cephalosporin use.

It is most important for the nurse to assess a patient receiving a cephalosporin for the development of which manifestation of antibiotic-associated pseudomembranous colitis (AAPMC)? A Rigidity B Ileus C Ascites D Diarrhea

Answer: D *Say no to viagra*

It is most important to instruct a patient prescribed nitroglycerin to avoid which substance(s)? A Antacids B Grapefruit juice C Thiazide diuretics D Erectile dysfunction medications

Answer: C Drugs can be metabolized in the gastrointestinal tract; however, the liver is the primary site of metabolism.

Most drugs are metabolized in the A. kidney. B. small intestine. C. liver. D. brain.

Answer: A

Nitrates relieve angina pain by reducing preload. How would the nurse explain the term preload to a patient? A It is the blood volume within the heart. B It is the pressure within the superior vena cava. C It is the pressure against which the heart must pump. D It is the oxygen demand of the heart.

ANS: D Pruritis is a common opioid side effect and can be managed with diphenhydramine. Patients developing anaphylaxis will have urticaria and hypotension, and these patients will need epinephrine and resuscitation. Respiratory depression is a sign of morphine overdose, which will require naloxone.

One hour after receiving intravenous morphine sulfate, a patient reports generalized itching. The nurse assesses the patient and notes clear breath sounds, no rash, respirations of 14 breaths per minute, a heart rate of 68 beats per minute, and a blood pressure of 110/70 mm Hg. Which action will the nurse take? a. Administer naloxone to reverse opiate overdose. b. Have resuscitation equipment available at the bedside. c. Prepare an epinephrine injection in case of an anaphylactic reaction. d. Reassure the patient that this is a common side effect of this drug.

Answer: D

Patient teaching regarding sublingual nitroglycerin should include which statement? A "You can take up to five doses every 3 minutes for chest pain." B "Chew the tablet for the quickest effect." C "Keep the tablets locked in a safe place until you need them." D "Sit or lie down after you take a nitroglycerin tablet to prevent dizziness."

Answer: C Rationale: The patient's symptoms are indicative of an allergic reaction to the medication.

The health care provider has prescribed lansoprazole (Prevacid) for the patient. Within 30 minutes of receiving the first dose of the medication, the patient experiences shortness of breath and develops a rash on his skin. What does the nurse expect that the patient is experiencing? A Unexpected side effect of the medication B Toxic level of the medication C Allergic reaction to the medication D Typical side effect of the medication

Answer: B Rationale: Administering the bronchodilator albuterol (Proventil) first allows the other drugs to reach deeper into the lungs as the bronchioles dilate. Anticholinergics such as ipratropium bromide (Atrovent) also help bronchodilate, but to a lesser extent. Corticosteroids such as beclomethasone (Vanceril) do not dilate and are therefore given last.

The health care provider orders ipratropium bromide (Atrovent), albuterol (Proventil), and beclomethasone (Vanceril) inhalers for a patient. What is the nurse's best action? A Question the order; three inhalers should not be given at one time. B Administer the albuterol, wait 5 minutes, administer ipratropium bromide, then beclomethasone several minutes later. C Administer each inhaler at 30-minute intervals. D Administer beclomethasone, wait 2 minutes, administer ipratropium bromide, then albuterol several minutes later.

Answer: C Rationale: Lansoprazole (Prevacid) is a proton pump inhibitor that is effective in suppressing gastric acid secretions. An absence of esophageal pain would be an indication that the patient does not have reflux esophagitis.

The health care provider prescribes lansoprazole (Prevacid) for a patient. Which assessment indicates to the nurse that the medication has had a therapeutic effect? A The patient has no diarrhea. B The patient has no gastric pain. C The patient has no esophageal pain. D The patient is able to eat.

Answer: B RATIONALE: Hypokalemia (decreased serum potassium) will precipitate digitalis toxicity in persons receiving digoxin (B). (A and C) will not affect the administration of digoxin. (D) should be monitored if he/she is a diabetic and is perhaps receiving insulin.

The healthcare provider prescribes digitalis (Digoxin) for a client diagnosed with congestive heart failure. Which intervention should the nurse implement prior to administering the digoxin? A) Observe respiratory rate and depth. B) Assess the serum potassium level. C) Obtain the client's blood pressure. D) Monitor the serum glucose level.

ANS: A Excessive use of steroid inhalers reduces local immune function and increases the client's risk for oral-pharyngeal infections, including candidiasis. There is no evidence that the common cold, long-term use of the same oxygen tank, or using a toothbrush for a longer period will increase the client's chances of developing candidiasis.

The home care nurse observes oral candidiasis in the client with severe, chronic, airflow limitation. What information will the nurse obtain from this client? a. "How often are you using your steroid inhaler?" b. "Have you had a cold or other viral infection lately?" c. "When was the last time the oxygen tank was changed?" d. "Have you changed your toothbrush recently?"

Answer: A,C,E RATIONALE: HIV is a retrovirus that replicates itself in a host cell by transcribing RNA into DNA. The enzyme used for this process is viral RNA-dependent DNA polymerase, commonly known as reverse transcriptase. HIV DNA becomes integrated into the host's DNA under the influence of a viral enzyme called integrase. Final maturation of HIV depends on protease, which breaks large HIV polyproteins into smaller, functional forms. Dihydrofolate reductase and carbonic anhydrase are other enzymes used to catalyze chemical cellular reactions, but they are not participants in HIV replication.

The human immunodeficiency virus (HIV) uses which enzymes to replicate and infect a host cell? (Select all that apply.) A Integrase B Dihydrofolate reductase C Reverse transcriptase D Carbonic anhydrase E Protease

Answer: A RATIONALE: Heparin-induced thrombocytopenia (HIT) is a potential immune-mediated adverse effect of heparin infusions that can prove fatal. HIT is suspected when the platelet counts fall significantly. A platelet count below 100,000/mm3 would warrant discontinuation of the heparin.

The laboratory calls the nursing unit to report a drop in the platelet count to 90,000/mm3 for a patient receiving heparin for the treatment of postoperative deep vein thrombosis. Which action by the nurse is the most appropriate? A Call the healthcare provider to discuss the reduction or withdrawal of heparin. B Call the healthcare provider to discuss increasing the heparin dose to achieve a therapeutic level. C Obtain vitamin K and prepare to administer it by intramuscular (IM) injection. D Observe the patient and monitor the activated partial thromboplastin time (aPTT) as indicated.

ANS: A The patient should show improvement in breathing and oxygenation. Urine output should increase. Blood pressure and heart rate will decrease.

The nurse administers a dose of digoxin (Lanoxin) to a patient who has heart failure and returns to the room later to reassess the patient. Which finding indicates that the medication is effective? a. Decreased dyspnea b. Decreased urine output c. Increased blood pressure d. Increased heart rate

ANS: A An agonist medication is one that stimulates a certain type of cell to produce a response.

The nurse administers albuterol to a patient who has asthma. The albuterol acts by stimulating beta2-adrenergic receptors to cause bronchodilation. The nurse understands that albuterol is a beta-adrenergic a. agonist. b. antagonist. c. inhibitor. d. depressant.

ANS: B Epinephrine can cause renal vasoconstriction and thereby reduce renal perfusion and decrease urinary output. Epinephrine causes tachycardia and elevates blood pressure. Nausea and vomiting are not expected to occur.

The nurse administers subcutaneous epinephrine to a patient who is experiencing an anaphylactic reaction. The nurse should expect to monitor the patient for which symptom? a. Bradycardia b. Decreased urine output c. Hypotension d. Nausea and vomiting

ANS: C Methylxanthines, including aminophylline, stimulate the sympathetic nervous system, the cardiovascular system, and the kidneys. The development of a seizure would indicate central nervous system irritability. A bradycardiac heart rate would not be a side effect of the medication. Urinary output of 30 mL per hour is a normal assessment finding. A blood pressure of 100/60 is also considered within normal range.

The nurse assesses a client receiving aminophylline. What assessment finding indicates a dangerous side effect of this medication? a. Heart rate of 50 beats/min b. Urinary output of 30 mL in 1 hour c. Development of seizures d. Blood pressure of 100/60 mm Hg

ANS: C Naloxone is a narcotic antagonist, meaning that it reverses the effects of morphine by blocking morphine receptor sites.

The nurse assesses a patient who is receiving morphine sulfate intravenously using a patient-controlled analgesia pump. The nurse notes somnolence and respiratory depression, which are signs of morphine toxicity. The nurse will prepare to administer naloxone (Narcan) because it a. has synergistic effects with morphine. b. is a narcotic agonist. c. is a narcotic antagonist. d. potentiates narcotic effects.

ANS: B Older patients often minimize pain when asked, so the nurse should evaluate nonverbal cues to pain such as elevated heart rate and blood pressure and the fact that the patient is lying very still. The nurse should increase the dose the next time the pain medication is given.

The nurse assesses an older patient 60 minutes after administering 4 mg of intravenous morphine sulfate (MS) for postoperative pain. The patient's analgesia order is for 2 to 5 mg of MS IV every 2 hours. The nurse notes that the patient is lying very still. The patient's heart rate is 96 beats per minute, respiratory rate is 14 breaths per minute, and blood pressure is 140/90 mm Hg. When asked to rate the level of pain, the patient replies "just a 5." The nurse will perform which action? a. Give 3 mg of MS at the next dose. b. Give 5 mg of MS at the next dose. c. Request an order for an oral opioid to give now. d. Request an order for acetaminophen to give now.

ANS: B Furosemide is a loop diuretic and is given when the patient's condition warrants immediate removal of body fluid, as in heart failure. Digoxin improves cardiac function but does not remove fluid quickly. The other diuretics may be used when immediate fluid removal is not necessary.

The nurse is caring for a patient who develops marked edema and a low urine output as a result of heart failure. Which medication will the nurse expect the provider to order for this patient? a. Digoxin (Lanoxin) b. Furosemide (Lasix) c. Hydrochlorothiazide (HydroDIURIL) d. Spironolactone (Aldactone)

ANS: B When vancomycin is infused too rapidly, "red man" syndrome may occur; the rate should be 10 mg/min to prevent this. This is a toxic reaction, not an allergic one, so epinephrine is not indicated. Stevens Johnson syndrome is characterized by a rash and fever. Red man syndrome is not related to renal function.

The nurse assumes care for a patient who is currently receiving a dose of intravenous vancomycin (Vancocin) infusing at 20 mg/min. The nurse notes red blotches on the patient's face, neck, and chest and assesses a blood pressure of 80/55 mm Hg. Which action will the nurse take? a. Request an order for IV epinephrine to treat anaphylactic shock. b. Slow the infusion to 10 mg/min and observe the patient closely. c. Stop the infusion and obtain an order for a BUN and serum creatinine. d. Suspect Stevens-Johnson syndrome and notify the provider immediately.

ANS: B The patient's respiratory rate of 10 breaths per minute is lower than normal and is a sign of respiratory depression, which is a common adverse effect of opioid analgesics. The other effects may occur with opioids but are also not expected this soon after abdominal surgery.

The nurse assumes care of a patient in the post-anesthesia care unit (PACU). The patient had abdominal surgery and is receiving intravenous morphine sulfate for pain. The patient is asleep and has not voided since prior to surgery. The nurse assesses a respiratory rate of 10 breaths per minute and notes hypoactive bowel sounds. The nurse will contact the surgeon to report which condition? a. Paralytic ileus b. Respiratory depression c. Somnolence d. Urinary retention

Answer: D RATIONALE: The nurse's best action is to hold the Lasix and notify the physician. Loop diuretics, such as furosemide, can cause significant potassium loss. The normal potassium level is 3.5 to 5 mEq/L. The remaining electrolyte levels are normal. Administering the Lasix could result in a critically low potassium level. Effects of low potassium include cardiac dysrhythmias. Placing a patient on a cardiac monitor requires a physician's order and would warrant further assessment first, such as taking vital signs and asking the patient whether he or she is having any cardiac-related symptoms. Collecting a 24-hour urine specimen is not appropriate in this case.

The nurse caring for a patient taking furosemide (Lasix) is reviewing the patient's most recent laboratory results, which are: sodium, 136 mEq/L; potassium, 3.2 mEq/L; chloride, 100mEq/L; blood urea nitrogen, 15 mg/dL. What is the nurse's best action? A Administer Lasix as ordered B Place the patient on a cardiac monitor C Begin a 24-hour urine collection D Hold the Lasix and notify the physician

Answer: A

The nurse caring for a patient who is receiving beta1 agonist drug therapy needs to be aware that these drugs cause which effect? a) increased cardiac contractility b) decreased heart rate c) bronchoconstriction d) increased GI tract motility

ANS: A, B, F Adrenergic agents stimulate the sympathetic nervous system, evoking the "fight or flight" response. This response increases those functions needed to respond to stress (increased heart rate to perfuse muscles, bronchodilation to increase oxygen exchange). Adrenergic drugs shunt blood away from the reproductive tract and gastrointestinal organs as these functions are not needed during a fight or flight response.

The nurse caring for a patient who is taking an adrenergic agent will expect which side effects? (Select all that apply.) a. Dilated pupils b. Increased heart rate c. Increase gastrointestinal motility d. Vasodilation e. Bronchospasm f. Relaxed uterine muscles

ANS: B Erythromycin is the drug of choice when penicillin is not an option. Giving smaller doses of penicillin does not prevent hypersensitivity reactions. Benadryl is useful when a hypersensitivity reaction has occurred. A small percentage of patients allergic to penicillins may be hypersensitive to cephalosporins.

The nurse caring for a patient who will receive penicillin to treat an infection asks the patient about previous drug reactions. The patient reports having had a rash when taking amoxicillin (Amoxil). The nurse will contact the provider to a. discuss giving a smaller dose of penicillin. b. discuss using erythromycin (E-mycin) instead of penicillin. c. request an order for diphenhydramine (Benadryl). d. suggest that the patient receive cefuroxime (Ceftin).

ANS: B Liver diseases such as cirrhosis and hepatitis alter drug metabolism by inhibiting the drug metabolizing enzymes in the liver. When the drug metabolism rate is decreased, excess drug accumulation can occur and lead to toxicity.

The nurse gives a medication to a patient with a history of liver disease. The nurse will monitor this patient for a. decreased drug effects. b. increased drug effects. c. decreased therapeutic range. d. increased therapeutic range.

Answer: A,B,E RATIONALE: A patient who is allergic to penicillin has a 1% chance of also being allergic to cephalosporins. Patients who are allergic to penicillin are safely able to take vancomycin, erythromycin, and clindamycin. The other three statements are true.

The nurse identifies which statements about penicillins as true? (Select all that apply.) A Penicillins are the safest antibiotics available. B The principal adverse effect of penicillins is allergic reaction. C A patient who is allergic to penicillin always has a cross-allergy to cephalosporins. D A patient who is allergic to penicillin is also allergic to vancomycin, erythromycin, and clindamycin. E Penicillins are normally eliminated rapidly by the kidneys but can accumulate to harmful levels if renal function is severely impaired.

Answer: A RATIONALE: Epinephrine can cause a number of adverse effects, including hypertensive crisis, dysrhythmias, angina, necrosis after extravasation, and hyperglycemia. Monitoring of the heart rhythm is essential to assess the patient for dysrhythmias.

The nurse in the cardiac care unit is caring for a patient receiving epinephrine. Which assessment criterion takes priority in the monitoring for adverse effects of this drug? A Cardiac rhythm B Blood urea nitrogen C Central nervous system (CNS) tremor D Lung sounds

ANS: D Two drugs that are highly protein bound will compete for protein-binding sites, leaving more free drug in circulation and an increased risk of adverse effects as well as increased bioavailability, increased drug effects, and increased drug interactions.

The nurse is administering two drugs to a patient and learns that both drugs are highly protein-bound. The nurse may expect a. decreased bioavailability of both drugs. b. decreased drug effects. c. decreased drug interactions. d. increased risk of adverse effects.

Answer: C RATIONALE: Epinephrine (C) is an adrenergic agent that stimulate beta receptors to increase cardiac automaticity in cardiac arrest and relax bronchospasms in anaphylaxis. Dopamine (A) is a vasopressor used to treat clients with shock. Ephedrine (B) causes peripheral vasoconstriction and is used in the treatment of nasal congestion. Diphenhydramine (D) is an antihistamine decongestant used in the treatment of mild allergic reactions and motion sickness.

The nurse is assessing a client who is experiencing anaphylaxis from an insect sting. Which prescription should the nurse prepare to administer this client? A) Dopamine. B) Ephedrine. C) Epinephrine. D) Diphenhydramine.

Answer: B RATIONALE: The nurse's priority is to monitor for hypersensitivity reactions. The most serious response to sulfonamide therapy is Stevens-Johnson syndrome, which manifests as symptoms of the skin and mucous membranes, lesions, fever, and malaise. In rare cases, hematologic effects occur, requiring periodic blood studies.

The nurse is assessing a patient who is receiving a sulfonamide for treatment of a urinary tract infection. To monitor the patient for the most severe response to sulfonamide therapy, the nurse will assess for what? A Diarrhea B Skin rash and lesions C Hypertension D Bleeding

ANS: A An increased heart rate followed by a decreased systolic pressure can indicate a fluid volume deficit caused by internal or external bleeding. The nurse should examine the patient's mouth, nose, and skin for bleeding. These vital signs do not indicate a pulmonary problem. Skin turgor and mucous membranes as well as urine output and level of consciousness may be assessed to determine the level of fluid deficit, but finding the source of blood loss is more important. Signs of gastrointestinal bleeding should also be assessed.

The nurse is assessing a patient who takes warfarin (Coumadin). The nurse notes a heart rate of 92 beats per minute and a blood pressure of 88/78 mm Hg. To evaluate the reason for these vital signs, the nurse will assess the patient's a. gums, nose, and skin. b. lung sounds and respiratory effort. c. skin turgor and oral mucous membranes. d. urine output and level of consciousness.

ANS: D Tetracyclines should not be given to children younger than 8 years of age because they irreversibly discolor the permanent teeth.

The nurse is caring for a 7-year-old patient who will receive oral antibiotics. Which antibiotic order will the nurse question for this patient? a. Azithromycin (Zithromax) b. Clarithromycin (Biaxin) c. Clindamycin (Cleocin) d. Tetracycline (Sumycin)

ANS: C Aminoglycosides can cause ototoxicity. Any changes in hearing should be reported to the provider so that serum drug levels can be monitored. The dose is correct for this patient's weight (5 mg/kg/day in 4 divided doses). A hearing test is not indicated unless changes in hearing persist.

The nurse is caring for a 70-kg patient who is receiving gentamicin (Garamycin) 85 mg 4 times daily. The patient reports experiencing ringing in the ears. The nurse will contact the provider to discuss a. decreasing the dose to 50 mg QID. b. giving the dose 3 times daily. c. obtaining a serum drug level. d. ordering a hearing test.

Answer: C RATIONALE: The primary action of nitroglycerin is vasodilation, which leads to decreased venous return, decreased ventricular filling, and decreased preload, thus reducing oxygen demand on the heart.

The nurse is caring for a patient receiving nitrates for relief of angina. What pharmacodynamic action is responsible for the relief of anginal pain with nitrates? A Vasoconstriction leads to improved cardiac output. B Decreased force of contraction leads to decreased oxygen demand. C Vasodilation leads to decreased preload, which decreases oxygen demand. D Influx of calcium ions leads to relaxation of vascular smooth muscle.

ANS: A Albuterol is used to treat bronchospasm during symptom flares. Oral prednisone is given for acute flares but not generally on a daily basis until symptoms are chronic and severe because of the risk of adrenal suppression. Prophylactic antibiotics are not given regularly because of the risk of antibiotic resistance. Symptoms of emphysema are not reversible.

The nurse is caring for a patient recently diagnosed with mild emphysema and provides teaching about the disease and medications for treatment. Which statement by the patient indicates understanding of the medication regimen? a. "I should use albuterol when my symptoms worsen." b. "I will need to take oral prednisone on a daily basis." c. "My provider will prescribe prophylactic antibiotics." d. "My symptoms are reversible with proper medications."

ANS: A Hydantoins commonly cause gingival hyperplasia, which causes overgrowth of reddened gum tissue that bleeds easily. It is not a sign of a drug interaction or a symptom of hepatotoxicity. It does not indicate a lack of self care.

The nurse is caring for a patient who has a seizure disorder. The nurse notes that the patient has reddened gums that bleed when oral care is given. The nurse recognizes this finding as a. an adverse effect of the phenytoin. b. a drug interaction with aspirin. c. a symptom of hepatotoxicity. d. a sign of poor self-care.

ANS: A Drugs that act on beta2 receptors activate glyconeogenesis in the liver causing increased blood glucose. Selective beta2 drugs act on beta2 receptors only and not on beta1 receptors, so they do not cause increased blood pressure or increased heart rate. Adrenergic agonists cause decreased GI motility.

The nurse is caring for a patient who has asthma and administers a selective beta2-adrenergic agonist to treat bronchospasm. The nurse will expect this drug to also cause which side effect? a. Increased blood glucose b. Increased blood pressure c. Increased heart rate d. Increased gastrointestinal (GI) motility

ANS: B This medication is effective for both pain and swelling. After 4 weeks, there should be some decrease in swelling, so the nurse should report that this medication is ineffective. There is no indication that this patient is seeking an opioid analgesic. The drug should be effective within several weeks. NSAIDs are given for pain and swelling.

The nurse is caring for a patient who has been taking an NSAID for 4 weeks for osteoarthritis. The patient reports decreased pain, but the nurse notes continued swelling of the affected joints. The nurse will perform which action? a. Assess the patient for drug-seeking behaviors. b. Notify the provider that the drug is not effective. c. Reassure the patient that swelling will decrease eventually. d. Remind the patient that this drug is given for pain only.

ANS: C Aspirin is a weak acid and is more readily excreted in alkaline urine. Sodium bicarbonate alkalizes the urine. It does not act as an antidote to aspirin, decrease the half life, or neutralize its pH.

The nurse is caring for a patient who has taken an overdose of aspirin several hours prior. The provider orders sodium bicarbonate to be given. The nurse understands that this drug is given for which purpose? a. To counter the toxic effects of the aspirin b. To decrease the half-life of the aspirin c. To increase the excretion of the aspirin d. To neutralize the acid of the aspirin

ANS: C Persons exposed to the blood of HIV infected patients should receive 4 weeks of antiretroviral therapy.

The nurse is caring for a patient who is HIV-positive and has been receiving antiretroviral therapy for several months. The nurse experiences a needlestick injury resulting in exposure to the patient's blood. The nurse asks the Occupational Health nurse if treatment is necessary. How will the Occupational Health nurse respond? a. "No treatment is necessary since the patient is receiving antiretroviral therapy." b. "We will treat you if the patient's VL is > 20 copies/mL." c. "You will require 4 weeks of antiretroviral therapy." d. "You will undergo HIV testing and will be treated if you are positive."

ANS: A Members of some cultures may use traditional healers, and this should be accommodated whenever possible. Showing respect for this patient's culture will help to establish trust and thus greater cooperation. It is important for the nurse not to make generalizations within and among cultural groups, so asking a family member to describe what this particular patient needs is the better choice. Finding a hospital staff member who is Native American assumes that all Native Americans have the same practices. Deferring to a Social Worker is not necessary. Enlisting a family member to explain the need for the medications is just another way of imposing treatments on this person without respecting their cultural needs.

The nurse is caring for a patient who is a member of the local Native American community. The patient is refusing medications and treatments in spite of repeated attempts to explain the importance of these interventions. Which is an appropriate nursing action? a. Ask a family member about traditional healing practices that might be better accepted. b. Enlist the help of a family member to explain the need for the medications and treatments. c. Find a hospital staff member who is Native American to help provide teaching for this patient. d. Suggest a Social Work consult to the patient's provider.

ANS: A INH is the drug of choice for prophylactic treatment of patients who have had close contact with a patient who has tuberculosis.

The nurse is caring for a patient who is diagnosed with tuberculosis. The patient tells the nurse that the provider plans to order a prophylactic antitubercular drug for family members and asks which drug will be ordered. The nurse will expect the provider to order which drug? a. Isoniazid (INH) b. Pyrazinamide c. Rifampin (Rifadin) d. Streptomycin

ANS: A High doses of tetracyclines can lead to nephrotoxicity, especially when given along with other nephrotoxic drugs. Renal function tests should be performed to monitor for nephrotoxicity.

The nurse is caring for a patient who is receiving a high dose of tetracycline (Sumycin). Which laboratory values will the nurse expect to monitor while caring for this patient? a. Blood urea nitrogen (BUN) and creatinine levels b. Complete blood counts c. Electrolytes d. Liver enzyme levels

ANS: D Medications with a narrow therapeutic index have a limited range between the therapeutic dose and a toxic dose. It is important to monitor these medications closely by evaluating regular serum peak and trough levels.

The nurse is caring for a patient who is receiving an intravenous antibiotic. The nurse notes that the provider has ordered serum drug peak and trough levels. The nurse understands that these tests are necessary for which type of drugs? a. Drugs with a broad spectrum b. Drugs with a narrow spectrum c. Drugs with a broad therapeutic index d. Drugs with a narrow therapeutic index

ANS: A Low peak levels may indicate that the medication is below the therapeutic level. They do not indicate altered risk for superinfection, a decrease in adverse effects, or a slowed onset of action.

The nurse is caring for a patient who is receiving an intravenous antibiotic. The patient has a serum drug trough of 1.5 mcg/mL. The normal trough for this drug is 1.7 mcg/mL to 2.2 mcg/mL. What will the nurse expect the patient to experience? a. Inadequate drug effects b. Increased risk for superinfection c. Minimal adverse effects d. Slowed onset of action

ANS: B Theophylline has a narrow therapeutic index and a risk for severe symptoms with toxic levels. When patients report symptoms of theophylline adverse effects, a serum drug level should be obtained. Giving an oral theophylline would only compound the problem if the patient has a toxic drug level. Analgesics may be used, but only after toxicity is ruled out. Adding a different methylxanthine will compound the symptoms and will likely result in drug interaction or unwanted synergism.

The nurse is caring for a patient who is receiving intravenous theophylline. The patient complains of headache and nausea. The nurse will contact the provider to a. change the medication to an oral theophylline. b. obtain an order for a serum theophylline level. c. request an order for an analgesic medication. d. suggest an alternative methylxanthine medication.

ANS: A The INR is the test used most frequently to report prothrombin time results in patients taking warfarin. Warfarin is not an antiplatelet drug, so platelet levels are not indicated. PTT and aPTT are used to monitor heparin therapy. Vitamin K is an antidote for warfarin; levels are not routinely checked.

The nurse is caring for a patient who is receiving warfarin (Coumadin) and notes bruising and petechiae on the patient's extremities. The nurse will request an order for which laboratory test? a. International normalized ratio (INR) b. Platelet level c. Partial thromboplastin time (PTT) and activated partial thromboplastin time (aPTT) d. Vitamin K level

ANS: C The antidysrhythmics phenytoin and lidocaine are effective in treating digoxin induced ventricular dysrhythmias. Digoxin immune Fab is used to treat severe digitalis toxicity, characterized by bradycardia, nausea, and vomiting. Unless a potassium deficit is present, giving potassium could worsen the dysrhythmia.

The nurse is caring for a patient who is taking digoxin to treat heart failure. The patient's electrocardiogram shows a ventricular dysrhythmia. The nurse will notify the provider and will anticipate an order for which medication? a. Digoxin immune Fab (Digibind) b. Furosemide (Lasix) c. Phenytoin (Dilantin) d. Potassium

ANS: C TMP/SMX can result in hyperkalemia when taken with an ACE inhibitor.

The nurse is caring for a patient who is taking trimethoprim-sulfamethoxazole (TMP-SMX). The nurse learns that the patient takes an angiotension-converting enzyme (ACE) inhibitor. To monitor for drug interactions, the nurse will request an order for which laboratory test(s)? a. A complete blood count b. BUN and creatinine c. Electrolytes d. Glucose

ANS: C Thiazides block uric acid secretion and elevated levels can contribute to gout. Patients with a history of gout should take thiazide diuretics with caution; they may need behavioral and/or pharmacologic changes to their gout treatment.

The nurse is caring for a patient who is to begin receiving a thiazide diuretic to treat heart failure. When performing a health history on this patient, the nurse will be concerned about a history of which condition? a. Asthma b. Glaucoma c. Gout d. Hypertension

ANS: B Cimetidine is an enzyme inhibitor that decreases the metabolism of drugs such as theophylline. If the cimetidine is discontinued, the theophylline dose should be decreased to avoid toxicity. The nurse should observe the patient for increased theophylline effects.

The nurse is caring for a patient who receives theophylline, which has a narrow therapeutic index. The patient has been receiving cimetidine but will stop taking that drug in 2 days. In 2 days, the nurse will observe the patient closely for a. decreased effectiveness of theophylline. b. increased effectiveness of theophylline. c. decreased toxicity of theophylline. d. prolonged effectiveness of theophylline.

ANS: C Diuretics such as furosemide promote water and sodium excretion from the renal tubules, especially sodium and potassium. Hypokalemia can result, and this will enhance the action of digoxin, and digitalis toxicity can occur.

The nurse is caring for a patient who takes digoxin to treat heart failure. The provider orders furosemide to treat edema. The nurse will monitor the patient for digitalis toxicity because of a. adverse drug reactions caused by giving these drugs in combination. b. altered hepatic blood flow caused by the furosemide. c. changes in reabsorption of water and electrolytes in the kidneys. d. additive effects of these two drugs given together.

Answer: D RATIONALE: An INR in the range of 2 to 3 is considered the level for warfarin therapy. For a level of 1.2, the nurse should contact the healthcare provider to discuss an order for an increased dose.

The nurse is caring for a patient who takes warfarin (Coumadin) for prevention of deep vein thrombosis. The patient has an international normalized ratio (INR) of 1.2. Which action by the nurse is most appropriate? A Prepare to administer protamine sulfate. B Continue with the current prescription. C Prepare to administer vitamin K. D Call the healthcare provider to increase the dose.

ANS: A Use of opioid analgesics is contraindicated for patients with head injuries because of the risk of increased intracranial pressure. If opioids are necessary because of severe pain, they must be given in reduced doses. This patient is experiencing mild pain, so acetaminophen is an appropriate analgesic.

The nurse is caring for a patient who was admitted with a fractured leg and for observation of a closed head injury after a motor vehicle accident. The patient reports having pain at a level of 3 on a 1 to 10 pain scale. The nurse will expect the provider to order which analgesic medication for this patient? a. Acetaminophen (Tylenol) PO b. Hydromorphone HCl (Dilaudid) IM c. Morphine sulfate PCA d. Transdermal fentanyl (Duragesic)

ANS: B Patients who have renal insufficiency will not require a decrease in dose with fosinopril, as they would with other angiotensin converting enzyme (ACE) inhibitors. If captopril is given, it should be given in a reduced dose. Increased IV fluids are not indicated.

The nurse is caring for a patient who will begin taking captopril (Capoten) for hypertension. The nurse reviews the patient's laboratory test results and notes increased BUN and creatinine. Which action will the nurse take? a. Administer the captopril and monitor vital signs. b. Contact the provider to discuss changing to fosinopril (Monopril). c. Obtain an order for intravenous fluids to improve urine output. d. Request an order to add hydrochlorothiazide (HydroDIURIL).

ANS: C Doxycycline is a lipid soluble tetracycline and is better absorbed when taken with milk products and food. It should not be taken on an empty stomach. Antacids impair absorption of tetracyclines. Small sips of water are not necessarily indicated.

The nurse is caring for a patient who will begin taking doxycycline to treat an infection. The nurse should plan to give this medication a. 1 hour before or 2 hours after a meal. b. with an antacid to minimize GI irritation. c. with food to improve absorption. d. with small sips of water.

ANS: D Proton pump inhibitors can enhance the effects of digoxin, so patients should be monitored for digoxin toxicity. Changing the dose of either medication is not indicated prior to obtaining lab results that are positive for digoxin toxicity.

The nurse is caring for a patient who will begin taking omeprazole (Prevacid) 20 mg per day for 4 to 8 weeks to treat gastroesophageal reflux disease esophagitis. The nurse learns that the patient takes digoxin. The nurse will contact the provider for orders to a. decrease the dose of omeprazole. b. increase the dose of digoxin. c. increase the omeprazole to 60 mg per day. d. monitor for digoxin toxicity.

ANS: B Tobacco smoking increases the metabolism of theophylline, so the dose should be increased. Decreasing the dose will lead to subtherapeutic effects.

The nurse is caring for a patient who will begin taking theophylline at home. During the assessment, the nurse learns that the patient smokes. The nurse reports this to the provider and will expect the provider to a. decrease the dose of theophylline. b. increase the dose of theophylline. c. keep the theophylline dose as ordered. d. discontinue the theophylline.

ANS: B Tachycardia and hypotension indicate bleeding. The nurse should check the patient's surgical dressing to assess for bleeding. These signs do not indicate anaphylaxis. They may indicate dehydration, but bleeding is the more likely cause of fluid volume deficit. The nurse should continue to evaluate vital signs, but it is imperative that the nurse assess the patient to explore the potential cause.

The nurse is caring for a postoperative patient who is receiving alteplase tPA (Activase) after developing a blood clot. The nurse notes a heart rate of 110 beats per minute and a blood pressure of 90/60 mm Hg. The nurse will perform which action? a. Ask the patient about itching or shortness of breath. b. Assess the surgical dressing for bleeding. c. Evaluate the patient's urine output and fluid intake. d. Recheck the patient's vital signs in 15 minutes.

ANS: C Ketorolac is the first injectable NSAID and has shown analgesic efficacy equal or superior to that of opioid analgesics. The other NSAIDs listed are not used for postoperative pain.

The nurse is caring for a postpartum woman who is refusing opioid analgesics but is rating her pain as a 7 or 8 on a 10-point pain scale. The nurse will contact the provider to request an order for which analgesic medication? a. Diclofenac sodium (Voltaren) b. Ketoprofen (Orudis) c. Ketorolac (Toradol) d. Naproxyn (Naprosyn)

ANS: B Most of digoxin is eliminated by the kidneys, so a decline in kidney function can cause digoxin accumulation, which can cause bradycardia. Digoxin should not be given to any patient with a pulse less than 60 beats per minute.

The nurse is caring for an 82-year-old patient who takes digoxin to treat chronic atrial fibrillation. When caring for this patient, to monitor for drug side effects, what will the nurse will carefully assess? a. Blood pressure b. Heart rate c. Oxygen saturation d. Respiratory rate

ANS: A African Americans do not respond well to beta blockers and ACE inhibitors, but do tend to respond to diuretics and calcium channel blockers. Changing to an ACE inhibitor or altering the beta blocker dose are not indicated. Hypertension in African American patients can be controlled by combining beta blockers with diuretics.

The nurse is caring for an African-American patient who has been taking a beta blocker to treat hypertension for several weeks with only slight improvement in blood pressure. The nurse will contact the provider to discuss a. adding a diuretic medication. b. changing to an ACE inhibitor. c. decreasing the beta blocker dose. d. doubling the beta blocker dose.

ANS: A Certain classifications of medications have different effects in individuals whose genetic markers are predominantly characteristic of a certain biologic group. African American patients will tend to have a decreased therapeutic effect from warfarin (Coumadin).

The nurse is caring for an African-American patient who is taking warfarin (Coumadin) to prevent blood clots. The nurse will monitor this patient carefully for which effect? a. Decreased therapeutic effects b. Heightened risk for hemorrhage c. Increased risk of hypersensitivity d. Potential risk of paradoxical effects

ANS: D Older patients who take doses of hydrochlorothiazide between 25 to 50 mg/day have increased risk of electrolyte imbalances, so potassium should be monitored closely.

The nurse is caring for an older patient who is taking 25 mg per day of hydrochlorothiazide. The nurse will closely monitor which lab value in this patient? a. Coagulation studies b. White blood count c. Liver function tests d. Serum potassium

Answer: D Rationale: The side effects of epinephrine include tachycardia, dysrhythmias, and palpitations. This patient should not receive epinephrine.

The nurse is caring for multiple patients on the pulmonary unit. The nurse would question the administration of prescribed epinephrine to which patient? A The patient with a history of emphysema B The patient with a history of type 2 diabetes C The patient who is 16 years old D The patient with atrial fibrillation with a rate of 100

Answer: A RATIONALE: Interactions are especially important with drugs that have a low therapeutic index, because an interaction that produces a modest increase in drug levels can cause toxicity.

The nurse is concerned with minimizing adverse drug-drug interactions for the patient. Which drug characteristic could result in the most serious consequences from a drug-drug interaction? A Low therapeutic index B High biologic half-life C Low potency D First-pass effect

Answer: B RATIONALE: Alcohol can intensify the hypotensive effects of nitrates, so the patient should avoid alcohol. Patients develop tolerance to nitrates rather quickly. Patients receiving transdermal nitrates are recommended to have 10 to 12 hours of patch-free time each evening. Sildenafil (Viagra) and other drugs for erectile dysfunction also can cause significant hypotension with nitroglycerin and are contraindicated. Nitroglycerin causes orthostatic hypotension; therefore, patients should change positions slowly.

The nurse is conducting discharge teaching for a patient with a new prescription for transdermal nitroglycerin. Which statement by the patient indicates a need for further teaching? A "I will remove my patch at bedtime each evening." B "I will limit my alcohol to one drink per day." C "I will not use Viagra as long as I am on nitroglycerin." D "I will move slowly when changing positions."

ANS: A Patients should drink several quarts of water daily while taking sulfonamides to prevent crystalluria. Patients should not take antacids with sulfonamides. Patients should not go out into the sun. Sore throat should be reported.

The nurse is counseling a patient who will begin taking a sulfonamide drug to treat a urinary tract infection. What information will the nurse include in teaching? a. "Drink several quarts of water daily." b. "If stomach upset occurs, take an antacid." c. "Limit sun exposure to no more than 1 hour each day." d. "Sore throat is a common, harmless side effect."

ANS: D Nonspecific drugs can act on one type of receptor but in different body tissues, or a variety of receptor types, or act on hormones to produce effects. Nonspecific drugs do not require higher doses.

The nurse is explaining to the patient why a nonspecific drug has so many side effects. Which statement by the patient indicates a need for further teaching? a. "Nonspecific drugs can affect specific receptor types in different body tissues." b. "Nonspecific drugs can affect a variety of receptor types in similar body tissues." c. "Nonspecific drugs can affect hormone secretion as well as cellular functions." d. "Nonspecific drugs require higher doses than specific drugs to be effective."

ANS: A Aspirin causes tinnitus at low toxicity levels. The nurse should question the patient about this medication. The other medications do not have this side effect.

The nurse is performing a health history on a patient who has arthritis. The patient reports tinnitus. Suspecting a drug adverse effect, the nurse will ask the patient about which medication? a. Aspirin (Bayer) b. Acetaminophen (Tylenol) c. Anakinra (Kineret) d. Prednisone (Deltasone)

ANS: C The beta2 agonists can increase serum glucose levels and montelukast can elevate liver enzymes, so these should be monitored in patients taking these medications.

The nurse is performing a medication history on a patient who reports long-term use of montelukast (Singulair) and an albuterol metered-dose inhaler (Proventil). The nurse will contact the provider to discuss an order for which laboratory tests? a. Cardiac enzymes and serum calcium b. Electrolytes and a complete blood count c. Liver function tests and serum glucose d. Urinalysis and serum magnesium

ANS: C Patients of Asian descent might speak in soft tones and avoid direct eye contact while being comfortable with long silences. It is not correct to ask family members to evaluate pain. Without assessment of non verbal cues, the nurse cannot determine whether the pain is minimal or severe.

The nurse is performing a pain assessment on a patient of Asian descent. The patient does not describe the pain when asked to do so and looks away from the nurse. What will the nurse do next? a. Ask the patient's family member to evaluate the patient's pain. b. Conclude that the patient's pain is minimal. c. Evaluate the patient's non-verbal pain cues. d. Suspect that the patient is experiencing severe pain.

ANS: D Sulfonamides can increase the anticoagulant effects of warfarin. The nurse should request INR levels. An increased dose of warfarin would likely lead to toxicity and to undesirable anticoagulation.

The nurse is preparing to give a dose of trimethoprim-sulfamethoxazole (TMP-SMX) and learns that the patient takes warfarin (Coumadin). The nurse will request an order for a. a decreased dose of TMP-SMX. b. a different antibiotic. c. an increased dose of warfarin. d. coagulation studies.

ANS: C Large doses or overdoses of acetaminophen can be toxic to hepatic cells, so when large doses are administered over a long period, liver function should be assessed. Daily headaches are not typical of migraine headaches, so SSRA medication is not indicated. Hydrocodone with acetaminophen is not indicated without further evaluation of headaches. Serum glucose is not indicated.

The nurse is performing an admission assessment on an adolescent who reports taking extra-strength acetaminophen (Tylenol) regularly to treat daily headaches. The nurse will notify the patient's provider and discuss an order for a. a selective serotonin receptor agonist (SSRA). b. hydrocodone with acetaminophen for headache pain. c. liver enzyme tests. d. serum glucose testing.

Answer: A RATIONALE: The nurse should first withhold the scheduled dose of Cozaar (A) because the client is hyperkalemic (normal range 3.5 to 5 mEq/L). Although hypokalemia is usually associated with diuretic therapy in heart failure, hyperkalemia is associated with several heart failure medications, including ARBs. Because hyperkalemia may lead to cardiac dysrhythmias, the nurse should check the apical pulse for rate and rhythm (B), and the blood pressure. Before repeating the serum study (D), the nurse should notify the healthcare provider (C) of the findings.

The nurse is preparing the 0900 dose of losartan (Cozaar), an angiotensin II receptor blocker (ARB), for a client with hypertension and heart failure. The nurse reviews the client's laboratory results and notes that the client's serum potassium level is 5.9 mEq/L. What action should the nurse take first? A) Withhold the scheduled dose. B) Check the client's apical pulse. C) Notify the healthcare provider. D) Repeat the serum potassium level.

ANS: B Codeine is normally a category II drug, except when it is part of a combination product such as with acetaminophen, making it a category III drug.

The nurse is preparing to administer a combination drug containing acetaminophen and codeine. The nurse knows that this drug is classified as which drug schedule? a. C-II b. C-III c. C-IV d. C-V

Answer: A RATIONALE: Before giving digoxin, the nurse will assess the heart rate and rhythm. The dosage will be held and the prescriber notified if the heart rate is below 60 beats per minute or if the cardiac rhythm has changed. Digoxin can cause bradycardia and electrical changes in the heart.

The nurse is preparing to administer a daily dose of digoxin (Lanoxin). Which assessment receives priority at this time? A Evaluating for a change in the heart rhythm B Assessing for Homans' sign C Checking the blood pressure D Palpating the pedal pulses

Answer: D RATIONALE: Patients receiving penicillin are at high risk for dangerous allergic reactions. This intervention represents the nurse's role in identifying situations with high risk. This situation does not represent the remaining responses

The nurse is preparing to administer a dose of penicillin. Before administering the medication, the nurse assesses the patient's allergy history. Which aspect of drug therapy does this represent? A)Making PRN (as needed) decisions B)Evaluating therapeutic effects C)Ensuring proper dosage D)Identifying high-risk patients

ANS: D Drugs that are highly protein bound bind with albumin and other proteins, leaving less free drug in circulation. If a patient has a low albumin, the drug is not bound, and there is more free drug to cause drug effects. There would be increased absorption, increased interactions with other drugs, and increased toxicity.

The nurse is preparing to administer a drug and learns that it binds to protein at a rate of 90%. The patient's serum albumin level is low. The nurse will observe the patient for a. decreased drug absorption. b. decreased drug interactions. c. decreased drug toxicity. d. increased drug effects.

ANS: D Increased creatinine and BUN indicate decreased renal function so a drug that is eliminated through the kidneys can become toxic. The nurse should discuss a lower dose with the provider. The drug will have a longer half life and will exhibit increased effects with decreased renal function.

The nurse is preparing to administer a drug that is eliminated through the kidneys. The nurse reviews the patient's chart and notes that the patient has increased serum creatinine and blood urea nitrogen (BUN). The nurse will perform which action? a. Administer the drug as ordered. b. Anticipate a shorter than usual half-life of the drug. c. Expect decreased drug effects when the drug is given. d. Notify the provider and discuss giving a lower dose.

ANS: B A drug with a longer half life should be given at longer intervals to avoid drug toxicity.

The nurse is preparing to administer a drug that is ordered to be given twice daily. The nurse reviews the medication information and learns that the drug has a half-life of 24 hours. What will the nurse do next? a. Administer the medication as ordered. b. Contact the provider to discuss daily dosing. c. Discuss every-other-day dosing with the provider. d. Hold the medication and notify the provider.

Answer: C RATIONALE: This order does not include a drug route. The nurse should clarify any questionable orders with the prescriber. The other responses are incorrect.

The nurse is preparing to administer a medication with the following order: "Aldomet 250 mg daily." What should the nurse do? A)Administer the medication as it was given last time. B)Administer the medication by mouth. C)Verify the order with the prescriber. D)Ask the patient how this medication is usually given.

ANS: A Schedule II drugs are controlled substances, and all must be accounted for. When wasting a portion of a drug, another nurse should observe and cosign that a drug was wasted.

The nurse is preparing to administer a schedule II injectable drug and is drawing up half of the contents of a single-use vial. Which nursing action is correct? a. Ask another nurse to observe and cosign wasting the remaining drug from the vial. b. Keep the remaining amount in the patient's drawer to give at the next dose. c. Record the amount unused in the patient's medication record. d. Dispose of the vial with the remaining drug into a locked collection box.

Answer: A RATIONALE: Digoxin levels have an inverse relationship with potassium levels. Because hydrochlorothiazide can lower potassium levels, combined use of hydrochlorothiazide and digoxin poses a risk for elevated digoxin levels and ensuing digoxin toxicity.

The nurse should monitor for which adverse effect after administering hydrochlorothiazide (HydroDIURIL) and digoxin (Lanoxin) to a patient? A Digoxin toxicity B Decreased diuretic effect C Dehydration D Heart failure

ANS: C Patients who have previously experienced manifestations of allergy to a penicillin should not use penicillins again unless necessary. The nurse should contact the provider to discuss using another antibiotic from a different class. Epinephrine and antihistamines are useful when patients are experiencing allergic reactions, depending on severity.

The nurse is preparing to administer amoxicillin (Amoxil) to a patient and learns that the patient previously experienced a rash when taking penicillin. Which action will the nurse take? a. Administer the amoxicillin and have epinephrine available. b. Ask the provider to order an antihistamine. c. Contact the provider to discuss using a different antibiotic. d. Request an order for a beta-lactamase resistant drug.

ANS: B The patient has signs of angioedema which indicates a hypersensitivity reaction. The nurse should hold the dose and notify the provider. Giving the dose will make the reaction more serious. These are not signs of edema, so a diuretic is not indicated. Electrolytes and renal function tests are not indicated.

The nurse is preparing to administer an angiotensin-converting enzyme (ACE) inhibitor to a patient who has hypertension. The nurse notes peripheral edema and swelling of the patient's lips. The patient has a blood pressure of 160/80 mm Hg and a heart rate of 76 beats per minute. What is the nurse's next action? a. Administer the dose and observe carefully for hypotension. b. Hold the dose and notify the provider of a hypersensitivity reaction. c. Notify the provider and request an order for a diuretic medication. d. Request an order for serum electrolytes and renal function tests.

ANS: D The sensitivity results from the patient's culture will reveal whether the organism is sensitive or resistant to a particular antibiotic. The patient is not responding to the antibiotic being given, so the antibiotic should be held and the provider notified. Another culture is not indicated. Antibiotics should be added only when indicated by the sensitivity.

The nurse is preparing to administer an antibiotic to a patient who has been receiving the antibiotic for 2 days after a culture was obtained. The nurse notes increased erythema and swelling, and the patient has a persistent high fever of 39° C. What is the nurse's next action? a. Administer the antibiotic as ordered. b. Contact the provider to request another culture. c. Discuss the need to add a second antibiotic with the provider. d. Review the sensitivity results from the patient's culture.

ANS: B Water soluble drugs require a carrier enzyme or protein to pass through the GI membrane.

The nurse is preparing to administer an oral medication that is water-soluble. The nurse understands that this drug a. must be taken on an empty stomach. b. requires active transport for absorption. c. should be taken with fatty foods. d. will readily diffuse into the gastrointestinal tract.

ANS: B The therapeutic range of digoxin is between 0.5 and 2 mg/mL. This patient's level is high, indicating toxicity. The nurse should not give the next dose or request a change in dose.

The nurse is preparing to administer digoxin to a patient who has a serum digoxin level of 2.5 ng/mL. The patient takes 0.25 mg of digoxin per day. What action will the nurse take? a. Administer the next dose as ordered. b. Notify the provider of digoxin toxicity. c. Request an order to decrease the digoxin dose. d. Suggest that the patient may need an increased digoxin dose.

ANS: B Nausea, vomiting, and headache are common signs of digoxin toxicity as is a heart rate less than 60 beats per minute. The nurse should hold the dose and notify the provider.

The nurse is preparing to administer digoxin to a patient who has heart failure. The patient reports nausea, vomiting, and a headache. The nurse notes a respiratory rate of 18 breaths per minute, a heart rate of 58 beats per minute, and a blood pressure of 120/78 mm Hg. What will the nurse do next? a. Administer the next dose as ordered since these are mild side effects. b. Hold the dose and notify the provider of possible digoxin toxicity. c. Reassure the patient that these are common, self-limiting side effects. d. Request an order for an antiemetic and an analgesic medication.

ANS: C When thiazide diuretics are taken with digoxin, patients are at risk of digoxin toxicity because thiazides can cause hypokalemia. The patient has bradycardia and blurred vision, which are both signs of digoxin toxicity. The nurse should hold the digoxin and notify the provider. Serum electrolytes may be ordered, but the digoxin should not be given.

The nurse is preparing to administer doses of hydrochlorothiazide (HydroDIURIL) and digoxin (Lanoxin) to a patient who has heart failure. The patient reports having blurred vision. The nurse notes a heart rate of 60 beats per minute and a blood pressure of 140/78 mm Hg. Which action will the nurse take? a. Administer the medications and request an order for serum electrolytes. b. Give both medications and evaluate serum blood glucose frequently. c. Hold the digoxin and notify the provider. d. Hold the hydrochlorothiazide and notify the provider.

ANS: D Gentamicin can cause nephrotoxicity. When changes in urine output occur, the provider should be notified, and serum trough levels should be obtained to make sure the drug is not at a toxic level. If the drug level is determined to be safe, giving extra fluids either orally or intravenously may be indicated. Serum peak levels give information about therapeutic levels but are not a substitution for avoiding nephrotoxicity in the face of possible oliguria.

The nurse is preparing to administer intravenous gentamicin to an infant through an intermittent needle. The nurse notes that the infant has not had a wet diaper for several hours. The nurse will perform which action? a. Administer the medication and give the infant extra oral fluids. b. Contact the provider to request adding intravenous fluids when giving the medication. c. Give the medication and obtain a serum peak drug level 45 minutes after the dose. d. Hold the dose and contact the provider to request a serum trough drug level.

ANS: A Drowsiness is a common side effect of phenytoin and is not cause for alarm. The patient's drug level is normal, since 10 20 mcg/mL is the therapeutic range. The nurse should administer the dose. It is not necessary to decrease the dose or monitor the patient more closely than usual.

The nurse is preparing to administer phenytoin (Dilantin) to a patient who has a seizure disorder. The patient appears drowsy, and the nurse notes that the last random serum drug level was 18 mcg/mL. What action will the nurse take? a. Administer the dose since the patient is not toxic. b. Contact the provider to discuss decreasing the phenytoin dose. c. Give the drug and monitor closely for adverse effects. d. Report drug toxicity to the providers.

ANS: A Intravenous phenytoin should be administered undiluted through a 3 way stopcock or Y-tubing. In older patients it should be infused at a rate of 25 mcg/min. The dose and the route are appropriate. Phenytoin will precipitate in dextrose solution. Intramuscular injection is very irritating to tissues and is not used.

The nurse is preparing to administer phenytoin to an 80-year-old patient and notes the following order: IVP phenytoin 50 mg. The nurse will perform which action? a. Administer the undiluted drug through a Y-tube over two minutes. b. Contact the provider to question the route and the dose. c. Dilute the drug in dextrose solution and infuse over 15 to 20 minutes. d. Request an order to administer the drug intramuscularly.

ANS: D To obtain the most accurate culture, the specimen should be obtained before antibiotic therapy begins. It is important to obtain cultures when possible in order to correctly identify the organism and help determine which antibiotic will be most effective. Administering test doses to determine hypersensitivity is sometimes done when there is a strong suspicion of allergy when a particular antibiotic is needed. Epinephrine is kept close at hand when there is a strong suspicion of allergy.

The nurse is preparing to administer the first dose of an antibiotic to a patient admitted for a urinary tract infection. Which action is most important prior to administering the antibiotic? a. Administering a small test dose to determine whether hypersensitivity exists b. Having epinephrine available in the event of a severe hypersensitivity reaction c. Monitoring baseline vital signs, including temperature and blood pressure d. Obtaining a specimen for culture and sensitivity

ANS: C Thiazide diuretics are contraindicated in renal failure. This patient has oliguria and should be evaluated for renal failure prior to administration of the diuretic—especially in the absence of known renal failure for this patient. Drinking more fluids will not increase urine output in patients with renal failure.

The nurse is preparing to administer the first dose of hydrochlorothiazide (HydroDIURIL) 50 mg to a patient who has a blood pressure of 160/95 mm Hg. The nurse notes that the patient had a urine output of 200 mL in the past 12 hours. The nurse will perform which action? a. Administer the medication as ordered. b. Encourage the patient to drink more fluids. c. Hold the medication and request an order for serum BUN and creatinine. d. Request an order for serum electrolytes and administer the medication.

ANS: A A small percentage of patients who are allergic to penicillin could also be allergic to a cephalosporin product. Patients should be monitored closely after receiving a cephalosporin if they are allergic to penicillin. There is no difference in hypersensitivity potential between different generations or method of delivery of cephalosporins.

The nurse is preparing to administer the first dose of intravenous ceftriaxone (Rocephin) to a patient. When reviewing the patient's chart, the nurse notes that the patient previously experienced a rash when taking amoxicillin. What is the nurse's next action? a. Administer the drug and observe closely for hypersensitivity reactions. b. Ask the provider whether a cephalosporin from a different generation may be used. c. Contact the provider to report drug hypersensitivity. d. Notify the provider and suggest an oral cephalosporin.

ANS: C Taking oral antidiabetic agents (sulfonylurea) with sulfonamides increases the hypoglycemic effect. Sulfonylureas do not increase the incidence of headaches, hypertension, or superinfection when taken with sulfonamides. Examples of antidiabetic sulfonylurea medications are glipizide, glimepride, glyburide, tolaamide, and tolbutamide.

The nurse is preparing to administer trimethoprim-sulfamethoxazole (TMP-SMX) to a patient who is being treated for a urinary tract infection. The nurse learns that the patient has type 2 diabetes mellitus and takes a sulfonylurea oral antidiabetic drug. The nurse will monitor this patient closely for which effect? a. Headaches b. Hypertension c. Hypoglycemia d. Superinfection

ANS: B Captopril is an angiotensin II inhibitor. Native American patients do not respond well to beta blockers. Acebutolol, carteolol, and metoprolol are all beta blockers.

The nurse is preparing to care for a Native-American patient who has hypertension. The nurse understands that which antihypertensive medication would be most effective in this patient? a. Acebutolol (Sectral) b. Captopril (Capoten) c. Carteolol HCl (Cartrol) d. Metoprolol (Lopressor)

ANS: C The nurse should provide factual information and answer questions. Persons of Latin American descent have less dependence on time schedules and do not tend to have a future orientation. They are not comfortable with periods of silence. Nurses should be receptive to traditional healing practices and seek ways to include those in care when they do not hinder safe and effective care; highlighting practices that won't work may convey a lack of respect for these traditions.

The nurse is preparing to discuss long-term care needs with a patient newly diagnosed with a chronic disease. The patient is of Latin American descent. The nurse will plan to take which action when teaching this patient? a. Discussing long-term outcomes associated with compliance of the prescribed regimen b. Highlighting various traditional healing practices that will not be effective for this patient's care c. Providing factual information and answering all questions as they arise d. Providing teaching in increments, allowing periods of silence to allow assimilation of information

ANS: D The patient's symptoms may indicate a superinfection and should be reported to the physician so it can be treated; however, the drug does not need to be held. It is not necessary to culture the lesions. The symptoms do not indicate impending anaphylaxis.

The nurse is preparing to give a dose of a cephalosporin medication to a patient who has been receiving the antibiotic for 2 weeks. The nurse notes ulcers on the patient's tongue and buccal mucosa. Which action will the nurse take? a. Hold the drug and notify the provider. b. Obtain an order to culture the oral lesions. c. Gather emergency equipment to prepare for anaphylaxis. d. Report a possible superinfection side effect of the cephalosporin.

ANS: D The trough drug level for gentamicin should be less than 2 mcg/mL. The nurse should not administer the drug and should notify the provider of the toxic level.

The nurse is preparing to give a dose of gentamicin to a patient and notes that the most recent serum gentamicin trough level was 2 mcg/mL. What will the nurse do next? a. Administer the drug as ordered. b. Administer the drug and monitor for adverse effects. c. Notify the provider to discuss decreasing the dose. d. Notify the provider to report a toxic drug level.

ANS: A A petechial rash can indicate a severe adverse reaction and should be reported.

The nurse is preparing to give trimethoprim-sulfamethoxazole (TMP-SMX) to a patient and notes a petechial rash on the patient's extremities. The nurse will perform which action? a. Hold the dose and notify the provider. b. Request an order for a blood glucose level. c. Request an order for a BUN and creatinine level. d. Request an order for diphenhydramine (Benadryl).

ANS: C Levofloxacin may increase the effects of oral hypoglycemic medications, so patients taking these should be advised to monitor their serum glucose levels closely. Antacids decrease the absorption of levofloxacin. NSAIDs taken with levofloxacin can cause central nervous system reactions, including seizures.

The nurse is providing discharge teaching for a patient who will receive oral levofloxacin (Levaquin) to treat pneumonia. The patient takes an oral hypoglycemic medication and uses over-the-counter (OTC) antacids to treat occasional heartburn. The patient reports frequent arthritis pain and takes acetaminophen when needed. Which statement by the nurse is correct when teaching this patient? a. "You may take antacids with levofloxacin to decrease gastrointestinal upset." b. "You may take nonsteroidal anti-inflammatory medications (NSAIDs) for arthritis pain." c. "You should monitor your serum glucose more closely while taking levofloxacin." d. "You should take levofloxacin on an empty stomach to improve absorption."

ANS: A AD is chronic and progressive, and there is no cure. It affects memory and personality. The onset is usually between 45 and 65 years. Symptoms cannot be arrested but may be slowed with treatment.

The nurse is providing teaching for the family of a patient who has been newly diagnosed with Alzheimer's disease (AD). Which statement by the family member indicates understanding of the teaching? a. "Alzheimer's disease is a chronic, progressive condition." b. "Alzheimer's disease affects memory but not personality." c. "The onset of Alzheimer's disease is usually between 65 and 75 years." d. "With proper treatment, symptoms of this disease can be arrested."

ANS: A Patients should take all of an antibiotic regimen even after symptoms clear to ensure complete treatment of the infection. Patients are often advised to eat yogurt or drink buttermilk to prevent superinfection. A rash is a sign of hypersensitivity, and patients should be counseled to stop taking the drug and notify the provider if this occurs. Alcohol consumption may cause adverse effects and should be avoided by patients while they are taking cephalosporins.

The nurse is providing teaching to a patient who will begin taking a cephalosporin to treat an infection. Which statement by the patient indicates a need for further teaching? a. "I may stop taking the medication if my symptoms clear up." b. "I should eat yogurt while taking this medication." c. "I should stop taking the drug and call my provider if I develop a rash." d. "I will not consume alcohol while taking this medication."

ANS: A Aspirin is not constipating, so patients do not need to be counseled to consume extra fluids and fiber. Abdominal pain can occur with gastrointestinal bleeding, and tinnitus (ringing in the ears) can be an early sign of toxicity, so patients should be taught to contact their provider if these occur. Taking a full glass of water with each dose helps minimize gastrointestinal side effects.

The nurse is providing teaching to a patient who will begin taking aspirin to treat arthritis pain. Which statement by the patient indicates a need for further teaching? a. "I should increase fiber and fluids while taking aspirin." b. "I will call my provider if I have abdominal pain." c. "I will drink a full glass of water with each dose." d. "I will notify my provider of ringing in my ears."

ANS: D Drug dosage for phenytoin is age related and children, who have a rapid metabolism, may need higher doses than those used for newborns and adults. Phenytoin has many drug interactions and many side effects. The therapeutic range is 10-20 mcg/mL.

The nurse is providing teaching to the parents of a 5-year-old child who will begin taking phenytoin (Dilantin). What information will the nurse include when teaching these parents about their child's medication? a. "Drug interactions are uncommon with phenytoin." b. "There are very few side effects associated with this drug." c. "The therapeutic range of phenytoin is between 15 and 30 mcg/mL." d. "Your child may need a higher dose than expected."

ANS: C Gentamycin peak values should be 5 to 8 mcg/mL, and trough levels should be 0.5 to 2 mcg/mL. Peak levels give information about whether or not a drug is at toxic levels, while trough levels indicate whether a therapeutic level is maintained. This drug is at a toxic level, and the next dose should not be given.

The nurse is reviewing a patient's chart prior to administering gentamycin (Garamycin) and notes that the last serum peak drug level was 9 mcg/mL and the last trough level was 2 mcg/mL. What action will the nurse take? a. Administer the next dose as ordered. b. Obtain repeat peak and trough levels before giving the next dose. c. Report possible drug toxicity to the patient's provider. d. Report a decreased drug therapeutic level to the patient's provider.

Answer: C RATIONALE: When beta1 receptors are activated, cardiac contraction is stimulated. When a beta1 agonist is indicated, epinephrine (given IV) is the preferred drug. Beta1 agonist drugs (e.g., epinephrine), which are kept on the emergency cart, may be injected directly into the heart during an arrest. Alpha1 activation causes hemostasis, nasal decongestion, vasoconstriction, and mydriasis. Alpha2 activation reduces sympathetic outflow to the heart and blood vessels and relieves severe pain. Beta2 activation causes bronchodilation and relaxation of uterine smooth muscle. Topical phenylephrine is helpful for nasal congestion. Terbutaline is indicated for preterm labor and/or asthma. Albuterol is indicated for asthma.

The nurse is reviewing drugs on the emergency cart with regard to their therapeutic action. Which medications can help initiate heart contraction during a cardiac arrest? A Topical phenylephrine B Subcutaneous terbutaline C Intravenous epinephrine D Inhaled albuterol

ANS: A Drugs that undergo first pass metabolism are absorbed into the portal vein from the intestinal lumen and go through the liver where they are either unchanged or are metabolized to an inactive or a more active form.

The nurse is reviewing medication information with a nursing student prior to administering an oral drug and notes that the drug has extensive first-pass effects. Which statement by the student indicates a need for further teaching about this medication? a. "The first-pass effect means the drug may be absorbed into systemic circulation from the intestinal lumen." b. "The first-pass effect means the drug may be changed to an inactive form and excreted." c. "The first-pass effect means the drug may be changed to a metabolite, which may be more active than the original." d. "The first-pass effect means the drug may be unchanged as it passes through the liver."

Answer: B RATIONALE: A life-threatening side effect of intravenous administration of morphine sulfate, an opiate narcotic, is respiratory depression (B). The PCA pump should be stopped and the healthcare provider notified if the client's respiratory rate falls below 12 breaths per minute, and the nurse should anticipate adjustments in the client's dosage before the PCA pump is restarted. (A, C, and D) provide helpful information, but are not as high a priority as the assessment described in (B).

The nurse is reviewing the use of the patient-controlled analgesia (PCA) pump with a client in the immediate postoperative period. The client will receive morphine 1 mg IV per hour basal rate with 1 mg IV every 15 minutes per PCA to total 5 mg IV maximally per hour. What assessment has the highest priority before initiating the PCA pump? A) The expiration date on the morphine syringe in the pump. B) The rate and depth of the client's respirations. C) The type of anesthesia used during the surgical procedure. D) The client's subjective and objective signs of pain.

ANS: A The maximum daily dose of acetaminophen is 4000 mg. If this patient takes 650 mg/dose 6 times daily, this amount is safe. Taking acetaminophen with caffeine increases the effect of the acetaminophen. Taking acetaminophen with OCPs decreases the effect of the acetaminophen but does not diminish the effect of the OCP. Many over the-counter medications contain acetaminophen, so patients should be advised to read labels carefully to avoid overdose.

The nurse is teaching a female patient who will begin taking 2 tablets of 325 mg acetaminophen every 4 to 6 hours as needed for pain. Which statement by the patient indicates understanding of the teaching? a. "I may take acetaminophen up to 6 times daily if needed." b. "I should increase the dose of acetaminophen if I drink caffeinated coffee." c. "If I take oral contraceptive pills, I should use back-up contraception." d. "It is safe to take acetaminophen with any over-the-counter medications."

ANS: B To measure liquid medications accurately, line up the bottom of the curve of the medication with the desired line on the syringe. Suspensions are liquids in which particles are mixed but not dissolved. As a general rule, elixirs do not require shaking; shaking may suspend air into the liquid and affect accurate dosing by volume. Many liquids require refrigeration.

The nurse is teaching a nursing student about giving liquid medications. Which statement by the student indicates understanding of the teaching? a. "A suspension is a mixture in which drug particles are dissolved in solution." b. "I will line up the bottom of the medication curve with the line in the syringe." c. "I will need to shake an elixir before measuring the dose." d. "I will not need to refrigerate liquids once they are reconstituted."

ANS: A Amphotericin B is not absorbed from the gastrointestinal tract, so is not given by mouth. It can cause nephrotoxicity and electrolyte imbalance. It is highly toxic and is reserved for severe, systemic infections.

The nurse is teaching a nursing student about the antifungal drug amphotericin B. Which statement by the student indicates a need for further teaching? a. "Amphotericin B may be given intravenously or by mouth." b. "Patients who take this drug should have potassium and magnesium levels assessed." c. "Patients with renal disease should not take amphotericin B." d. "This drug is used for severe systemic infections."

ANS: D Drugs that cause photosensitivity make sunburn more likely, so patients should stay out of the sun, wear protective clothing, and use sunscreen with an SPF greater than 15. It is not necessary to wear sunglasses indoors.

The nurse is teaching a patient about a drug that causes photosensitivity. Which statement by the patient indicates a need for further teaching? a. "I should apply sunscreen with a sun protection factor greater than 15." b. "I should avoid sunlight when possible while taking this drug." c. "I will wear protective clothing when I am outdoors." d. "I will wear sunglasses even while I am indoors."

ANS: C Patients should be instructed to draw up regular insulin first so that NPH is not mixed into the vial of regular insulin. It is not necessary to use separate syringes. Patients do not mix the medications in a vial.

The nurse is teaching a patient about home administration of insulin. The patient will receive regular (Humulin R) and NPH (Humulin NPH) insulin at 0700 every day. What is important to teach this patient? a. "Draw up the medications in separate syringes." b. "Draw up the NPH insulin first." c. "Draw up the regular insulin first." d. "Draw up the medications after mixing them in a vial."

ANS: D Because nitroglycerin can cause hypotension, patients should be cautioned to take them while sitting or lying down. If pain is not better or has worsened 5 minutes after the first dose, patients should call 911. The tablets must dissolve under the tongue and should not be swallowed.

The nurse is teaching a patient about sublingual nitroglycerin administration. What information will the nurse include when teaching this patient? a. Call 911 if pain does not improve after three doses. b. If pain persists after one dose, administer a second dose. c. Swallow the tablet with small sips of water. d. Take the first tablet while sitting or lying down.

ANS: A Patients do not need extra sodium or calcium while taking thiazide diuretics. Thiazide diuretics can lead to hypokalemia, so patients should be counseled to eat fruits and vegetables that are high in potassium. Patients can develop orthostatic hypotension and should be counseled to rise from sitting or lying down slowly. Taking the medication in the morning helps to prevent nocturia induced insomnia.

The nurse is teaching a patient about taking hydrochlorothiazide. Which statement by the patient indicates a need for further teaching? a. "I may need extra sodium and calcium while taking this drug." b. "I should eat plenty of fruits and vegetables while taking this medication." c. "I should take care when rising from a bed or chair when I'm on this medication." d. "I will take the medication in the morning to minimize certain side effects."

ANS: C Patients should be taught to rotate application sites when using the transdermal nitroglycerin. Transdermal nitroglycerin is not used as needed. Patients should remove the patch at bedtime to provide an 8 to 12-hour nitrate-free interval. Patients should use the patch even when symptom-free unless otherwise instructed by the provider.

The nurse is teaching a patient about the use of a transdermal nitroglycerin patch. Which statement by the patient indicates understanding of the teaching? a. "I will apply the patch as needed when I experience anginal pain." b. "I will remove the old patch and replace it with a new one at bedtime each day." c. "I should rotate sites when changing the patch to prevent skin irritation." d. "When I am symptom-free, I may stop using the patch on a regular basis."

ANS: D Drugs given sublingually should be placed under the tongue. No foods or fluids should be given, since the tablet must remain under the tongue until it is fully absorbed. Medications ordered to be given "buccally" should be placed between the cheek and gum.

The nurse is teaching a patient about using sublingual nitroglycerin at home. Which statement by the patient indicates understanding of the teaching? a. "I may put the tablet in food if I don't like the taste." b. "I may take a sip of water after placing the tablet in my mouth." c. "I will place the tablet between my cheek and gum." d. "I will place the tablet under my tongue and let it absorb."

ANS: C In a thin person, with little fatty tissue, the needle is inserted at a 45 to 60-degree angle. In other patients, a 45- to 90-degree angle is acceptable. There is no recommendation for preferring one site over another.

The nurse is teaching a patient how to administer insulin. The patient is thin with very little body fat. The nurse will suggest injecting insulin a. by pinching up the skin and injecting straight down. b. in the abdomen only with the needle at a 90-degree angle. c. subcutaneously with the needle at a 45-to 60-degree angle. d. using the thigh and buttocks areas exclusively.

ANS: D Taking carbidopa levodopa with food decreases absorption of the drug, although gastrointestinal distress may decrease when the medication is taken with food. Cholinergic side effects are common. Orthostatic hypotension occurs early and will resolve over time. Nightmares and mental disturbances should be reported.

The nurse is teaching a patient who has Parkinson's disease about the side effects of carbidopa-levodopa. Which statement by the patient indicates a need for further teaching? a. "I may experience urinary retention, dry mouth, and constipation." b. "I may feel dizzy at first, but this side effect will go away with time." c. "I should report nightmares and mental disturbances to my provider." d. "I should take the drug with food to increase absorption."

ANS: D Patients who take beta blockers should avoid all alcohol because it increases the hypotensive effects. It does not cause reflex hypertension.

The nurse is teaching a patient who has hypertension about long-term management of the disease and a beta blocker. The patient reports typically consuming 1 to 2 glasses of wine each evening with meals. How will the nurse respond? a. "Beta blockers and wine cause a reflex hypertension." b. "Four to 6 ounces of wine is considered safe with these medications." c. "Wine in moderation helps you relax and get better blood pressure control." d. "Wine increases the hypotensive effects of the beta blocker."

ANS: C Lipohypertrophy is a raised lump or knot on the skin surface caused by repeated injections into the same site, and this can interfere with insulin absorption. Patients are encouraged to use the same site for a week, giving each injection a knuckle length away from the previous injection. Insulin absorption is greater when given in abdominal areas.

The nurse is teaching a patient who is newly diagnosed with type 1 diabetes mellitus about insulin administration. Which statement by the patient indicates a need for further teaching? a. "I may use a chosen site daily for up to a week." b. "I should give each injection a knuckle length away from a previous injection." c. "I will not be concerned about a raised knot under my skin from injecting insulin." d. "Insulin is absorbed better from subcutaneous sites on my abdomen.

ANS: C Patients who develop signs of allergy, such as rash, should notify their provider before continuing medication therapy. Patients should be counseled to continue taking their antibiotics until completion of the prescribed regimen even when they feel well. Diarrhea is an adverse effect but does not warrant cessation of the drug. Before deciding to stop taking a medication due to a side effect, encourage the patient to contact the provider first. Patients should discard any unused antibiotic.

The nurse is teaching a patient who will be discharged home from the hospital to take amoxicillin (Amoxil) twice daily for 10 days. Which statement by the nurse is correct? a. "Discontinue the antibiotic when your temperature returns to normal and your symptoms have improved." b. "If diarrhea occurs, stop taking the drug immediately and contact your provider." c. "Stop taking the drug and notify your provider if you develop a rash while taking this drug." d. "You may save any unused antibiotic to use if your symptoms recur."

ANS: C Enteric coated tablets resist disintegration in the acidic environment of the stomach and disintegrate when they reach the small intestine. There is usually some delay in onset of actions after taking these medications. Enteric-coated tablets should not be crushed or chewed, which would alter the time and location of absorption. Acidic foods will not enhance the absorption of the medication. The patient should not to eat high-fat food before ingesting an enteric-coated tablet, because high-fat foods decrease the absorption rate.

The nurse is teaching a patient who will be discharged home with a prescription for an enteric-coated tablet. Which statement by the patient indicates understanding of the teaching? a. "I may crush the tablet and put it in applesauce to improve absorption." b. "I should consume acidic foods to enhance absorption of this medication." c. "I should expect a delay in onset of the drug's effects after taking the tablet." d. "I should take this medication with high-fat foods to improve its action."

ANS: C Targeted therapy differs from traditional cancer chemotherapy by taking advantage of biologic features particular to cancer cells and targeting specific mechanisms. They block the growth and spread of cancer by interfering with specific molecules within the cancer cells. Traditional chemotherapeutic agents damage cell DNA of cancer cells as well as normal cells. Targeted therapies do not directly kill or damage cancer cells or prevent metastasis.

The nurse is teaching a patient who will begin receiving targeted therapy for cancer. The patient asks how targeted therapy differs from other types of chemotherapies. The nurse will explain that targeted therapy a. damages cancer cell DNA to prevent cell replication. b. directly kills or damages cancerous cells. c. interferes with specific molecules in cancer cells. d. prevents metastasis of cancer cells.

ANS: B Dairy products, multivitamins, and antacids should be avoided 1 hour before and 2 hours after taking ciprofloxacin because these products contain divalent cations that form a drug complex that prevents absorption of the ciprofloxacin.

The nurse is teaching a patient who will begin taking ciprofloxacin. What instruction will the nurse include when teaching this patient about this drug? a. "Do not take this medication with oral contraceptive pills." b. "Take at least 1 hour after or 2 hours before taking antacids." c. "Take in the morning with your multivitamin tablet." d. "Take with milk to reduce gastric upset."

ANS: C The interaction of furosemide and a steroid drug can result in an increased loss of potassium. Patients should take a potassium supplement. Patients should avoid licorice while taking furosemide, partially due to the hypokalemic effects of both substances. Urine output greater than 600 mL/24 hours is normal. Patients should take furosemide in the morning to avoid nocturia.

The nurse is teaching a patient who will begin taking furosemide. The nurse learns that the patient has just begun a 2-week course of a steroid medication. What will the nurse recommend? a. Consume licorice to prevent excess potassium loss. b. Report a urine output greater than 600 mL/24 hours. c. Obtain an order for a potassium supplement. d. Take the furosemide at bedtime.

ANS: C Caffeine and theophylline are both xanthine derivatives and should not be taken together because of the increased risk of toxicity and severe adverse effects. Theophylline has a narrow therapeutic range and must be dosed carefully; patients should never increase or decrease the dose without consulting their provider. Gastrointestinal symptoms are common side effects. Food slows absorption but does not prevent the full dose from being absorbed.

The nurse is teaching a patient who will begin taking oral theophylline (Theo-Dur) when discharged home from the hospital. What information will the nurse include when teaching the patient about this drug? a. An extra dose should be taken when symptoms worsen. b. Anorexia and gastrointestinal upset are unexpected side effects. c. Avoid caffeine while taking this medication. d. Food will decrease the amount of drug absorbed.

ANS: D Patients taking warfarin should tell their dentists that they are taking the medication because of the increased risk for bleeding. Patients should avoid foods high in vitamin K, which can decrease the effects of warfarin. Patients should not take NSAIDs or cimetidine (Tagamet) because they can displace warfarin from protein binding sites.

The nurse is teaching a patient who will begin taking warfarin (Coumadin) for atrial fibrillation. Which statement by the patient indicates understanding of the teaching? a. "I should eat plenty of green, leafy vegetables while taking this drug." b. "I should take a nonsteroidal anti-inflammatory drug (NSAID) instead of acetaminophen for pain or fever." c. "I will take cimetidine (Tagamet) to prevent gastric irritation and bleeding." d. "I will tell my dentist that I am taking this medication."

ANS: C Patients taking acyclovir should increase fluid intake to maintain hydration. A complete blood count is not required. Dizziness and confusion should be reported to the provider. Antiviral medications have many side effects.

The nurse is teaching a patient who will receive acyclovir for a herpes virus infection. What information will the nurse include when teaching this patient? a. Blood cell counts should be monitored closely. b. Dizziness and confusion are harmless side effects. c. Increase fluid intake while taking this medication. d. Side effects are rare with this medication.

Answer: D RATIONALE: The EpiPen should be stored in a cool, dark place, but refrigeration can damage the injection mechanism. The medication should be taken at the first sign of symptoms. Anaphylaxis can develop within minutes after allergen exposure. To use the EpiPen, the patient should form a fist around the unit with the black tip pointing down, remove the activation cap, jab the device firmly into the outer thigh, wait 10 seconds, remove the unit, and massage the area for 10 seconds. The medication can be given directly through clothing if necessary.

The nurse is teaching a patient with a history of anaphylaxis how to use an EpiPen. Which statement made by the patient indicates that he understands the proper use of this drug? A "I will keep my medication in the refrigerator when I'm not using it." B "I should take this medication within 30 minutes of the onset of symptoms." C "I must remove my pants before injecting the medication into the leg." D "I will jab this medication firmly into my outer thigh if needed."

Answer: C RATIONALE: Full analgesic effects can take up to 24 hours to develop with fentanyl patches. Most patches are changed every 72 hours. Fentanyl has the same adverse effects as other opioids, including respiratory depression. Patients should avoid exposing the patch to external heat sources, because this may increase the risk of toxicity.

The nurse is teaching a patient with cancer about a new prescription for a fentanyl (Sublimaze) patch, 25 mcg/hr, for chronic back pain. Which statement is the most appropriate to include in the teaching plan? A "You will need to change this patch every day, regardless of your pain level." B "This type of pain medication is not as likely to cause breathing problems." C "With the first patch, it will take about 24 hours before you feel the full effects." D "Use your heating pad for the back pain. It will also improve the patch's effectiveness."

Answer: C RATIONALE: Enoxaparin acts primarily on factor Xa and also but to a lesser degree on thrombin. Unfractionated heparin equally reduces the action of thrombin and factor Xa. Fondaparinux (Arixtra) causes selective inhibition of factor Xa. Low-molecular-weight (LMW) heparins, such as enoxaparin, have greater bioavailability and a longer half-life than those of unfractionated heparin.

The nurse knows that which statement is accurate for enoxaparin (Lovenox)? A It equally reduces the activity of thrombin and factor Xa. B It has selective inhibition of factor Xa and no effect on thrombin. C It reduces the activity of factor Xa more than the activity of thrombin. D It has a lower bioavailability and shorter half-life than unfractionated heparin.

ANS: A Increased blood pressure can result when ginkgo is used in combination with a thiazide diuretic. Hawthorne can potentiate hypotension. Licorice can increase potassium loss, leading to hypokalemia. St. John's wort is not listed as an herbal alert substance with thiazide diuretics.

The nurse notes a blood pressure of 160/90 mm Hg in a patient taking a thiazide diuretic. The patient reports taking an herbal medication that a friend recommended. Which herbal product is likely, given this patient's blood pressure? a. Ginkgo b. Hawthorne c. Licorice d. St. John's wort

ANS: D Certain classifications of medications have different effects in individuals whose genetic markers are predominantly characteristic of a certain biologic group. African Americans respond poorly to several classes of antihypertensive agents.

The nurse notes that a patient of African American descent who is taking an oral antihypertensive medication continues to have elevated blood pressure three months after beginning the medication regimen. The nurse suspects that the patient may be a. consuming ethnic foods that interfere with absorption of the drug. b. discarding the medication. c. experiencing allergic reactions to the medication. d. metabolizing the drug differently than expected.

ANS: D If a patient is taking digoxin and a potassium wasting diuretic such as thiazide, the patient should also take a potassium supplement to prevent hypokalemia that could result in digoxin toxicity. It is not necessary to take cortisone, lidocaine, or nitroglycerin unless the patient has symptoms that warrant these drugs.

The nurse performs a medication history and learns that the patient takes a thiazide diuretic and digoxin (Lanoxin). The nurse will question the patient to ensure that the patient is also taking which medication? a. Cortisone b. Lidocaine c. Nitroglycerin d. Potassium

Answer: C RATIONALE: High-ceiling loop diuretics, such as furosemide, are the most effective diuretic agents. They produce more loss of fluid and electrolytes than any others. A sudden loss of fluid can result in decreased blood pressure. When blood pressure drops, the pulse probably will increase rather than decrease. Lasix should not affect respirations or temperature. The nurse should also closely monitor the patient's potassium level.

The nurse plans to closely monitor for which clinical manifestation after administering Furosemide (Lasix)? A Decreased pulse B Decreased temperature C Decreased blood pressure D Decreased respiratory rate

ANS: A Oral drugs may have less bioavailability because a lower percentage of the drug reaches the systemic circulation. Pinocytosis refers to the process by which cells carry a solute across a membrane. Protein binding can occur with both routes. Tachyphylaxis describes a rapid decrease in response to drugs that occurs when tolerance develops quickly.

The nurse prepares to change a patient's medication from an intravenous to an oral form and notes that the oral form is ordered in a higher dose. The nurse understands that this is due to differences in a. bioavailability. b. pinocytosis. c. protein binding. d. tachyphylaxis.

ANS: D Phenytoin will cause reddish brown colored urine. Patients should be counseled to report a rash to the provider because it could be a serious adverse reaction. Bleeding gums are common, but patients should never stop taking anticonvulsants abruptly, or they may develop seizures. Changes in blood glucose may occur but do not necessarily result in diabetes.

The nurse provides teaching for a patient who will begin taking phenytoin. Which statement by the patient indicates understanding of the teaching? a. "If I develop a rash, I should take diphenhydramine to control the itching." b. "If I experience bleeding gums, I should stop taking the medication immediately." c. "I may develop diabetes while I am taking this medication." d. "I should not be alarmed if my urine turns reddish-brown."

ANS: D Montelukast and other leukotriene receptor antagonists are not used to treat acute symptoms. Because they can affect liver enzymes, periodic liver function tests should be performed. Patients taking this drug should not use ibuprofen or aspirin for pain or fever if they have an aspirin sensitivity. Patients will achieve maximum effectiveness if the drug is taken in the evening.

The nurse provides teaching for patient who will begin taking montelukast sodium (Singulair). The patient reports sensitivity to aspirin. Which statement by the patient indicates a need for further teaching? a. "I will need to have periodic laboratory tests while taking this medication." b. "I will not take ibuprofen for pain or fever while taking this drug." c. "I will take one tablet daily at bedtime." d. "I will use this as needed for acute symptoms."

ANS: C The statins can affect visual acuity, so patients should be counseled to have annual eye examinations for assessment of cataract formation. The bile acid sequestrants, not statins, cause diarrhea. Statin drug therapy is lifelong or until behavioral changes prove equally effective (uncommon). Bile acid sequestrants, not statins, decrease the absorption of fat soluble vitamins.

The nurse provides teaching to a patient who will begin taking simvastatin (Zocor) to treat hyperlipidemia. Which statement by the patient indicates understanding of the teaching? a. "I may have diarrhea as a result of taking this medication." b. "I may stop taking this medication when my lipid levels are normal." c. "I will need an annual eye examination while taking this medication." d. "I will increase my intake of vitamins A, D, and E while taking this medication.

ANS: B Purine nucleosides, such as acyclovir, are used to treat herpes simplex viruses 1 and 2, herpes zoster virus, varicella zoster virus, and cytomegalovirus.

The nurse receives an order to administer a purine nucleoside antiviral medication. The nurse understands that this medication treats which type of virus? a. Hepatitis virus b. Herpes virus c. HIV d. Influenza virus

ANS: A Acyclovir is used for herpes zoster, but the dose should be 800 mg 5 times daily for 7 to 10 days. The nurse should clarify the dose and frequency. For herpes simplex, 400 mg 3 times daily is correct.

The nurse receives the following order for a patient who is diagnosed with herpes zoster virus: PO acyclovir (Zovirax) 400 mg TID for 7 to 10 days. The nurse will contact the provider to clarify which part of the order? a. Dose and frequency b. Frequency and duration c. Drug and dose d. Drug and duration

ANS: B Only regular insulin can be given intravenously. The nurse should clarify the order. It is not correct to give Humulin NPH insulin IV. The nurse should not administer the drug by a different route without first discussing with the provider.

The nurse receives the following order for insulin: IV NPH (Humulin NPH) 10 units. The nurse will perform which action? a. Administer the dose as ordered. b. Clarify the insulin type and route. c. Give the drug subcutaneously. d. Question the insulin dose.

ANS: D Insulin is given by the subcutaneous route. Only regular insulin may be given IV.

The nurse will administer parenteral insulin to a patient who will receive a mixture of NPH (Humulin NPH) and regular (Humulin R). The nurse will give this medication via which route? a. Intradermal b. Intramuscular c. Intravenous d. Subcutaneous

ANS: D Low molecular-weight heparin is an anticoagulant, which is used to inhibit clot formation and is used prophylactically to prevent postoperative deep vein thrombosis. Alteplase is a thrombolytic, which is used to break down clots after they form; alteplase is contraindicated in any patient with recent surgery. Aspirin and clopidogrel are antiplatelet drugs and are used to prevent arterial thrombosis. - The nurse is caring for a postoperative patient

The nurse will anticipate administering which medication to this patient to help prevent thrombus formation caused by slow venous blood flow? a. Alteplase (Activase) b. Aspirin c. Clopidogrel (Plavix) d. Low-molecular-weight heparin

ANS: A Acetaminophen is safe to give children and does not cause gastrointestinal upset or interfere with platelet aggregation. Aspirin carries an increased risk of Reye's syndrome in children. Diflunisal (Dolobid) is not available over the counter.

The parent of a 5-year-old child asks the nurse to recommend an over-the-counter pain medication for the child. Which analgesic will the nurse recommend? a. Acetaminophen (Tylenol) b. Aspirin (Ecotrin) c. Diflunisal (Dolobid) d. Ibuprofen (Motrin)

ANS: B Metoclopramide can cause extrapyramidal symptoms, and these effects are more likely in children. Children are not more prone to sedative effects, paralytic ileus, or vertigo while taking this drug.

The parent of a child who is receiving chemotherapy asks the nurse why metoclopramide (Reglan) is not being used to suppress vomiting. The nurse will explain that, in children, this drug is more likely to cause which effect? a. Excess sedation b. Extrapyramidal symptoms c. Paralytic ileus d. Vertigo

ANS: D Patients generally need less insulin with increased exercise, so the child should consume a snack to prevent hypoglycemia. Exercise is an integral part of diabetes management. Hypoglycemia is more likely to occur, and extra insulin is not indicated.

The parent of a junior high-school child who has type 1 diabetes asks the nurse if the child can participate in sports. The nurse will tell the parent a. that strenuous exercise is not recommended for children with diabetes. b. that the child must be monitored for hyperglycemia while exercising. c. to administer an extra dose of regular insulin prior to exercise. d. to send a snack with the child to eat just prior to exercise.

ANS: D Some combination pens do not require refrigeration after first use. Storing insulin in the freezer is not recommended. Opened vials may either be kept at room temperature for a month or refrigerated for 3 months.

The patient asks the nurse about storing insulin. Which response by the nurse is correct? a. "All insulin vials must be refrigerated." b. "Insulin will last longer if kept in the freezer." c. "Opened vials of insulin must be discarded." d. "Some combination pens do not require refrigeration."

Answer: A Rationale: Nonselective adrenergic agonist bronchodilators stimulate beta1 receptors in the heart and beta2 receptors in the lungs. Stimulation of beta1 receptors can increase heart rate and contractility, increasing oxygen demand. This increased oxygen demand may lead to angina or myocardial ischemia in patient with coronary artery disease. Cautious use of these agents is indicated if the patient has coronary artery disease.

The patient is taking a nonselective adrenergic agonist bronchodilator and has a history of coronary artery disease. What is a priority nursing intervention? A Monitoring patient for potential chest pain B Monitoring blood pressure continuously C Assessing daily for hyperkalemia D Assessing 12-lead ECG each shift

ANS: D Penicillin G procaine (Wycillin) has a milky appearance; therefore, the appearance should not concern the nurse.

The patient will begin taking penicillin G procaine (Wycillin).The nurse notes that the solution is milky in color. What action will the nurse take? a. Call the pharmacist and report the milky color. b. Add normal saline to dilute the medication. c. Call the physician and report the milky appearance. d. Administer the medication as ordered by the physician.

ANS: B PD is a progressive disorder. Nonpharmacologic measures can lessen symptoms and help patients and families cope with the disorder. Although the aging process may contribute to the development of PD, it is not necessarily a normal part of aging. Treatment may slow the progression but does not arrest or cure the disease.

The spouse of a patient newly diagnosed with mild, unilateral symptoms of Parkinson's disease (PD) asks the nurse what, besides medication, can be done to manage the disease. The nurse will a. counsel the spouse that parkinsonism is a normal part of the aging process in some people. b. recommend exercise, nutritional counseling, and group support to help manage the disease. c. tell the spouse that the disease will not progress if mild symptoms are treated early. d. tell the spouse that medication therapy can be curative if drugs are begun in time.

Answer: D RATIONALE: Itching and redness at the IM injection site indicate an allergy to penicillin. The primary treatment is epinephrine (subcutaneous, IM, or IV) plus respiratory support. Elevation, ice packs, and calming the patient are done once epinephrine has been administered.

Thirty minutes after receiving an intramuscular (IM) injection of penicillin G (Pfizerpen), a patient reports itching and redness at the injection site. Which action should the nurse take first? A Elevate the lower legs. B Place an ice pack on the site. C Make sure the patient stays calm. D Administer subcutaneous epinephrine.

Answer: C RATIONALE: Parkinson's disease results from a decrease in dopaminergic (inhibitory) activity, leaving an imbalance with too much cholinergic (excitatory) activity. With an increase in dopamine, the neurotransmitter activity becomes more balanced, and symptoms are controlled.

What is the goal of pharmacologic therapy in the treatment of Parkinson's disease? A To increase the amount of acetylcholine at the presynaptic neurons B To reduce the amount of dopamine available in the substantia nigra C To balance cholinergic and dopaminergic activity in the brain D To block dopamine receptors in both presynaptic and postsynaptic neurons

Answer: C

What is the most important thing for the nurse to teach the client with a history of diabetes and asthma who has started on albuterol PRN? a. Take Tylenol for headaches when taking albuterol. b. Monitor for orthostatic hypotension every 2 hours when taking albuterol. c. Monitor blood glucose levels every 4 hours when taking albuterol. d. An antianxiety agent may be prescribed to help with nervousness.

Answer: D

What is the role of corticosteroids in the treatment of acute respiratory disorders? A. They stimulate the immune system. B. They directly dilate the bronchi. C. They increase gas exchange in the alveoli. D. They decrease inflammation.

Answer: D Rationale: St. John's wort has been shown to decrease serum montelukast (Singulair) levels. The other substances do not interact with montelukast.

What over-the-counter product will the nurse instruct the patient to avoid when taking montelukast (Singulair)? A Acetaminophen (Tylenol) B Echinacea C Diphenhydramine (Benadryl) D St. John's wort

Answer: B

What side effect of angiotensin-converting enzyme (ACE) inhibitors most often results in the provider changing the treatment plan to an angiotensin receptor blocker? A Orthostatic hypotension B Dry, nonproductive cough C Fatigue D Hypokalemia

Answer: C RATIONALE: Glucocorticoids given in larger pharmacologic doses for nonendocrine causes produce many adverse effects, including gastric irritation and ulcers. They should be given with food. Doses should be administered before 9 AM to maximize endocrine function. Prednisone should not be stopped abruptly; the patient should contact the healthcare provider before discontinuing the medication. The drug may be administered by many routes, including the parenteral route.

When administering prednisone to a patient, the nurse will do what? A Administer the prednisone in the evening to coincide with the natural secretion pattern of the adrenal cortex. B Instruct the patient to stop taking the prednisone immediately if diarrhea develops. C Ensure that meals are at bedside so that administration with food reduces gastric irritation. D Avoid intravenous delivery to prevent adverse effects.

Answer: C

When metoclopramide (Reglan) is given for nausea, the client is cautioned to avoid which substance? a. Milk b. MAOIs c. Alcohol d. Carbonated beverages

Answer: B,C,D RATIONALE: For a penicillin allergy skin test, a small amount of allergen is injected intradermally. The nurse observes for a local allergic reaction and has epinephrine and respiratory support readily available. Epinephrine is administered as the first-line agent should anaphylaxis occur.

When performing a skin test for penicillin allergy, the nurse will do what? (Select all that apply.) A Inject a tiny amount of the allergen subcutaneously. B Observe for a local allergic response. C Have epinephrine readily available. D Have respiratory support readily available. E Administer diphenhydramine (Benadryl) as the first-line agent should anaphylaxis occur.

ANS: A, C, E Frequent use of antibiotics increases the exposure of bacteria to an antibiotic and results in acquired resistance. Skipping doses of an antibiotic can lead to incomplete treatment of an infection, and the remaining bacteria may develop acquired resistance. Treating viral infections with antibiotics is unnecessary and may cause acquired resistance to develop from unneeded exposure to a drug. Infections adequately treated with an antibiotic do not result in resistance.

Which actions can contribute to bacterial resistance to antibiotics? (Select all that apply.) a. Frequent use of antibiotics b. Giving large doses of antibiotics c. Skipping doses d. Taking a full course of antibiotics e. Treating viral infections with antibiotics

Answer: A RATIONALE: The statin drugs, such as atorvastatin, are the most effective drugs available for lowering LDL cholesterol. They are better tolerated, have fewer adverse effects, and produce better clinical outcomes than any other agents available for lowering LDL.

Which agent is in the category of drugs considered most effective for lowering LDL cholesterol? A Atorvastatin (Lipitor) B Cholestyramine (Questran) C Gemfibrozil (Lopid) D Ezetimibe (Zetia)

Answer: A RATIONALE: Levofloxacin should not be administered with milk products or antacids containing magnesium or aluminum, because this reduces absorption from the gastrointestinal (GI) tract. However, this does not happen with most foods. Premedicating with diphenhydramine is unnecessary.

Which approach should a nurse take when administering an oral dose of levofloxacin (Levaquin)? A Give the medication with or without food. B Administer the drug with an oral dose of a magnesium-based antacid. C Premedicate the patient with diphenhydramine (Benadryl). D Administer the drug with milk products.

Answer: B,C,D

Which are common side effects of nitroglycerin? (Select all that apply.) A Blurred vision B Flushing C Headache D Hypotension

ANS: A, C, E The FDA ensures that drugs are labeled correctly, that they are tested and proven effective for the conditions they are marketed to treat, and that they are tested for harmful effects. The FDA does not ensure affordability or freedom from adverse reactions, although these must be noted in drug information materials.

Which are responsibilities of the U.S. Food and Drug Administration (FDA)? (Select all that apply.) a. To ensure a drug has accurate labeling b. To ensure a drug is affordable c. To ensure a drug is effective d. To ensure a drug is free from adverse reactions e. To ensure a drug is tested for harmful effects

Answer: B,C,D RATIONALE: The three main families of antianginal agents are organic nitrates, beta blockers, and calcium channel blockers. Platelet inhibitors and statins (unless contraindicated) are incorporated into the treatment plan to reduce the risk of myocardial infarction.

Which are the main families of drugs used to prevent or relieve anginal pain? (Select all that apply.) A Platelet inhibitors B Beta blockers C Nitrates D Calcium channel blockers E Statins

Answer: B RATIONALE: Evaluation is one of the most important aspects of drug therapy, because it tells the nurse whether a drug is having its intended effect. The other aspects of drug administration are important but do not give information about a drug's effectiveness.

Which aspect of drug therapy indicates to the nurse whether a drug is having a beneficial effect? A Performing a preadministration assessment B Evaluating therapeutic responses C Minimizing adverse effects D Managing toxicity

ANS: C Donepezil is used to treat Alzheimer's disease. Ambenonium and neostigmine are used to treat myasthenia gravis. Benztropine is used to treat Parkinson's disease.

Which cholinesterase inhibitor would be prescribed for a patient who has Alzheimer's disease? a. Ambenonium chloride (Myletase) b. Benztropine (Cogentin) c. Donepezil HCl (Aricept) d. Neostigmine methylsulfate (Prostigmin)

ANS: A, D, E Herpes viruses cause chicken pox, mononucleosis, and shingles.

Which diseases are caused by herpes viruses? (Select all that apply.) a. Chicken pox b. Hepatitis c. Influenza d. Mononucleosis e. Shingles

Answer: B RATIONALE: Tetracyclines are bacteriostatic antibiotics; photosensitivity and severe sunburn are common adverse effects. A full course of antibiotics must always be taken. Blood studies are not necessary for therapeutic levels. Absorption decreases after ingestion of chelates, such as calcium and magnesium, so doses should be given 2 hours before or 2 hours after ingestion of milk products.

Which instruction should a nurse include in the discharge teaching for a patient who is to start taking tetracycline (Sumycin)? A "You may stop taking the pills when you begin to feel better." B "Use sunscreen and protective clothing when outdoors." C "You'll have to come back to the clinic for weekly blood work." D "Take the medication with yogurt or milk so you won't have nausea."

Answer: A RATIONALE: To ensure a consistent response, only NPH insulin is appropriate for mixing with a short-acting insulin. Unopened vials of insulin should be refrigerated; current vials can be kept at room temperature for up to 1 month. Drawing up the regular insulin into the syringe first prevents accidental mixture of NPH insulin into the vial of regular insulin, which could alter the pharmacokinetics of subsequent doses taken out of the regular insulin vial. NPH insulin is a cloudy solution, and it should always be rotated gently to disperse the particles evenly before loading the syringe. Subcutaneous injections should be made using one region of the body (e.g., the abdomen or thigh) and rotated within that region for 1 month.

Which instruction should the nurse provide when teaching a patient to mix regular insulin and NPH insulin in the same syringe? A "Draw up the clear regular insulin first, followed by the cloudy NPH insulin." B "It is not necessary to rotate the NPH insulin vial when it is mixed with regular insulin." C "The order of drawing up insulin does not matter as long as the insulin is refrigerated." D "Rotate subcutaneous injection sites each day among the arm, thigh, and abdomen."

Answer: A, D, E, F RATIONALE: Increased fluid intake and cranberry juice (A) are recommended for prevention and treatment of urinary tract infections, which frequently accompany vaginal infections. It is not necessary to taper use of this drug (B) or to check the blood pressure daily (C), as this condition is not related to hypertension. Flagyl can cause a disulfiram-like reaction if taken in conjunction with ingestion of alcohol, so the client should be instructed to avoid alcohol (D). All sexual partners should be treated at the same time (E) and condoms should be used until after treatment is completed to avoid reinfection (F).

Which instruction(s) should the nurse give to a female client who just received a prescription for oral metronidazole (Flagyl) for treatment of trichomonas vaginalis? (Select all that apply.) A) Increase fluid intake, especially cranberry juice. B) Do not abruptly discontinue the medication; taper use. C) Check blood pressure daily to detect hypertension. D) Avoid drinking alcohol while taking this medication. E) Use condoms until treatment is completed. F) Ensure that all sexual partners are treated at the same time.

ANS: D Personal contact with a person having a diagnosis of tuberculosis is required to indicate prophylactic treatment with antitubercular therapy. Attending the same school does not necessarily mean close contact occurs. Health care professionals do not need prophylactic treatment. HIVpositive individuals with negative TB skin tests do not need prophylaxis.

Which person should be treated with prophylactic antitubercular medication? a. A child who attends the same school with a child who has tuberculosis b. A nurse who is working in a hospital c. An individual who is HIV-positive with a negative TB skin test d. A patient who has close contact with someone who has tuberculosis

Answer: A, D, E, F RATIONALE: Increased fluid intake and cranberry juice (A) are recommended for prevention and treatment of urinary tract infections, which frequently accompany vaginal infections. It is not necessary to taper use of this drug (B) or to check the blood pressure daily (C), as this condition is not related to hypertension. Flagyl can cause a disulfiram-like reaction if taken in conjunction with ingestion of alcohol, so the client should be instructed to avoid alcohol (D). All sexual partners should be treated at the same time (E) and condoms should be used until after treatment is completed to avoid reinfection (F).

Which instruction(s) should the nurse give to a female client who just received a prescription for oral metronidazole (Flagyl) for treatment of trichomonas vaginalis? (Select all that apply.) A) Increase fluid intake, especially cranberry juice. B) Do not abruptly discontinue the medication; taper use. C) Check blood pressure daily to detect hypertension. D) Avoid drinking alcohol while taking this medication. E) Use condoms until treatment is completed. F) Ensure that all sexual partners are treated at the same time. A) Increase fluid intake, especially cranberry juice. D) Avoid drinking alcohol while taking this medication. E) Use condoms until treatment is completed. F) Ensure that all sexual partners are treated at the same time.

Answer: A,C RATIONALE: Cephalosporins may enhance bleeding tendencies, so drugs such as aspirin that may promote bleeding should be avoided. Cephalosporins may be taken with food, and they are safe to take if a patient has lactose intolerance. Severe diarrhea should be reported, because it may indicate the development of C. difficile infection. Any indication of an allergic reaction, including a rash, should be reported to the healthcare provider.

Which instructions will the nurse include when teaching a patient about cephalosporin therapy? (Select all that apply.) A "Notify your healthcare provider if you develop diarrhea." B "Take aspirin if you develop a headache." C "Notify your healthcare provider if you develop a rash." D "Cephalosporins may not be taken with food." E "Do not take cephalosporins if you have lactose intolerance."

ANS: B Sulfonamides are bacteriostatic, not bactericidal. They are not derived from biologic substances. They are not antifungals or antivirals. They act by decreasing bacterial synthesis of folic acid.

Which is a characteristic that distinguishes sulfonamides from other drugs used to treat bacterial infection? a. Sulfonamides are bactericidal. b. Sulfonamides are derived from biologic substances. c. Sulfonamides have antifungal and antiviral properties. d. Sulfonamides increase bacterial synthesis of folic acid.

Answer: C RATIONALE: The Controlled Substances Act of 1970 set rules covering drugs of abuse and defined categories of controlled substances.

Which legislation set rules for the manufacture and distribution of drugs considered to have the potential for abuse? A Food, Drug, and Cosmetic Act of 1938 B Harris-Kefauver Amendments of 1962 C Controlled Substances Act of 1970 D Food and Drug Administration Modernization Act of 1997

Answer: A,B,D RATIONALE: Hydrochlorothiazide and furosemide promote hyperglycemia, and metoprolol suppresses glycogenolysis and can mask signs of hypoglycemia. Therefore, these medications should be administered with caution to patients with diabetes. Diltiazem and enalapril do not cause either of these effects.

Which medication or medications should be used with caution in a hypertensive diabetic patient? (Select all that apply.) A Furosemide (Lasix) B Metoprolol (Lopressor) C Diltiazem (Cardizem) D Hydrochlorothiazide (HCTZ) E Enalapril (Vasotec)

Answer: A RATIONALE: The first-pass effect is a pharmacokinetic phenomenon that is related to the drug's metabolism in the liver. After oral (A) medications are absorbed from the gastrointestinal tract, the drug is carried directly to the liver via the hepatic portal circulation where hepatic inactivation occurs and reduces the bioavailability of the drug. Alternative method of administration, such as sublingual (B), IV (C), and subcutaneous (D) routes, avoid this first-pass effect.

Which method of medication administration provides the client with the greatest first-pass effect? A) Oral. B) Sublingual. C) Intravenous. D) Subcutaneous.

Answer: B RATIONALE: Heparin should be administered subcutaneously into the fatty layer of the abdomen with a ½- to ⅝-inch needle, 25 or 26 gauge. The only appropriate option for injection in the list shown is the ⅝-inch, 25-gauge needle.

Which needle length and gauge should the nurse choose to administer subcutaneous heparin? A ½ inch; 20 gauge B ⅝ inch; 25 gauge C 1½ inch; 18 gauge D 1 inch; 26 gauge

Answer: A RATIONALE: In women, trichomoniasis may be asymptomatic or may cause a diffuse, malodorous, yellow-green vaginal discharge, along with burning and itching sensations. Painful urination and watery discharge are associated with infection with herpes simplex virus type 2. Pain in the back of the testicles is the primary symptom of acute epididymitis, which is caused by gonorrhea or Chlamydia organisms. Genital and perianal warts are caused by human papilloma viruses (HPVs).

Which outcome would a nurse establish for a patient with trichomoniasis who is receiving metronidazole (Flagyl)? A Decrease in yellow-green, odorous vaginal discharge B Absence of painful urination and watery discharge C Improvement in pain in the back of the testicles D Resolution of genital and perianal warts

ANS: A Opioids are titrated for oncology patients until pain relief is achieved or the side effects become intolerable, and extremely high doses may be required. Patient with closed head injuries should receive reduced doses of opioids if at all to reduce the risk of increased intracranial pressure. Patients with hypotension should receive reduced doses to prevent further decrease in blood pressure. Patients who are 3 days post operation should not be experiencing severe pain.

Which patient may require a higher than expected dose of an opioid analgesic? a. A patient with cancer b. A patient with a concussion c. A patient with hypotension d. A patient 3 days after surgery

Answer: B Rationale: Patients taking histamine2-receptor blocking agents should avoid spicy foods, extremes in temperatures, alcohol, and smoking. They should also increase bulk and fluids in their diets to prevent constipation. The medications should be taken with meals, not after meals.

Which statement demonstrates to the nurse that the patient understands instructions regarding the use of histamine2-receptor antagonists? A "Since I am taking this medication, it is all right for me to eat spicy foods." B "Smoking decreases the effects of the medication, so I should try a cessation program." C "I should take this medication 1 hour after each meal to decrease gastric acidity." D "I should decrease bulk and fluids in my diet to prevent diarrhea."

ANS: B Corticosteroids decrease inflammatory and immune responses in many ways, including preventing the synthesis of mediators. Both inhaled corticosteroids and those taken orally are preventive; they are not effective in reversing symptoms during an asthma attack and should not be used as rescue drugs. Systemic corticosteroids, because of severe side effects, are avoided for mild to moderate intermittent asthma and are used on a short-term basis for moderate asthma.

Which statement indicates that the client understands teaching about the correct use of a corticosteroid medication? a. "This drug can reverse my symptoms during an asthma attack." b. "This drug is effective in decreasing the frequency of my asthma attacks." c. "This drug can be used most effectively as a rescue agent." d. "This drug can safely be used on a long-term basis for multiple applications daily."

Answer: B RATIONALE: Postural hypotension is common early in treatment, so the patient should be instructed to change positions slowly. Administration with meals should be avoided, if possible, because food delays the absorption of the levodopa component. If the patient is experiencing side effects of nausea and vomiting, administration with food may need to be considered. The levodopa component in Sinemet may darken the color of the urine. Carbidopa has no adverse effects of its own.

Which statement should the nurse include in the teaching plan for a patient being started on levodopa/carbidopa (Sinemet) for newly diagnosed Parkinson's disease? A Take the medication on a full stomach. B Change positions slowly. C The drug may cause the urine to be very dilute. D Carbidopa has many adverse effects.

Answer: A,B,C RATIONALE: Patients who are allergic to penicillin are able to take vancomycin. The major toxicity of vancomycin therapy is kidney failure. The other three statements are true.

Which statements about vancomycin (Vancocin) does the nurse identify as true? (Select all that apply.) A Vancomycin is the most widely used antibiotic in U.S. hospitals. B Vancomycin is effective in the treatment of Clostridium difficile infection. C Vancomycin is effective in the treatment of MRSA infections. D Patients who are allergic to penicillin are also allergic to vancomycin. E The major toxicity of vancomycin therapy is liver failure.

Answer: B RATIONALE: Numbness and tingling in the extremities is associated with the development of peripheral neuropathy and should be reported to the healthcare provider. Rifampin, not isoniazid, causes discoloration of body fluids. Ethambutol, not isoniazid, is associated with optic neuritis. The other two statements are true and can be included in patient teaching.

Which statements will the nurse include when teaching a patient about isoniazid therapy for the treatment of tuberculosis? (Select all that apply.) A "Take the isoniazid on an empty stomach." B "Notify your healthcare provider if your skin starts to turn yellow." C "Numbness or tingling in your extremities is a normal response when taking this drug." D "You urine will turn reddish orange because of the effects of this drug." E "Use of this drug is associated with vision problems." A "Take the isoniazid on an empty stomach."

Answer: B) RATIONALE: (B) represents the most serious adverse effects of beta-blocking agents. AV block is generally associated with bradycardia and results in potentially life-threatening decreases in cardiac output. Additionally, wheezing secondary to bronchospasm and hypotension represent life-threatening respiratory and cardiac disorders. (A, C, and D) are not associated with beta-blockers.

Which symptoms are serious adverse effects of beta-adrenergic blockers such as propranolol (Inderal)? A) Headache, hypertension, and blurred vision. B) Wheezing, hypotension, and AV block. C) Vomiting, dilated pupils, and papilledema. D) Tinnitus, muscle weakness, and tachypnea.

Answer: B RATIONALE: (B) represents the most serious adverse effects of beta-blocking agents. AV block is generally associated with bradycardia and results in potentially life-threatening decreases in cardiac output. Additionally, wheezing secondary to bronchospasm and hypotension represent life-threatening respiratory and cardiac disorders. (A, C, and D) are not associated with beta-blockers.

Which symptoms are serious adverse effects of beta-adrenergic blockers such as propranolol (Inderal)? A) Headache, hypertension, and blurred vision. B) Wheezing, hypotension, and AV block. C) Vomiting, dilated pupils, and papilledema. D) Tinnitus, muscle weakness, and tachypnea.

Answer: B RATIONALE: Cross allergies exist between penicillins and cephalosporins (B). Penicillin allergies are unrelated to allergies associated with (A, C, or D).

While taking a nursing history, the client states, "I am allergic to penicillin." What related allergy to another type of antiinfective agent should the nurse ask the client about when taking the nursing history? A) Aminoglycosides. B) Cephalosporins. C) Sulfonamides. D) Tetracyclines.

Answer: B RATIONALE: Metoprolol is a second-generation beta blocker and as such is more selective. At therapeutic doses, it causes less bronchoconstriction and suppression of glycogenolysis, which can cause problems in diabetic patients. Propranolol blocks both beta1 and beta2 receptors.

Why does the nurse anticipate administering metoprolol (Lopressor) rather than propranolol (Inderal) for diabetic patients who need a beta-blocking agent? A Metoprolol is less likely to cause diabetic nephropathy. B Propranolol causes both beta1 and beta2 blockade. C Metoprolol helps prevent retinopathy in individuals with diabetes. D Propranolol is associated with a higher incidence of foot ulcers.

Answer: A,C,D

he nurse is providing education to a patient on the primary uses of cholinergic drugs. Which uses would the nurse include in the teaching? (Select all that apply.) A To stimulate peristalsis B To elevate heart rate C To decrease intraocular pressure D To stimulate bladder emptying E To dilate pulmonary airways

Answer: C RATIONALE: Nosebleeds and sore throats may be a sign of blood dyscrasias and should be reported to the health care provider. A reddish-pink discoloration of the urine may be expected. To prevent injury to the gums, a soft toothbrush should be utilized. Orthostatic hypertension is not associated with phenytoin use.

(22/30) The nurse is preparing discharge teaching for a patient who has been started on phenytoin for a seizure disorder. What information about side effects of this medication should the nurse provide to the patient and his family? a. "There may be a green discoloration of the patient's urine." b. "It is best to use a hard-bristle toothbrush for dental care." c. "Nosebleeds and sore throats should be reported to the health care provider." d. "The patient should get up slowly to prevent fainting."

Answer: A

(22/31) What would the nurse expect to see if the patient is experiencing a common side effect of phenytoin? a. gingival hyperplasia b. excessive thirst c. weight gain d. muscle tremors

Answer: A,B,C

(22/33) Which statement(s) is/are true about seizures and anticonvulsant use in pregnancy? (Select all that apply) a. Seizures may increase up to 25% in epileptic women. b. Many anticonvulsants have teratogenic properties. c. Anticonvulsant use increases loss of folic acid. d. Anticonvulsants increase the effects of vitamin K. e. Valproic acid causes malformation in 40% to 80% of fetuses

Answer: A RATIONALE: Warfarin is an anticoagulant that may be taken by patients with atrial fibrillation. Aspirin displaces warfarin from its protein-binding site. This increases the anticoagulant effect and may lead to excessive bleeding, which may initially be indicated by bruising. Although the other medications may have drug-drug interactions, they do not cause increased anticoagulant effects, as evidenced by the ebruising.

(25/19) A patient with a complicated medical history including hypertension, atrial fibrillation, and arthritis calls the health care provider's office to speak with a nurse about "all of these bruises I have all of a sudden." Which potential drug interaction should concern the nurse with these symptoms? a. Aspirin and warfarin b. Sulfasalazine and acetaminophen c. Tolmetin and propanolol d. Meloxicam and amplodipine

Answer: D

(25/20) A father presents to the emergency department with his 4 y/o son. The father explains that his son had a fever, so he gave the child baby aspirin to decrease the fever and it has not worked. What should concern the nurse about a 4 y/o receiving aspirin? a. Aspirin has the potential to cause GI bleeding in children. b. Aspirin has the potential to cause ringing in the ears in children. c. Aspirin has the potential to cause hyperglycemia in children. d. Aspirin has the potential to cause Reye's syndrome in children.

Answer: C RATIONALE: Drug toxicity is an adverse drug reaction in which certain drugs are toxic to specific organs. Signs and symptoms of liver toxicity include jaundice, dark urine, light-colored stools, nausea, vomiting, malaise, abdominal discomfort, and loss of appetite.

1. The nurse is caring for a patient who has jaundice, dark urine, malaise, light-colored stools, nausea, and vomiting. This patient is most likely experiencing what? A. An idiosyncratic drug effect on the bone marrow B. Iatrogenic disease of the kidneys C. Drug toxicity of the liver D. An allergic reaction

Answer: B RATIONALE: Drug accumulation secondary to reduced renal excretion is the most important cause of ADRs in the elderly. Creatinine clearance, not serum creatinine levels, is the proper index of renal function in older adult patients.

1. The nurse will monitor which laboratory result closely when administering medications to an older adult patient while assessing for adverse drug reactions (ADRs)? A. Serum creatinine levels B. Creatinine clearance C. Serum albumin levels D. Liver function tests

Answer: 1 RATIONALE: Digoxin is a cardiac glycoside that is used to treat heart failure and acts by increasing the force of myocardial contraction. Because bradycardia may be a clinical sign of toxicity, the nurse counts the apical heart rate for 1 full minute before administering the medication. If the pulse rate is less than 60 beats/minute in an adult client, the nurse would withhold the medication and report the pulse rate to the registered nurse, who would then contact the health care provider.

125.) A nurse is preparing to administer digoxin (Lanoxin), 0.125 mg orally, to a client with heart failure. Which vital sign is most important for the nurse to check before administering the medication? 1. Heart rate 2. Temperature 3. Respirations 4. Blood pressure

Answer: 1 RATIONALE: Albuterol is a bronchodilator. Beclomethasone dipropionate is a glucocorticoid. Bronchodilators are always administered before glucocorticoids when both are to be given on the same time schedule. This allows for widening of the air passages by the bronchodilator, which then makes the glucocorticoid more effective.

145.) A nurse has a prescription to give a client albuterol (Proventil HFA) (two puffs) and beclomethasone dipropionate (Qvar) (nasal inhalation, two puffs), by metered-dose inhaler. The nurse administers the medication by giving the: 1. Albuterol first and then the beclomethasone dipropionate 2. Beclomethasone dipropionate first and then the albuterol 3. Alternating a single puff of each, beginning with the albuterol 4. Alternating a single puff of each, beginning with the beclomethasone dipropionate

Answer: 2 RATIONALE: Theophylline is a xanthine bronchodilator. The nurse teaches the client to limit the intake of xanthine-containing foods while taking this medication. These include coffee, cola, and chocolate.

146.) A client has begun therapy with theophylline (Theo-24). The nurse tells the client to limit the intake of which of the following while taking this medication? 1. Oranges and pineapple 2. Coffee, cola, and chocolate 3. Oysters, lobster, and shrimp 4. Cottage cheese, cream cheese, and dairy creamers

Answer: 2 RATIONALE: The client taking a single daily dose of theophylline, a xanthine bronchodilator, should take the medication early in the morning. This enables the client to have maximal benefit from the medication during daytime activities. In addition, this medication causes insomnia. The client should take in at least 2 L of fluid per day to decrease viscosity of secretions. The client should check with the physician before changing brands of the medication. The client also checks with the HCP before taking OTC cough, cold, or other respiratory preparations because they could cause interactive effects, increasing the side effects of theophylline and causing dysrhythmias.

147.) A client with a prescription to take theophylline (Theo-24) daily has been given medication instructions by the nurse. The nurse determines that the client needs further information about the medication if the client states that he or she will: 1. Drink at least 2 L of fluid per day. 2. Take the daily dose at bedtime. 3. Avoid changing brands of the medication without health care provider (HCP) approval. 4. Avoid over-the-counter (OTC) cough and cold medications unless approved by the HCP.

Answer: 3 RATIONALE: Double vision, loss of appetite, and nausea are signs of digoxin toxicity. Additional signs of digoxin toxicity include bradycardia, difficulty reading, visual alterations such as green and yellow vision or seeing spots or halos, confusion, vomiting, diarrhea, decreased libido, and impotence. *gastrointestinal (GI) and visual disturbances occur with digoxin toxicity*

150.) A client complaining of not feeling well is seen in a clinic. The client is taking several medications for the control of heart disease and hypertension. These medications include a β-blocker, digoxin (Lanoxin), and a diuretic. A tentative diagnosis of digoxin toxicity is made. Which of the following assessment data would support this diagnosis? 1. Dyspnea, edema, and palpitations 2. Chest pain, hypotension, and paresthesia 3. Double vision, loss of appetite, and nausea 4. Constipation, dry mouth, and sleep disorder

Answer: 1 RATIONALE: The antidote to heparin is protamine sulfate; it should be readily available for use if excessive bleeding or hemorrhage occurs. Potassium chloride is administered for a potassium deficit. Vitamin K is an antidote for warfarin sodium. Aminocaproic acid is the antidote for thrombolytic therapy.

152.) Intravenous heparin therapy is prescribed for a client. While implementing this prescription, a nurse ensures that which of the following medications is available on the nursing unit? 1. Protamine sulfate 2. Potassium chloride 3. Phytonadione (vitamin K ) 4. Aminocaproic acid (Amicar)

Answer: C

3. Which of the following statements about Aspirin is TRUE? A. Aspirin has no ototoxic characteristics B. Aspirin is least likely to cause gastric irritation. C. Aspirin should generally be avoided for 1 to 2 weeks before and after surgery D. enteric-coated Aspirin (Ecospirin) should be chewed or crushed before swallowing.

Answer: 2 RATIONALE: INH is hepatotoxic, and therefore the client is taught to report signs and symptoms of hepatitis immediately (which include yellow skin and sclera). For the same reason, alcohol should be avoided during therapy. The client should avoid intake of Swiss cheese, fish such as tuna, and foods containing tyramine because they may cause a reaction characterized by redness and itching of the skin, flushing, sweating, tachycardia, headache, or lightheadedness. The client can avoid developing peripheral neuritis by increasing the intake of pyridoxine (vitamin B6) during the course of INH therapy for TB.

48.) A client is to begin a 6-month course of therapy with isoniazid (INH). A nurse plans to teach the client to: 1. Drink alcohol in small amounts only. 2. Report yellow eyes or skin immediately. 3. Increase intake of Swiss or aged cheeses. 4. Avoid vitamin supplements during therapy.

Answer: 3 RATIONALE: INH therapy can cause an elevation of hepatic enzyme levels and hepatitis. Therefore, liver enzyme levels are monitored when therapy is initiated and during the first 3 months of therapy. They may be monitored longer in the client who is greater than age 50 or abuses alcohol.

52.) A client with tuberculosis is being started on antituberculosis therapy with isoniazid (INH). Before giving the client the first dose, a nurse ensures that which of the following baseline studies has been completed? 1. Electrolyte levels 2. Coagulation times 3. Liver enzyme levels 4. Serum creatinine level

Answer: 2 RATIONALE: Therapeutic levels for digoxin range from 0.5 to 2 ng/mL. Therefore, options 1, 3, and 4 are incorrect.

55.) A client who is receiving digoxin (Lanoxin) daily has a serum potassium level of 3.0 mEq/L and is complaining of anorexia. A health care provider prescribes a digoxin level to rule out digoxin toxicity. A nurse checks the results, knowing that which of the following is the therapeutic serum level (range) for digoxin? 1. 3 to 5 ng/mL 2. 0.5 to 2 ng/mL 3. 1.2 to 2.8 ng/mL 4. 3.5 to 5.5 ng/mL

Answer: 4 RATIONALE: Metformin (Glucophage) needs to be withheld 24 hours before and for 48 hours after cardiac catheterization because of the injection of contrast medium during the procedure. If the contrast medium affects kidney function, with metformin in the system, the client would be at increased risk for lactic acidosis. The medications in the remaining options do not need to be withheld 24 hours before and 48 hours after cardiac catheterization.

A client admitted to the hospital with chest pain and a history of type 2 diabetes mellitus is scheduled for cardiac catheterization. Which medication would need to be withheld for 24 hours before the procedure and for 48 hours after the procedure? 1. Regular insulin 2. Glipizide (Glucotrol) 3. Repaglinide (Prandin) 4. Metformin (Glucophage)

Answer: B

A client is prescribed theophylline to relax the smooth muscles of the bronchi. The nurse monitors the client's theophylline serum levels to maintain which therapeutic range? a. 1 to 10 mcg/mL b. 10 to 20 mcg/mL c. 20 to 30 mcg/mL d. 30 to 40 mcg/mL

Answer: B

A client with COPD has an acute bronchospasm. The nurse knows that which is the best medication for this emergency situation? a. zafirlukast (Accolate) b. epinephrine (Adrenalin) c. dexamethasone (Decadron) d. oxtriphylline-theophyllinate (Choledyl)

Answer: A

A client with COPD is taking a leukotriene antagonist, montelukast (Singulair). The nurse is aware that this medication is given for which purpose? a. Maintenance treatment of asthma b. Treatment of an acute asthma attack c. Reversing bronchospasm associated with COPD d. Treatment of inflammation in chronic bronchitis

Answer: C

A client with a history of asthma is short of breath and says, "I feel like I'm having an asthmatic attack." What is the nurse's best action? a. Call a code. b. Ask the client to describe the symptoms. c. Administer a beta2 adrenergic agonist. d. Administer a long-acting glucocorticoid.

Answer: B RATIONALE: To move through the body, drugs must cross membranes. They cross membranes to enter the bloodstream, to exit the bloodstream and reach the site of action, and to undergo metabolism and excretion. Selectivity and effectiveness are not related to drug movement. Development of an electric charge (ionization) reduces a drug's ability to be absorbed. Transporter proteins are not required for drugs to move through the body.

A new graduate nurse, who is preparing to administer medications, knows that what is required for a drug to move through the body? A. Selectivity and effectiveness B. The ability to cross membranes C. Development of an electric charge D. A transporter protein

Answer: A

A nurse reviews a client's medication history and notes that the client is taking a nonselective adrenergic agonist bronchodilator and has a history of coronary artery disease. What is a priority nursing intervention? a. Monitor client for potential chest pain. b. Monitor blood pressure continuously. c. Assess daily for hyperkalemia. d. Assess 12-lead ECG each shift.

Answer: B

A patient receiving intravenous nitroglycerin at 20 mcg/min complains of dizziness. Nursing assessment reveals a blood pressure of 85/40 mm Hg, heart rate of 110 beats/min, and respiratory rate of 16 breaths/min. What is the nurse's best action? A Assess the patient's lung sounds. B Decrease the intravenous nitroglycerin by 10 mcg/min. C Increase the intravenous nitroglycerin by 10 mcg/min. D Recheck the patient's vital signs in 1 hour.

Answer: B Distribution is the process by which the drug becomes available to body fluids and body tissues. Drug distribution is influenced by blood flow, the drug's affinity to tissue, and the protein-binding effect. This distribution can be affected by the effects of edema in the fluid overload phase.

A patient sustains significant burns to the skin and is experiencing fluid shift associated with edema in the fluid overload phase. The nurse would anticipate that this will interfere most with which phase of pharmacodynamics? a. Absorption b. Distribution c. Metabolism d. Excretion

Answer: C

A patient who is taking sublingual nitroglycerin is complaining of flushing and headaches. What is the nurse's best response? A "This is a normal response to your chest pain." B "Stop taking the nitroglycerin because you are probably allergic to it." C "These are the most common side effects of nitroglycerin. They should subside with continued use of nitroglycerin." D "These symptoms are not related to your sublingual nitroglycerin. You should notify your doctor for diagnostic testing."

Answer: A RATIONALE: Warfarin is an anticoagulant with a high affinity for binding with albumin. If the albumin level is low, more free drug is available for action, resulting in an increased prothrombin time/international normalized ratio (PT/INR). Deep vein thromboses can be prevented with warfarin. An increased risk of bruising and bleeding would occur with more free drug available. Warfarin acts on vitamin K, not on platelets. Aspirin is an example of an antiplatelet aggregator.

The nurse is administering warfarin, an anticoagulant, to a patient with a low albumin level. As a result, the nurse can expect to observe which effect of this medication? A. Increased PT/INR levels B. Deep vein thromboses C. Reduced risk of bruising D. Increased platelet aggregation

Answer: C

The nurse is caring for a client with a theophylline level of 14 mcg/mL. What is the priority nursing intervention? a. Increase the IV drip rate. b. Monitor the client for toxicity. c. Continue to assess the client's oxygenation. d. Stop the IV for an hour then restart at lower rate.

Answer: C RATIONALE: Acetylcholine (ACh) levels naturally decline by a small percentage with age. Patients with severe AD may have ACh levels that are as much as 90% below normal. This is likely part of the explanation for the pathophysiology of AD.

The nurse is caring for a group of patients diagnosed with Alzheimer's disease (AD). Which neurotransmitter level is decreased by as much as 90% in patients with severe AD? A Norepinephrine B Serotonin C Acetylcholine D Dopamine

Answer: A,B,D RATIONALE: Adverse effects associated with phenytoin at therapeutic doses include mild sedation, gingival hyperplasia (swollen, tender gums), morbilliform (measles-like) rash, cardiovascular effects, and other effects, such as hirsutism (unusual hair growth) and interference with vitamin D metabolism.

The nurse is caring for a patient receiving phenytoin (Dilantin) for treatment of tonic-clonic seizures. Which symptoms, if present, would indicate an adverse effect of this drug? (Select all that apply.) A Swollen, tender gums B Measles-like rash C Productive cough D Unusual hair growth E Nausea and vomiting

Answer: B RATIONALE: Toxicity is an adverse drug reaction caused by excessive dosing. A side effect is a nearly unavoidable secondary drug effect produced at a therapeutic dose. An allergic reaction is an immune response. An idiosyncratic effect is an uncommon drug response resulting from a genetic predisposition.

The nurse is caring for a patient who is experiencing a respiratory rate of 6 breaths per minute as a result of a large dose of pain medication. Which term most accurately describes this reaction? A. Side effect B. Toxicity C. Allergic reaction D. Idiosyncratic reaction

Answer: B RATIONALE: Gastric distress is a common problem with uncoated aspirin. Enteric-coated tablets can be used. Changing to another medication or taking the medication with milk are extreme measures that may not relieve the gastric distress.

The nurse is caring for a patient who states, "I can't take aspirin. It makes my stomach hurt." What is the nurse's best response to the patient? A. "You should take ibuprofen instead." B. "You can try enteric-coated aspirin." C. "I will see if you can get a prescription for another pain reliever." D. "Try taking the aspirin with milk."

Answer: D

The nurse is caring for clients on the pulmonary unit. Which client should not receive epinephrine if ordered? a. The client with a history of emphysema b. The client with a history of type 2 diabetes c. The client who is 16 years old d. The client with atrial fibrillation with a rate of 100

Answer: B

The nurse is conducting a community education program. When explaining different medication regimens to treat hypertension, it would be accurate to state that African Americans probably respond best to which combination of medications? A ACE inhibitors and diuretics B Diuretics and calcium channel blockers C Diuretics and beta blockers D ACE inhibitors and beta blockers

Answer: B,D RATIONAEL: Patients taking an antiepileptic drug are at increased risk for suicidal thoughts and behavior beginning early in their treatment. The U.S. Food and Drug Administration (FDA) advises that patients, families, and caregivers be informed of the signs that may precede suicidal behavior and be encouraged to report these immediately. Mild sedation can occur in patients taking phenytoin, even at therapeutic levels. Carbamazepine, not phenytoin, increases the risk for hematologic effects, such as easy bruising. Phenytoin causes gingival hyperplasia in about 20% of patients who take it; dental hygiene is important. Patients receiving phenytoin should avoid alcohol and other central nervous system depressants, because they have an additive depressant effect.

The nurse is conducting discharge teaching related to a new prescription for phenytoin (Dilantin). Which statements are appropriate to include in the teaching for this patient and family? (Select all that apply.) A "Be sure to call the clinic if you or your family notice increased anxiety or agitation." B "You may have some mild sedation. Do not drive until you know how this drug will affect you." C "This drug may cause easy bruising. If you notice this, call the clinic immediately." D "It is very important to have good oral hygiene and to visit your dentist regularly." E "You may continue to have wine with your evening meals, but only in moderation."

Answer: A,C,D,F

The nurse is monitoring a patient who is receiving an infusion of a beta-adrenergic agonist. Which adverse effects may occur w/this infusion? (select all that apply) a) mild tremors b) bradycardia c) tachycardia d) palpitations e) drowsiness f) nervousness

Answer: C

The nurse is planning to administer metoclopramide (Reglan). What is a primary intervention? a. Administer with food to decrease gastrointestinal upset. b. Administer every 6 hours around the clock. c. Administer 30 minutes before meals and at bedtime. d. Give with a full glass of water first thing in the morning.

Answer: B RATIONALE: Cell membranes are composed of lipids; therefore, a lipid-soluble drug passes through rapidly. A water-soluble drug passes through more slowly. The nurse would expect to observe the effects of a lipid-soluble drug more quickly, because the drug is absorbed more rapidly.

The nurse is preparing to give a medication for pain. The label states that the drug is "lipid soluble." Based on the nurse's knowledge of lipid-soluble drugs, how quickly would the nurse expect to observe the effects of the drug? A. Slowly B. Rapidly C. Unpredictably D. Variably

Answer: A RATIONALE: The therapeutic range for phenytoin is 10 to 20 mcg/mL. Because this level is within normal limits, the nurse would continue with the routine plan of care.

The nurse receives a laboratory report indicating that the phenytoin (Dilantin) level for the patient seen in the clinic yesterday is 16 mcg/mL. Which intervention is most appropriate? A Continue as planned, because the level is within normal limits. B Tell the patient to hold today's dose and return to the clinic. C Consult the prescriber to recommend an increased dose. D Have the patient call 911 and meet the patient in the emergency department.

Answer: D RATIONALE: Enteric-coated tablets are covered with a material designed to dissolve in the intestine instead of the stomach. They should not be chewed or broken before administration. Sublingual tablets are placed under the tongue for absorption and are not enteric coated.

The nurse should provide which teaching point when administering an enteric-coated oral tablet to a patient? A. "Chew the tablet before swallowing." B. "Break the tablet in half before swallowing." C. "Allow the tablet to be absorbed under the tongue." D. "Swallow the tablet whole after double-checking the dose."

Answer: B

When applying nitroglycerin ointment, the nurse should perform which action? B Use the fingers to spread the ointment evenly over a 3-inch area. B Apply the ointment to a nonhairy part of the upper torso. C Massage the ointment into the skin. D Apply two thick lines of ointment over the prescribed measured area on the nitroglycerin paper.

Answer: B,D,E RATIONALE: Manifestations of phenytoin toxicity can occur when plasma levels are higher than 20 mcg/mL. Nystagmus (back-and-forth movement of the eyes) is a common indicator of toxicity, as are ataxia (staggering gait), diplopia (double vision), sedation, and cognitive impairment. Hirsutism (excess hair growth in unusual places) and gingival hyperplasia (swollen, tender, bleeding gums) are adverse effects of phenytoin.

The nurse suspects that a female patient is experiencing phenytoin toxicity if which manifestation is noted? (Select all that apply.) A The patient complains of excessive facial hair growth. B The patient is walking with a staggering gait. C The patient's gums are swollen, tender, and bleed easily. D The patient complains of double vision. E The nurse observes rapid back-and-forth movement of the patient's eyes.

Answer: B

The patient asks how nitroglycerin should be stored while traveling. What is the nurse's best response? A "You can protect it from heat by placing the bottle in an ice chest." B "It's best to keep it in its original container away from heat and light." C "You can put a few tablets in a resealable bag and carry in your pocket." D "It's best to lock them in the glove compartment of your car to keep them away from heat and light."

Answer: A

The priority nursing diagnosis for a patient taking metoprolol (Lopressor) would be A. Risk for decreased cardiac tissue perfusion related to effects of medication. B. Acute confusion related to adverse central nervous system effects of the drug. C. Deficient knowledge related to therapeutic regimen. D. Risk for injury related to possible side effects of the adrenergic blockers

Answer: C

The provider has ordered donepezil (Aricept) for the patient, and the patient states "I have no idea why I take this medication." What is the most common diagnosis associated with the administration of donepezil (Aricept)? A.Parkinson's disease B. Bladder retention C. Alzheimer's disease D. Urinary retention

Answer: D

When assessing for cardiovascular effects of a beta blocker, the nurse understands that these drugs produce A a positive inotropic, positive chronotropic, and positive dromotropic effect. B a positive inotropic, negative chronotropic, and negative dromotropic effect. C a negative inotropic, positive chronotropic, and positive dromotropic effect. D a negative inotropic, negative chronotropic, and negative dromotropic effect.

Answer: C Rationale: Creatinine clearance is the most accurate test to determine renal function. Creatinine is a metabolic byproduct of muscle that is excreted by the kidneys. Creatinine clearance varies with age and gender. Lower values are expected in older adult and female patients because of their decreased muscle mass. A decrease in renal GFR (common in older adults) results in a decrease in urine creatinine clearance.

When assessing older adults and those with renal dysfunction, the nurse knows that creatinine clearance is usually a. substantially increased. b. slightly increased. c. decreased. d. in the normal range.

Answer: 1

When monitoring the plasma drug levels in a patient taking theophylline (Theo-24), which of the following adverse effects should the HCP see if the patients level is toxic? 1. Seizures 2. Constipation 3. Vomiting 4. Restlessness

Answer: C RATIONALE: Pharmacodynamics can be thought of as the impact of drugs on the body. Pharmacokinetics describes the movement of drugs through the body. Selectivity is the ability of a drug to elicit only the response for which it is given. Predictability is the degree of certainty about how a patient will respond to a certain drug.

When studying the impact a drug has on the body, the nurse is reviewing what? A. The drug's pharmacokinetics B. The drug's selectivity C. The drug's pharmacodynamics D. The drug's predictability

Answer: D

When teaching a patient about beta blockers such as atenolol (Tenormin) and metoprolol (Lopressor), it is important to inform the patient that A. these medications may be taken with antacids to minimize gastrointestinal distress. B. hot baths and showers will help enhance the therapeutic effects and are encouraged. C. alcohol intake is encouraged for its vasodilating effects. D. abrupt medication withdrawal may lead to a rebound hypertensive crisis.

Answer: A RATIONALE: A major side effect of NSAID therapy is gastrointestinal (GI) distress with potential GI bleeding. Black, tarry stools are indicative of a GI bleed. Headaches, cough, and palpitations should not result from the use of NSAID medications.

Which assessment finding in a patient taking NSAIDs requires immediate intervention? A. Black, tarry stools B. Headache C. Nonproductive cough D. Palpitations

Answer: B

Which body organ is the major site of drug metabolism? a. kidney b. liver c. lung d. skin

Answer: D RATIONALE: Ketorolac (Toradol) is the only NSAID that can be administered by injection (intramuscularly or intravenously) and is indicated for short-term use for severe-to-moderate pain. Acute postoperative pain cannot be effectively managed on oral medication

Which intervention is most appropriate for a patient who needs treatment for acute postoperative pain? A. Administer allopurinol (Zyloprim) PO every 4 hours. B. Administer celecoxib (Celebrex) PO every 6 hours. C. Administer indomethacin (Indocin) PO every 4 hours. D. Administer ketorolac (Toradol) IV every 4 hours PRN.

Answer: 4

Which should a healthcare professional include when advising a patient about montelukast (Singulair)? 1. Use spacer to improve inhalation 2. Take the drug at the onset of bronchospasm 3. Rinse the mouth daily the prevent an oral fungal infection 4. Take the drug once a day in the evening

Answer: D

Which statement by the patient demonstrates a need for further education regarding nitroglycerin? A "If I get a headache, I should keep taking my nitroglycerin and use Tylenol for pain relief." B "I should keep my nitroglycerin in a cool, dry place." C "I should change positions slowly to avoid getting dizzy from the nitroglycerin's effect on my blood pressure." D "I can take up to four tablets at 5-minute intervals for chest pain."

Answer: D

Which statement places the four process of pharmacokinetics in the correct sequence? a. absorption, metabolism, distribution, excretion b. distribution, absorption, metabolism, excretion c. distribution, metabolism, absorption, excretion d. absorption, distribution, metabolism, excretion

Answer: D

Which type of drug can be eliminated through the kidneys? a. enteric-coated b. lipid-soluble c. protein-bound d. water-soluble

Answer: B

Which type of drug passes rapidly through the GI membrane? a. lipid-soluble and ionized b. lipid-soluble and nonionized c. water-soluble and ionized d. water-soluble and nonionized


Kaugnay na mga set ng pag-aaral

Chapter 18: Open-Economy Macroeconomics

View Set

Professional Scrum Master 1, 2, and 3

View Set

MTA 98-364 Database Administration Fundamentals

View Set

Chapter 6 - Life Span Development

View Set